You are on page 1of 379

‫שנה ד'‬

‫הקורס האינטגרטיבי‬

‫חוברת מבחנים‬

‫‪2008-2009‬‬
‫מבואות‬

‫* משנת ‪ 2006‬המבואות נכללים בבחינת מקבץ ‪1‬‬


‫מקבץ ‪I‬‬
‫מקבץ ‪2007-2008 - I‬‬
‫חלק א' ‪ -‬מבואות‬
‫‪ .1‬מה נכון לגבי הדמיה ברפואה גרעינית ?‬
‫א‪ .‬ההדמיה מבוססת על מעקב באמצעות גלאי חיצוני אחר דפוס הפיזור בגוף של חומר‬
‫רדיואקטיבי המוחדר לגוף‬
‫ב‪ .‬ההדמיה היא הדמיה פונקציונלית כיוון שהיא מספקת מידע על שינויים הקורים‬
‫בתגובה לחומר המוזרק‬
‫ג‪ .‬כמות הקרינה אותה מקבל הנבדק בבדיקת ‪ SPECT‬גבוהה יותר מהכמות בבדיקה‬
‫פלנרית רגילה של אותו מיפוי‬

‫‪ .2‬איזה משפט נכון לגבי מיפוי כליות ?‬


‫א‪ .‬ניתן לאבחן באמצעות המיפוי דחייה של כליה מושתלת ממת‪ ,‬אך לא ניתן להדגים‬
‫כליה מושתלת מתורם חי‬
‫ב‪ .‬על מנת ללמוד במיפוי על חסימה בניקוז הכלייתי ניתן להיעזר בתרופה כמו פוסיד‬
‫ג‪ .‬מיפוי כליות משמש לזיהוי הישנות של סרטן הערמונית‬
‫ד‪ .‬במיפוי ‪ DMSA‬ניתן לאבחן דלף מכליה מושתלת‬

‫‪ .3‬כל המשפטים הבאים לגבי הדמיה ב‪ FDG PET/CT -‬נכונים פרט ל‪:‬‬
‫א‪ .‬אחד היתרונות החשובים של הבדיקה הוא זיהוי רקמה סרטנית במסה שארית אחרי‬
‫טיפול בסרטן‬
‫ב‪ FDG PET/CT .‬משמשת כבדיקה סקר לזיהוי ואבחון ראשוני של סרטן המעי הגס‬
‫ג‪ FDG .‬נקלט באופן פיזיולוגי ברקמת המוח ולעיתים במיוקרד‪ ,‬ומופרש בשתן‬
‫ד‪ .‬הדמיה בטכנולוגית ‪ PET‬מבוססת על דטקציה בו זמנית של שני קרני ‪ γ‬הנפלטות ב‪-‬‬
‫‪ 180°‬זו לזו‪ ,‬לאחר תהליך אנהילציה של פוזיטרון הנפלט מאיזוטופ רדיואקטיבי עם‬
‫אלקטרון‬

‫‪ .4‬תוצאות בדיקת סידן בדם תושפע מ‪:‬‬


‫א‪ .‬רמה נמוכה של אנזימים הפועלים באמצעות סידן‬
‫ב‪ .‬שימוש בהפרין כנוגד קרישה‬
‫ג‪ .‬שימוש ב‪ EDTA-‬כנוגד קרישה‬
‫ד‪ .‬בפלסמה רמת הסידן נמוכה יותר מאשר בסרום‬

‫‪ .5‬במדידה של רמות אשלגן בדם של חולה התקבלו התוצאות הבאות )חזרות על אותה‬
‫דגימה(‪ .4.8 ,4.9 ,4.8 ,4.7 ,4.9 ,4.8 mEq/L :‬ניתן להסיק ש‪:‬‬
‫א‪ .‬שיטה זו בעלת דיוק )‪ (accuracy‬גבוה‬
‫ב‪ .‬שיטה זו בעלת דיוק נמוך‬
‫ג‪ .‬שיטה זו בבעלת הדירות גבוהה )‪(precision‬‬
‫ד‪ .‬שיטה זו בעלת חדירות נמוכה‬
‫ה‪ .‬לא ניתן להסיק מסקנות על חדירות ודיוק השיטה לפני נתונים אלה‬

‫‪ .6‬לגבי סמן נתון‪ ,‬כאשר בודקים רגישות וסגוליות ב‪ cutoff-‬נתון‪:‬‬


‫א‪ .‬כאשר מעלים ‪ cut-off‬הרגישות עולה‬
‫ב‪ .‬כאשר מעלים ‪ cut-off‬הרגישות יורדת‬
‫ג‪ .‬כאשר מעלים ‪ cut-off‬אין השפעה על הרגישות‬
‫ד‪ .‬אין תשובה נכונה‬
‫‪ .7‬בדם המוליטי לא ניתן לבדוק את ערכי‪:‬‬
‫א‪ .‬גלוקוז‬
‫ב‪ .‬נתרן‬
‫ג‪ .‬סידן‬
‫ד‪ .‬אשלגן‬

‫‪ .8‬מה נכון לגבי רגישות‪:‬‬


‫א‪ .‬רגישות של בדיקה משתנה עם שכיחות המחלה‬
‫ב‪ .‬רגישות של בדיקה איננה משתנה עם שכיחות המחלה‬
‫ג‪ .‬עליה ברגישות בד"כ גורמת לעליה בספציפיות )ייחודיות( של אותה בדיקה‬
‫ד‪ .‬רגישות הבדיקה מבטאת יכולת של תוצאה שלילית לנבא היעדרות מחלה‬

‫‪ .9‬מה המשמעות של חוסר התקדמות גלי ‪ R‬בחיבורים ‪:V1-4‬‬


‫א‪ .‬שכיח מאוד במרימי משקולות‬
‫ב‪ .‬זהו סימן למחלה ריאתית משמעותית‬
‫ג‪ .‬מרמז על אוטם קדמי‪-‬מחיצתי קודם‬
‫ד‪ .‬מרמז על בעיה טכנית בחיבור אלקטרודות האק"ג‬

‫‪ .10‬בתרשים שלפניך מהו הציר החשמלי )תמונה ‪:(1‬‬


‫א‪ .‬מינוס ‪ 45‬מעלות‬
‫ב‪ 0 .‬מעלות‬
‫ג‪ 30 .‬מעלות‬
‫ד‪ 90 .‬מעלות‬

‫‪ .11‬קביעת קצב‪ ,‬ציר ומרווח ‪PR‬‬

‫‪ .12‬קביעת קצב‪ ,‬ציר ומרווח ‪PR‬‬

‫‪– .13‬‬

‫‪- .14‬‬

‫חלק ב' ‪ -‬ריאות‬


‫‪ .15‬מהי הפעילות המניעתית היעילה ביותר למניעת התפשטות דלקת ריאות בבית החולים‪:‬‬
‫א‪ .‬מתן אנטיביוטיקה רחבת טווח כפעילות מניעתית‬
‫ב‪ .‬רחיצת ידיים‬
‫ג‪ .‬מתן טיפול קבוע בחוסמי ‪2H‬‬
‫ד‪ .‬השכבת החולה המונשם על הבטן‬
‫ה‪ .‬השכבת החולה במצב ‪supine‬‬

‫‪ .16‬כל הבאים הינם מחלוללים שכיחים לדלקת ריאות הנרכש בביה"ח לאחר למעלה מ‪5-‬‬
‫ימים של אשפוז פרט ל‪:‬‬
‫א‪ .‬אצינטובקטר‬
‫ב‪ .‬קלבסיאלה מסוג ‪ESBL‬‬
‫ג‪ .‬המופילוס‬
‫ד‪ .‬סטאפילוקוק מסוג ‪MRSA‬‬
‫ה‪ .‬פסאודומונאס‬
‫‪ .17‬תמט )‪ (Atelectasis‬של ריאה אחת גורם ל‪:‬‬
‫א‪ .‬דחיקה של המיצר לצד הנגדי‬
‫ב‪ .‬משיכת מיצר לצד הפתולוגי‬
‫ג‪ .‬אינו משפיע כלל על המיצר‬
‫ד‪ .‬מרים את המיצר כלפי מעלה‬

‫‪ .18‬איזה מהמשפטים הבאים מאפיין תסחיף ריאתי?‬


‫א‪ .‬הסימפטום השכיח הינו גניחת דם )המופטזיס(‬
‫ב‪ .‬ברוב המקרים בבדיקה הפיזיקלית ניתן למצוא סימנים של ‪DVT‬‬
‫ג‪ .‬לרוב החולים יש הפרעות קצב עם שינויים ספציפיים באק"ג‬
‫ד‪ .‬בדיקת גזים עורקיים תקינה שוללת את המחלה‬

‫‪ .19‬גבר בן ‪ 52‬בריא בד"כ‪ ,‬מעשן כבד שהגיע עם המופטזיס‪ .‬צילום חזה‪ CT ,‬ריאה ו‪PET/CT -‬‬
‫הדגימו גידול באונה שמאלית עליונה ללא הגדלת בלוטות וללא פיזור גרורתי‪FNA .‬‬
‫הראה תאים מטיפוס קשקשי עם פניני קרטין‪ .‬טיפול הבחירה יהיה‪:‬‬
‫א‪ .‬הקרנות ואח"כ כמורפיה‬
‫ב‪ .‬כמותרפיה ואח"כ הקרנות‬
‫ג‪ .‬פנאומונקטומיה של ריאה שמאלית‬
‫ד‪ .‬לובקטומיה של ‪LUL‬‬

‫‪ .20‬איזה מהמחלות הבאות גורמות להדיות יתר בניקוש בית החזה?‬


‫א‪Pneumothorax .‬‬
‫‪Pneumonia‬‬ ‫ב‪.‬‬
‫‪Pleural effusion‬‬ ‫ג‪.‬‬
‫אטלקטזיס )תמט(‬ ‫ד‪.‬‬
‫ה‪ .‬כל התשובות נכונות‬
‫‪- .21‬‬
‫‪ .22‬ברור גניחת דם כולל במרבית המקרים את כל הפעולות הבאות פרט ל‪:‬‬
‫א‪ .‬צילום חזה‬
‫בדיקת כיח לתרבית וציטולוגיה‬ ‫ב‪.‬‬
‫ברונכוסקופיה‬ ‫ג‪.‬‬
‫‪CT‬‬ ‫ד‪.‬‬
‫ה‪ .‬אנגיוגרפיה פולמונרית‬

‫‪ .22‬כל אחת מהבדיקות המוזכרות משמשת לקביעת ‪ stage‬של מחלת סרטן הריאות פרט ל‪:‬‬
‫א‪CT .‬‬
‫ב‪PET FDG .‬‬
‫מדיאסטינוסקופיה או מדיאסטינוטומיה‬ ‫ג‪.‬‬
‫ד‪ .‬מיפוי עצמות‬
‫ה‪ .‬בדיקת ‪ US‬של ה‪pleural space-‬‬

‫‪ .23‬כל הבאים הן סיבות שכיחות להמופטיזיס פרט ל‪:‬‬


‫א‪Pulmonary embolism .‬‬
‫ב‪Chronic bronchitis .‬‬
‫‪Tuberculosis‬‬ ‫ג‪.‬‬
‫ד‪ .‬גידול ריאתי‬
‫ה‪ .‬אסטמה‬
‫‪ .24‬בן ‪ 38‬מגיע עם חום ‪ 40°C‬וקשיי נשימה שהחלו כ‪ 3-‬ימים טרם קבלתו‪ .‬בצילום חזה‬
‫נמצאה הצללה של הריאה השמאלית עם סטיית המדיאסטינום לצד ימין‪ .‬האבחנה‬
‫הסבירה היא‪:‬‬
‫א‪ .‬תמט של הריאה השמאלית‬
‫ב‪ .‬דלקת של הריאה עם נוזל פלוירלי משמאל‬
‫פנאומוטורקס של הריאה השמאלית‬ ‫ג‪.‬‬
‫ד‪ .‬דקסטרוקרדיה‬

‫‪ .25‬ערכי דיפוזיה נמוכים )‪ (DLCO‬סביר שיהיו בכל המחלות הבאות פרט ל‪:‬‬
‫א‪ .‬מחלות רסטריקטיביות פרנכימטיות‬
‫ב‪ .‬אמפיזמה‬
‫ג‪ .‬מחלות וסקולריות‬
‫ד‪ .‬עישון כבד‬
‫ה‪ .‬אסטמה‬

‫שאלות ‪ 26-27‬מתייחסות לקטע הבא‪:‬‬


‫בת ‪ 33‬בריאה בד"כ‪ .‬הופנתה לחדר מיון בשל קוצר נשימה חריף וכאבים בבית חזה שמאלי‬
‫שמחמירים בנשימה עמוקה‪ .‬שוללת שיעול‪.‬‬
‫יום טרם פנייתה חזרה מטיסה ארוכה שנמשכה כ‪ 8-‬שעות‪.‬‬
‫פרט לגלולות לא נוטלת תרופות באופן קבוע‪ ,‬לא מעשנת‪.‬‬
‫בבדיקה‪ 22 :‬נשימות לדקה‪ ,‬דופק ‪ 98‬סדיר‪ ,‬ל‪.‬ד‪ ,120/60 .‬חום ‪37.5‬‬
‫בדיקת ריאות‪ ,‬בטן‪ ,‬לב ‪ -‬ב‪.‬מ‪.‬פ‪.‬‬
‫מעבדה‪12300 WBC :‬‬
‫‪70 - PO2 ,28 - PCO2 ,7.49 - PH :ABG‬‬

‫‪ .26‬מהי האבחנה הקלינית המסבירה את התלונות‪:‬‬


‫א‪ .‬דלקת ריאות‬
‫ב‪ .‬חזה אוויר‬
‫ג‪ .‬תסחיף ריאתי‬
‫ד‪ .‬פריקרדיטיס‬
‫ה‪ .‬היפרונטילציה בעקבות סטרס‬

‫‪ .27‬מהי הבדיקה הכי טובה לאבחנה‪:‬‬


‫א‪ .‬אקו לב‬
‫ב‪ .‬צילום חזה‬
‫ג‪ US .‬רגליים‬
‫ד‪ CT .‬אנגיו‬

‫‪ .28‬סלייד ‪ - 3‬נראה סימפון גדול שמזכיר ברונכיאקטזיה‪ ,‬אולם היתה בבירור הטרוגניות‬
‫טמפורלית ומאפייני ‪UIP‬‬

‫‪ .29‬הכל אופייני למחלה הנ"ל פרט ל‪:‬‬


‫‪ .1‬דלקת כרונית‬
‫‪ .2‬פיברוזיס אינטרסטיציאלית‬
‫‪ .3‬ריבוי סיבי שריר חלק בתוך רקמת חיבור‬
‫‪ .4‬התרחבות הסימפונות‬

‫‪ .30‬סלייד ‪ 4‬אבחנה – ‪lobar pneumonia‬‬

‫‪- .31‬‬
‫‪- .32‬‬
‫‪ .33‬כל הבאים מתאימים ל‪ Chronic bronchitis-‬פרט ל‪:‬‬
‫א‪ .‬ריבוי תאי ‪Goblet‬‬
‫ב‪ .‬ריבוי תאים מפרישי ריר בסבמוקוזה‬
‫ג‪ .‬ריבוי תאי שריר חלק‬
‫‪ .34‬כל הבאים יכולים להיות סיבה ל‪ Bronchiectasis-‬פרט ל‪:‬‬
‫א‪Cystic fibrosis .‬‬
‫ב‪α1 antitrypsin deficiency .‬‬
‫ג‪Primary cilliary dyskinesia .‬‬
‫ד‪Immunodeficiency .‬‬
‫‪ .35‬אקסודט קיצפי בחללי האוויר אופייני ל‪-‬‬
‫א‪CMV .‬‬
‫ב‪Pneumocystic carinni pneumonia .‬‬
‫ג‪Aspargillus .‬‬
‫ד‪Herpes virus .‬‬
‫‪ .36‬מה אופייני ל‪?Atypical pneumonia-‬‬
‫א‪ .‬תפליט דלקתי חריף באלבאולי‬
‫ב‪ .‬אטיפיה של אפיתל נשימתי‬
‫ג‪ .‬דלקת אינטרסטיציאלית‬
‫ד‪ .‬גרנולומות‬
‫‪ .37‬מה נכון לגבי שחפת?‬
‫א‪ .‬מעורבות בלוטות לימפה בשער הריאה אופייני ל‪.secondary TB‬‬
‫ב‪ miliary TB .‬מתאפיין בהופעת חללים בריאות‬
‫ג‪ .‬בדיקת ‪ Mantoux‬חיובית מעידה על זיהום פעיל‬
‫ד‪ .‬הופעת ‪ secondary TB‬ברוב המקרים היא תוצאה של ריאקטיביציה של ‪primary TB‬‬
‫‪ .38‬מה נכון לגבי ‪? Bronchioalveolar carcinoma‬‬
‫א‪ .‬מתחיל בסמפון גדול‬
‫ב‪ .‬לא חודר לסטרומה‬
‫ג‪ .‬מבנה סולידי‬
‫ד‪ .‬נמק נרחב‬
‫ה‪ .‬הופעה קלינית שכיחה כאטלקטזיס או אמפיזמה‬
‫‪ .39‬מה נכון לגבי ‪ solitary fibrous tumor‬של הפלאורה ?‬
‫א‪ .‬קשור לאזבסט‬
‫ב‪ .‬צורה בלוטית‬
‫ג‪ .‬תאי מזוטל אטיפיים‬
‫ד‪ .‬ברוב המקרים נראה כמו ‪Fibrosarcoma‬‬
‫ה‪ .‬אף תשובה אינה נכונה‬
‫‪ .40‬מהי האבחנה בסלייד מס' ‪? 2‬‬
‫א‪Squamous cell carcinoma .‬‬
‫ב‪Adenocarcinoma .‬‬
‫ג‪Bronchioalveolar carcinoma .‬‬
‫ד‪Carcinoid tumor .‬‬
‫בשאלות ‪ 41-44‬התאם בין ההיגדים הבאים לבין האבחנות‪:‬‬
‫א‪ .‬ברונכיטיס כרוני‬
‫ב‪ .‬אסטמה‬
‫ג‪ .‬אמפיזמה‬
‫‪ .41‬עליה בתחלואה קשורה בתיאורית היגיינת יתר‬
‫‪ .42‬מבחני אלרגיה חיוביים בחלק ניכר מהחולים‬
‫‪ .43‬האתיולוגיה העיקרית איננה עישון‬
‫‪ .44‬חסימת דרכי אוויר תתכן כתוצאה מאבדן ‪elastic recoil‬‬

‫חלק ג' ‪ -‬נוירולוגיה‬


‫‪ .45‬בחולה עם פגיעה בעצב האוקולומוטורי )עצב קרניאלי ‪ (3‬הימני עקב סכרת‪:‬‬
‫א‪ .‬האישון מורחב ואיננו מגיב לאור‬
‫ב‪ .‬האישון צר ואיננו מגיב לאור‬
‫ג‪ .‬האישון תקין ומגיב רק להארה ישירה עליו‪ ,‬אך לא קונסנסואלית )הארה על העין‬
‫השנייה(‬
‫ד‪ .‬האישון מורחב ומגיב להארה קונסנסואלית‪ ,‬אך לא ישירה‬
‫ה‪ .‬אף תשובה לא נכונה‬

‫‪ .46‬במקרה של פולינוירופתיה דה‪-‬מיאלינטיבית )‪:(demyelinating polyneuropathy‬‬


‫א‪ .‬אפשר להרגיע את החולה שתחול צמיחה מחדש של העצבים שנפגעו בקצב של ‪ 3-4‬ס"מ‬
‫לחודש עד להתחדשות התפקוד‬
‫ב‪ .‬משרעת הפוטנציאלים המוטוריים של ההולכה נמוכות מהנורמה‬
‫ג‪ .‬אפשר להרגיע את החולה שתחול צמיחה מחדש של העצבים שנפגעו בקצב של ‪ 1-2‬ס"מ‬
‫לחודש‪ ,‬עד להתחדשות התפקוד‪.‬‬
‫ד‪ .‬יש לצפות לחסמי הולכה בבדיקת ה‪EMG-‬‬
‫ה‪ .‬ההולכה העצבית תקינה בסיבים העבים‪ ,‬אך איטית בסיבים הקטנים‬
‫‪ .47‬לחולה עם כאבים עקב פולינוירופתיה‪:‬‬
‫א‪ .‬הטיפול היעיל ביותר הוא משכך כאב נוגד דלקת‬
‫ב‪ .‬התרופה אלטרול )אמיטריפטילין( היא תרופת הבחירה לחולים עם כאב התקפי חד‬
‫ג‪ .‬התרופה טגרטול )קרבמזפין( היא תרופת הבחירה לחולים עם כאב התקפי חד‬
‫ד‪ .‬הכאבים על פי רוב אינם סימטריים‬
‫ה‪ .‬הכאב נעלם מאזורי גוף בהם נעלמת התחושה העורית‬

‫‪ .48‬לגבי רעד‪ ,‬הכל נכון פרט ל‪:‬‬


‫א‪ .‬הרעד יכול לערב גפה אחת או מספר גפיים‪ ,‬את הראש‪ ,‬את השפתיים או את כל האיברים‬
‫הללו יחד‬
‫ב‪ .‬רעד של הידיים יכול להיות במנוחה או בפעולה‬
‫ג‪ .‬הרעד בדרך כלל לא סדיר )אי‪-‬רגולרי(‪ ,‬לעומת תנועות בלתי רצוניות אחרות שהן סדירות‬
‫)רגולריות(‬
‫ד‪ .‬רעד של הראש שכיח ב‪essential tremor-‬‬
‫ה‪ essential tremor .‬היא ברובה משפחתית‬

‫‪ .49‬הזרקות של רעלן הבוטוקס משמשות כטיפול בכל המצבים הבאים פרט ל‪:‬‬
‫א‪ .‬דיסטוניות פוקאליות‬
‫ב‪ .‬מיאסטניה גראביס‬
‫ג‪ .‬הזעת יתר‬
‫ד‪ .‬רעד וטיקים‬
‫ה‪ .‬ספסטיות‬

‫‪ .50‬איזו מבין ההצהרות הבאות נכונות לגבי דיסטוניה צווארית )‪?(cervical dystonia‬‬
‫א‪ .‬זוהי דיסטוניה פוקלית שאינה מגיבה היטב לטיפול בבוטוקס‪.‬‬
‫ב‪ .‬זוהי דיסטוניה של הפנים שגורמת לסגירה מאולצת של הפה‪.‬‬
‫ג‪ .‬זוהי דיסטוניה סגמנטלית שכוללת ראש‪ ,‬צוואר ובית חזה‬
‫ד‪ .‬נעלמת בשינה‬
‫ה‪ .‬הסיבה השכיחה ביותר לדיסטוניה זו היא טראומה‬
‫‪ .51‬כל התסמינים הבאים מהווים חלק מתסמיני מחלת פרקינסון‪ ,‬פרט ל‪:‬‬
‫א‪ .‬חוסר קואורדינציה‬
‫ב‪ .‬רעד במנוחה‬
‫ג‪ .‬איטיות‬
‫ד‪ .‬נוקשות שרירים‬
‫ה‪ .‬פני מסיכה‬

‫‪ .52‬בעת מתן ‪ Levodopa‬יתכנו תופעות לוואי מוטוריות‪ .‬באילו מן המצבים הבאים תופעות לוואי‬
‫מוטוריות אלו שכיחות יותר?‬
‫א‪ .‬בחולים מבוגרים ללא קשר למשך המחלה‬
‫ב‪ .‬בחולים עם משך מחלה ארוך יותר‬
‫ג‪ .‬תופעות לוואי מוטוריות אינן קשורות למינון התרופה‬
‫ד‪ .‬בחולים שקיבלו בנוסף קומטן )‪(comtan‬‬
‫ה‪ .‬בנשים‬

‫‪ .53‬אישה בת ‪ 23‬פונה לחדר מיון עקב התקף כאב ראש עז ביותר מזה ‪ 3-4‬שעות‪ ,‬מלווה בחילות‬
‫ורגישות לאור ורעש‪ .‬זהו ההתקף השלישי מאותו סוג בחצי השנה האחרונה‪ .‬בדיקתה‬
‫הנוירולוגית תקינה‪ .‬סימפטומים אלה מכוונים בסבירות הגבוהה ביותר לאבחנה הבאה‪:‬‬
‫א‪ .‬סינוסיטיס חריפה‬
‫ב‪ .‬דימום תת עכבישי‬
‫ג‪ .‬נויראלגיה טריגנימלית‬
‫ד‪ .‬התקף מיגרנה‬
‫ה‪ .‬דימום תוך מוחי‬

‫‪ .54‬איזו מבין ההצהרות הבאות נכונה לגבי כאב נוירופתי?‬


‫א‪ .‬עשוי להיגרם מפגיעה לכל אורך מערכת העצבים הסומטוסנסורית‬
‫ב‪ .‬לעיתים קרובות בעל אופי "שורף" או כמו "זרם"‬
‫ג‪ .‬בבדיקה ניתן למצוא חוסר תחושה באזור הכואב‬
‫ד‪ .‬נחשב לכאב פתולוגי‬
‫ה‪ .‬כל ההצהרות נכונות‬

‫‪ .55‬איזו מבין התרופות הבאות משמשת למניעת התקפי מיגרנה?‬


‫א‪ .‬חוסמי ביתא‬
‫ב‪ .‬טריפטנים‬
‫ג‪ .‬אגוניסטים דופאמינרגים‬
‫ד‪ .‬טגרטול‬
‫ה‪ .‬חוסמי האנזים ‪ACE‬‬

‫‪ .56‬מהי הדרך הטובה ביותר לאמוד עצמת כאב ?‬


‫א‪ .‬התבוננות בחולה והתנהגותו‬
‫ב‪ .‬עפ"י אומדן עם סרגל כאב‬
‫ג‪ .‬התבוננות בסימנים אנטומיים ‪ -‬לחץ דם‪ ,‬דופק‪ ,‬קצב נשימה‬
‫ד‪ .‬אין טעם לאמוד כאב משום שהוא ממילא סובייקטיבי‬
‫ה‪ .‬על פי תלונותיו של החולה‬

‫‪ .57‬גבר בן ‪ 54‬התמוטט בזמן ישיבה סוערת‪ .‬לאחר מספר דקות התאושש והחל לדבר ללא הפסק‪.‬‬
‫לא ניתן היה להפסיק אותו מדיבורו‪ .‬הוא לא הגיב לפניות חוזרות ולא ענה לשאלות‪ .‬למרות‬
‫שניתן היה להבין את רוב מילותיו‪ ,‬ניתן היה להבחין במילים בלתי מובנות‪ .‬בבדיקה‪ :‬עצבי‬
‫מוח‪ ,‬טונוס‪ ,‬כוח‪ ,‬החזרים גידיים‪ ,‬מבחני מוח קטן והליכה תקינים‪ .‬הרופא התרשם כי הנ"ל‬
‫סבל מאירוע מוחי המסתמן ב‪:‬‬
‫‪Broca aphasia‬‬ ‫א‪.‬‬
‫‪Conduction aphasia‬‬ ‫ב‪.‬‬
‫‪Wernike aphasia‬‬ ‫ג‪.‬‬
‫‪Trans-cortical aphasia‬‬ ‫ד‪.‬‬
‫‪Global aphasia‬‬ ‫ה‪.‬‬
‫‪ .58‬מה נכון לגבי אלצהיימר ?‬
‫א‪ .‬זאת הדמנציה השנייה בשכיחותה‬
‫ב‪ 30% .‬מבני גיל ‪ 70‬סובלים ממנה‬
‫ג‪ .‬חולי אלצהיימר עם ‪ HIV‬חיובי מגיבים טוב לתרופות מסוג חסמי אצטיל כולין אסטראז‬
‫ד‪ .‬טיפול בתרופות מסוג חסמי אצטיל כולין אסטראז‬
‫ה‪ .‬ההסתמנות הבולטת הראשונית היא תסמונת אקסראפירמידאלית‬

‫‪ .59‬חולה הובא לחדר מיון לאחר שנמצא מוטל על המדרכה מחוסר הכרה‪ .‬כל הסימנים הבאים‬
‫יעלו חשד שמדובר בהתקף אפילפטי פרט ל‪:‬‬
‫א‪ .‬סימנים לנשיכת לשון‬
‫ב‪ .‬אישונים לא שווים‬
‫ג‪ .‬דיסלוקציה של כתף ימין‬
‫ד‪ .‬מכנסיים רטובים משתן‬
‫ה‪ .‬החולה התעורר במהלך נסיעה באמבולנס‪ ,‬אולם הוא עדיין מבולבל בעת הגעתו לחדר‬
‫מיון‬

‫‪ .60‬חולה הגיע לחדר המיון בשל התקף של אובדן הכרה ופרכוס בארבע גפיים‪ .‬כשנבדקה במהלך‬
‫הברור בוצע ‪ CT‬ראש אשר הדגים גידול מסוג מנינגיומה באזור פרונטאלי שמאלי‪ .‬מהו סוג‬
‫ההתקף האפילפטי ?‬
‫א‪ .‬טוני‪-‬קלוני כללי‬
‫ב‪ .‬אבסנס‬
‫ג‪ .‬מיוקלונוס‬
‫ד‪ .‬מוקדי מורכב‬
‫ה‪ .‬מוקדי פשוט‬
‫ו‪ .‬אף תשובה אינה נכונה‬

‫‪ .61‬נערה בת ‪ 23‬מגיעה למיון לאחר התקף פירכוסי שכלל איבוד הכרה‪ .‬לטענתה סבלה מ‪3-‬‬
‫התקפים דומים בשנה האחרונה‪ ,‬כולל רגעים של ‪ 10-20‬שניות של ניתוק‪ .‬מהו הטיפול המיטבי‬
‫בחולה זו ?‬
‫א‪ .‬קרבמזפין‬
‫ב‪ .‬חומצה ולפרואית‬
‫ג‪ .‬בנזודיאזפינים‬
‫ד‪ .‬לא יינתן טיפול‬

‫‪– .62‬‬

‫‪- .63‬‬

‫‪ .64‬גבר בן ‪ 56‬נותח בשל גידול גדול באונה פרונטלית‪ .‬בבדיקה פתולוגית נראתה אטיפיה‪,‬‬
‫מיטוזות‪ ,‬נמק ופרוליפרציה של כלי דם בגידול‪ .‬צביעה ל‪ GFAP‬חיובית בתאי הגידול‬
‫מה האבחנה‪:‬‬
‫א‪astrocytoma grade 2 .‬‬
‫ב‪glioblastoma multiforme .‬‬
‫ג‪medulloblastoma .‬‬
‫ד‪malignant meningioma .‬‬
‫ה‪ .‬גרורה של קרצינומה‬
‫‪ .65‬מה נכון לגבי דמנציה עם גופיפי לואי )‪?(lewi bodies‬‬
‫א‪ .‬זאת סיבה נדירה מאוד של דמנציה‬
‫ב‪ lewi bodies .‬נמצאים באסטרוציטים )תאי גליה(‬
‫ג‪ .‬ה‪ Substantia Nigra-‬נשארת לא מעורבת ברוב המקרים‬
‫ד‪ .‬ב‪ 10%‬מהמקרים אפשר למצוא שינויים של מחלת אלצהיימר‬
‫ה‪ Lewi bodies .‬נצבעים חיובי ל‪ubiquitin-‬‬

‫‪ .66‬כל באים נכונים לגבי ‪ Multiple sclerosis‬פרט ל‪:‬‬


‫א‪ .‬הלימפוציטים שמתחילים את תהליך הדהמיאליניזציה הם ‪CD4 T lymphocytes‬‬
‫ב‪ .‬רב ה‪ plaques-‬נמצאים בקליפת המח‬
‫ג‪ .‬ב‪ active plaques-‬רואים הסננה ניכרת ע"י לימפוציטים ומאקרופאגים‬
‫ד‪ shadow plaques .‬מראים כמות מופחתת של מיאלין עקב ‪Incomplete‬‬
‫‪demyelinization‬או ‪.Remyelination‬‬
‫ה‪ plaque .‬עם ‪ Reactive Gliosis‬לפעמים דומה ל‪low grade astrocytosis-‬‬

‫‪ .67‬כל הבאים נכונים לגבי שטף דם מוחי‪ ,‬פרט ל‪:‬‬


‫א‪ – Berry aneurism .‬הסיבה הכי שכיחה ל‪Subarachnoid hemorrhage-‬‬
‫ב‪ .‬המקום השכיח ביותר של ‪ Hypertensive hemorrhage‬הוא ב‪Putamen-‬‬
‫ג‪ Cerebral vasculitis .‬הינה הסיבה השכיחה ביותר לשטף דם מוחי‬
‫ד‪ .‬ניתוח דם פתוח יכול להיות גורם סיכון‬
‫ה‪ .‬רב החולים הם בגיל ‪ 60‬ומעלה‬

‫‪ .68‬סמן מה נכון לגבי ‪:PML‬‬


‫א‪ .‬נגרם ע"י ‪HIV‬‬
‫ב‪ .‬הפגיעה העיקרית היא ב‪Oligodendroglia-‬‬
‫ג‪ .‬סימן אופייני באזורי הפגיעה – צביעה חזקה ב‪Luxol fast blue-‬‬
‫ד‪ Bizarre giant neurons .‬נראים מסביב לאזור הפגיעה‬
‫ה‪ .‬רב הנגעים נמצאים באזור קליפת המח‬

‫‪ .69‬כל הבאים נכונים לגבי ‪ Altzheimer's disease‬פרט ל‪:‬‬


‫א‪ .‬משקל המוח יכול להיות תקין‬
‫ב‪ .‬נראית אטרופיה של האונה הטמפורלית והפרונטלית‬
‫ג‪ .‬השינויים פזורים באזורים שונים במוח באופן אחיד‬
‫ד‪ .‬קליפת המוח נראית דקה מהרגיל‬
‫ה‪ .‬במח המזדקן יכולים להיות שינויים מיקרוסקופיים דומים למחלת אלצהיימר‬

‫‪ .70‬לגבי סלייד מס' ‪ ,5‬כל הבאים נכונים פרט ל‪) :‬מנינגיומה(‬


‫א‪ .‬ריבוי מיטוזות בגידול זה מעיד על התנהגות יותר אגרסיבית‬
‫ב‪ .‬נוטה להישנות אפילו כאשר שפיר‬
‫ג‪ .‬חדירה למוח מגבירה את הסיכוי להישנות‬
‫ד‪ .‬יותר שכיח אצל נשים‬
‫ה‪ .‬השינוי הכרומוזומלי השכיח ביותר קשור לכרומוזום ‪21‬‬

‫‪ .71‬כל הבאים הם סימנים של מחלת אלצהיימר‪ ,‬פרט ל‪:‬‬


‫א‪Amyloid (senile) plaque .‬‬
‫ב‪Neurofibrillay tangles .‬‬
‫ג‪Granulovacular degeneration .‬‬
‫ד‪Intranuclear inclusions .‬‬
‫ה‪Amyloid angiopathy .‬‬
‫‪ .72‬סמן מה נכון לגבי מחלת האלצהיימר?‬
‫א‪ Aβ-peptide .‬הינו תוצר של פעילות הפוספטאז‬
‫ב‪ Tau .‬הינו חלבון פתולוגי‪ ,‬שלא נמצא בתא תקין‬
‫ג‪ .‬למולקולה של ‪ APP‬יש חלק תוך תאי‪ ,‬חוץ תאי וטראנסממברנלי‬
‫ד‪ Amyloid Plaques .‬לא מכילים אקסונים‬
‫ה‪ .‬מספר ה‪ Amyloid Plaques -‬במוח מראה קורלוציה הכי טובה עם דרגת הדמנציה‬

‫‪ .73‬אישה בת ‪ ,56‬סובלת מיתר ל"ד‪ ,‬הגיעה לחדר המיון במצב של תרדמת שהתפתח תוך זמן קצר‬
‫מאד בעבודה‪ .‬ב‪ CT-‬נראה שטף דם סובארכנואידלי נרחב בבסיס המוח‪ .‬מה הסיבה הסבירה‬
‫ביותר למצב זה?‬
‫א‪Acute bacterial meningitis .‬‬
‫ב‪Tuberculous meningitis .‬‬
‫ג‪amyloid angopathy .‬‬
‫ד‪ruptured aneurism .‬‬
‫ה‪Vasculitis .‬‬

‫‪ .74‬סמן מה נכון לגבי ‪? CMV encephalitis‬‬


‫א‪ .‬שכיח אצל ילדים בני ‪5-10‬‬
‫ב‪ .‬גורם לנמק המוראגי מסביב לחדרים של המוח אצל חולים מדוכאי חיסון‬
‫ג‪ .‬במוח נראו גרנולומות עם תאי ענק‪.‬‬
‫ד‪ .‬לא מערב את חוט השדרה והשורשים‬
‫ה‪ .‬האבחנה אפשרית בד"כ רק באמצעות ‪PCR‬‬

‫‪ .75‬הסיבה העיקרית ל‪ Global cerebral ischemia -‬הינה‪:‬‬


‫א‪Thrombosis of middle cerebral artery .‬‬
‫ב‪Rupture of aneurysm .‬‬
‫ג‪Brain contusion .‬‬
‫ד‪Severe hypotensive episode .‬‬
‫ה‪Meningeal fibrosis .‬‬

‫חלק ד' ‪ -‬קרדיולוגיה‬


‫‪ .76‬ציין את ההצהרה הנכונה לגבי אספירין‪:‬‬
‫א‪ .‬הוא בעל מבנה כימי של פטיד‬
‫ב‪ .‬הוא בעל מבנה סוכרי‬
‫ג‪ .‬הוא חומצה סליצילית‬
‫ד‪ .‬הוא אצטיל חומצה סליצילית‬
‫ה‪ .‬הוא נקשר באופן הפיך לאנזים ‪ cyclooxygenase‬ובכך מונע יצירה של פרוסטגלנדינים‬

‫‪ - Reye’s syndrome .77‬מה נכון?‬


‫א‪ .‬נגרם בילדים והרקע שלו דמם כלייתי ויתר קרישיות פריפרית‪.‬‬
‫ב‪ .‬סינדרום אוטואימוני שמשופעל על ידי אספירין הגורם ליתר קרישיות‪.‬‬
‫ג‪ .‬קטלני בעד ‪ 30%‬מהלוקים בו ומאופיין על ידי דגנרציה שומנית של הכבד‪.‬‬
‫ד‪ .‬מהווה בעיה בילדים ובמבוגרים שבגללה ירד השימוש באספירין‪.‬‬
‫ה‪ .‬נגרם על ידי תרופות אנטי אנפלמטוריות שונות בנוסף לאספירין‪.‬‬
‫‪ .78‬המום האופייני לתסמונת ‪ Di George‬הינו‪:‬‬
‫א‪Supravalvular aortic stenosis .‬‬
‫ב‪Patent ductus arteriosus .‬‬
‫ג‪Tricuspid atresia .‬‬
‫ד‪Atrioventricular septal defect .‬‬
‫ה‪Truncus arteriosus .‬‬

‫‪ .79‬המיקום של האאורטה ב‪ Transposition of great arteries -‬הינו‪:‬‬


‫א‪ .‬מימין ומקדימה לעורק הריאתי‬
‫ב‪ .‬משמאל ומקדימה לעורק הריאתי‬
‫ג‪ .‬משמאל ומאחורה לעורק הריאתי‬
‫ד‪ .‬מימין ומאחורה לעורק הריאתי‬
‫ה‪ .‬אין שינוי לעומת לב תקין‬

‫‪.80‬כל הבאים שייכים לקבוצה של ‪ acyanotic congenital heart defects‬פרט ל‪:‬‬


‫א‪Atrial septal defect .‬‬
‫ב‪Tricuspid atresia .‬‬
‫ג‪ventricular septal defect .‬‬
‫ד‪Patent ductus arteriosus .‬‬

‫‪ .81‬מה הבסיס המורפולוגי להתפתחות של ‪? Stable angina‬‬


‫א‪ .‬בליטה אתרוסקלרוטית התופסת ‪ 20%‬מחלל העורק הכלילי‪.‬‬
‫ב‪ .‬בליטה אתרוסקלרוטית התופסת ‪ 70%‬מחלל העורק הכלילי‪.‬‬
‫ג‪ .‬בליטה אתרוסקלרוטית עם קרע ותרומבוס דופני‪.‬‬
‫ד‪ .‬דימום בתוך בליטה אתרוסקלרוטית‪.‬‬
‫ה‪ .‬הסתיידות בשכבה בינונית )‪ (Media‬בלי הצרות החלל‪.‬‬

‫‪ .82‬מהי סיבה יאטרוגנית להתפתחות ‪? restrictive cardiomiopathy‬‬


‫א‪Doxorubicin .‬‬
‫ב‪ .‬הקרנות‬
‫ג‪Cyclophosphamid .‬‬
‫ד‪ .‬ניתוח עם פתיחת השק הפריקרדיאלי‬
‫ה‪Beta blockers .‬‬
‫‪ .83‬מי מהבאים היא מחלת מסתם עם פגיעה על‪-‬ידי מנגנון אימוני?‬
‫א‪Mitral valve prolaps .‬‬
‫ב‪Nonbacterial thrombotic endocarditis .‬‬
‫ג‪Infective endocarditis .‬‬
‫ד‪Libman-Sack’s endocarditis .‬‬
‫ה‪Carcinoid syndrome .‬‬
‫‪ .84‬כל התוצאות הבאות קשורות לקרדיומיופתיה היפרטרופית פרט ל‪:‬‬
‫א‪ .‬מוות פתאומית‪.‬‬
‫ב‪Subaortic obstruction .‬‬
‫ג‪ .‬אוטם לב‬
‫ד‪Infective endocarditis .‬‬
‫ה‪ .‬אנאוריזמה בדופן הלב‬
‫‪ .85‬כל הבאים מופיעים ב ‪ infective endocarditis‬פרט ל‪:‬‬
‫א‪ .‬אי‪-‬ספיקה לבבית‬
‫ב‪Streptococus Group A .‬‬
‫ג‪ .‬וסקוליטיס לא ספטי ‪leukocytoclastic vasculitis‬‬
‫ד‪ .‬אנוריזמה לא ספטי )‪ (barry‬בעורק מוחי‬
‫ה‪ .‬אמבוליזם‬

‫‪ .86‬מה נכון לגבי ל‪? Libman-Sacks-‬‬


‫א‪ .‬תהליך פתוגני הקשור ל‪Chlamydia infection-‬‬
‫ב‪ .‬וגיטציות בשני צידי המסתם‬
‫ג‪ .‬תגובה אימונית השייכת ל‪Hypersensitivity reaction type IV-‬‬
‫ד‪ .‬מחלה הקשורה לפגיעה במטבוליזם של פוספוליפידים‬
‫ה‪ .‬זהו סוג של ‪infectious endocarditis‬‬

‫‪ .87‬כל הבאים שייכים לבליטה האתרומטית פרט ל‪:‬‬


‫א‪ .‬קרטין‬
‫ב‪ .‬פיברין‬
‫ג‪ .‬קולגן‬
‫ד‪ .‬סיבי אלסטיים‬
‫ה‪ .‬פרוטאוגליגאנים‬

‫‪ .88‬מהו התהליך ב‪:Rheumatic Carditis -‬‬


‫א‪ .‬נמק קרישתי מאסיבי הכולל את האנדוקרדיום‬
‫ב‪ .‬מוקדי ניוון עם נמק פיברינואידי באנדוקרד‬
‫ג‪ .‬הסננה של תאים פולימורפונוקלאריים בשריר הלב‬
‫ד‪ .‬תגובה אימונית של ‪hypersensitivity type III‬‬
‫ה‪ .‬ניוון של המסתם ללא תגובה דלקתית‬

‫‪ .89‬מה נכון לגבי טרשת עורקים?‬


‫א‪ .‬המחלה פוגעת בעורקים בינוניים וקטנים‬
‫ב‪ .‬המחלה פוגעת בעיקר בשכבה האמצעית של העורק‪ ,‬ובאופן משני בשכבה הפנימית‬
‫ג‪ .‬לימפוציטים הם התאים העיקריים בבליטה‬
‫ד‪ .‬תאי שריר משתתפים בצבירת ליפידים ע"י ‪foam cells‬‬
‫ה‪ .‬מונוציטים שמגיעים לאינטימה מהדם הם המקור לתאי שריר חלק בכיפה הפברוטית של‬
‫האתרומה‬

‫‪ .90‬איזה אזור של שריר הלב יותר רגיש לאיסכמיה ?‬


‫א‪ .‬האפקס‬
‫ב‪ .‬הספטום‬
‫ג‪ .‬השליש הפנימי של דופן הלב‬
‫ד‪ .‬השליש החיצוני של דופן הלב‬
‫ה‪ .‬שכבה דקיקה של שריר בעובי ‪ 0.1‬מ"מ מתחת לאנדוקרד‬

‫‪ .91‬מה נכון לגבי שינויים פתולוגיים הקשורים למחלת לב ראומטית‬


‫א‪ .‬עיוות של שסתום עקב פיברוזיס והצטלקות‬
‫ב‪ .‬ניוון מוקוידי )‪ (myxoid degeneration‬ברקמת שסתום‬
‫ג‪ .‬הרס של שסתום על ידי חיידקים מסוג ‪group A streptococcus‬‬
‫ד‪ .‬הסתיידויות בעיקר בסיס של שסתום ולא בקצה חופשי‬
‫ה‪ .‬התנקבות השסתום‬
‫‪ .92‬מה צפוי לראות בבדיקה ההיסטולוגית של שריר הלב שבועיים אחרי התחלת אוטם ?‬
‫א‪ .‬סיבי שריר עם ‪wavy pattern‬‬
‫ב‪ .‬רקמת גרעון‬
‫ג‪ .‬נמק טרי של סיבי שריר עם הסננה לוקיוציטרית נכרת‬
‫ד‪ .‬הצטלקות של אזור האוטם‬
‫ה‪ .‬רמקה נמקית מתפרקת‬

‫‪ .93‬לבליטות שנמצאות בקצה חופשי של שסתום מיטרלי אצל חולים עם מחלת לב ראומטית‬
‫קוראים‪:‬‬
‫א‪atheromas .‬‬
‫ב‪Maccallum plaquews .‬‬
‫ג‪Fibromas .‬‬
‫ד‪Papilomas .‬‬
‫ה‪Verrucae .‬‬

‫‪ .94‬כל הגורמים הבאים קשורים להתפתחות קרדיומיופתיה פרט ל‪:‬‬


‫א‪ .‬הריון‬
‫ב‪ .‬אלכוהול‬
‫ג‪ .‬איסכמיה‬
‫ד‪ .‬חוסר ויטמין ‪B1‬‬
‫ה‪ .‬שקיעת עמילואיד‬

‫‪ .95‬איזה סוג של נמק מופיע במחלת הלב שמתוארת בסלייד מס' ‪:(transmural MI) 1‬‬
‫א‪fibrinoid necrosis .‬‬
‫ב‪Coagulation necrosis .‬‬
‫ג‪Liquefaction necrosis .‬‬
‫ד‪Gangrenous necrosis .‬‬
‫ה‪Apoptosis .‬‬
‫‪ .96‬איזה מהמשפטים הבאים נכון?‬
‫א‪ .‬כדי לאבחן תעוקת חזה עלינו לבצע צינתור‪.‬‬
‫ב‪ .‬אבחנה של תעוקת חזה נעשית על סמך סיפור החולה )אנמנזה( בלבד‪.‬‬
‫ג‪ .‬חולה עם תעוקת חזה במנוחה לא זקוק לאשפוז‪.‬‬
‫ד‪ .‬אבחנה של תעוקת חזה לא יציבה מחייבת אישור באמצעות ‪.ECG‬‬
‫ה‪ .‬תעוקת חזה תמיד מופיעה במרכז החזה‪.‬‬

‫‪ .97‬אוטם שריר הלב עלול להתבטא ב‪:‬‬


‫א‪ .‬אי ספיקת לב פתאומית‪.‬‬
‫ב‪ .‬כאב ממושך )מעל ‪ 40‬דקות( במרכז החזה‪.‬‬
‫ג‪ .‬איבוד הכרה פתאומי‪.‬‬
‫ד‪ .‬כל התשובות נכונות‪.‬‬

‫‪ .98‬איזה משפט נכון?‬


‫א‪ .‬כדי להימנע מטעויות הרופא מבסס את הנחת העבודה על ממצאים באנמנזה ובדיקה‬
‫גופנית‬
‫ב‪ .‬כדי להימנע מטעויות מאשש הרופא את הנחת העבודה בבדיקות מעבדה‬
‫ג‪ .‬הרופא קובע פרוגנוזה לחולה לפני התחלת טיפול‬
‫ד‪ .‬בדיקות מעבדה אינן תחליף לאנמנזה ובדיקה גופנית קפדנית‬
‫ה‪ .‬כל התשובות נכונות‬
‫‪ .99‬תסמונת תעוקת חזה לא יציבה היא‪:‬‬
‫א‪ .‬תעוקת חזה שהחלה לפני ‪ 4‬חודשים‬
‫ב‪ .‬תעוקת חזה במאמץ בינוני‬
‫ג‪ .‬תעוקת חזה במאמץ קשה‬
‫ד‪ .‬תעוקת חזה במנוחה‬
‫ה‪ .‬כל התשובות נכונות‬

‫‪ .100‬איזה מהכללים הבאים נכון ?‬


‫א‪ .‬חולה עם תעוקת חזה במנוחה‪ ,‬ניתן לשחרר אם ‪ ECG‬תקין‬
‫ב‪ .‬חולה בת ‪ ,60‬היפרליפידמית‪ ,‬סובלת מכאב עז המתגבר בנשימה עמוקה כנראה סובלת‬
‫מאוטם שריר הלב‬
‫ג‪ .‬דיסקציה של אבי העורקים‪ ,‬פריקרדיטיס‪ ,‬אוטם ריאתי וחזה אוויר יכולים לגרום לכאב‬
‫עז בחזה‬
‫ד‪ .‬אבחנת תעוקת חזה נעשית באנמנזה ובדיקות מאמץ‬
‫ה‪ .‬כל התשובות נכונות‬

‫‪ .101‬במחלת סכרת הפגיעה בלב מתבטאת ב‪:‬‬


‫א‪ .‬ירידה בקצב הלב עקב עליה בגירוי הואגוס‬
‫ב‪ .‬עליה בתפוקת הלב עקב רמות גבוהות של סוכר בדם‬
‫ג‪ .‬שיפור בהתכווצות שריר הלב‬
‫ד‪ .‬ירידה בתפקוד חדר שמאל‬
‫ה‪ .‬עליה בל"ד סיסטולי‬

‫‪ .102‬הטיפול בתרופות ממשפחת הסטטינים‪:‬‬


‫א‪ .‬מוריד כמות רצפטורים ל‪ LDL -‬על פני הפטוציטים‪.‬‬
‫ב‪ .‬מקטין ספיגת שומנים מהדיאטה בפעולה במעי‪.‬‬
‫ג‪ .‬מגדיל הפרשת כולסטרול במלחי המרה ומונע ספיגתם‪.‬‬
‫ד‪ .‬מקטין ייצור ‪ LDL‬על ידי הפטוציטים‪.‬‬
‫ה‪ .‬מקטין שכיחות אוטמים רק בחולים שעברו אוטם בעבר‪.‬‬
‫‪ .103‬תלונות שכיחות בהצרות מסתם מיטרלי‬
‫א‪ .‬קוצר נשימה‬
‫ב‪ .‬דופק לא סדיר‬
‫ג‪ .‬תסחיפים‬
‫ד‪ .‬כל התשובות נכונות‪.‬‬
‫‪ .104‬בבדיקתו של חולה הסובל מקרדימיופתיה היפרטרופית ‪:‬‬
‫א‪ .‬גל הדופק איטי וממושך‬
‫ב‪ .‬לחץ הדם נמוך באופן טיפוסי‬
‫ג‪ .‬יש אי ספיקה של מסתם אבי העורקים‬
‫ד‪ .‬גל דופק בעוצמה רבה‬

‫‪ .105‬בבדיקתו של חולה עם אי ספיקת מסתם אבי העורקים )‪:(AI‬‬


‫א‪ .‬הדופק הוא איטי וממושך‪.‬‬
‫ב‪ .‬הדופק מכה )מהיר וגבוה(‬
‫ג‪ .‬לחץ הדם הדיאסטולי גבוה בצורה טיפוסית‬
‫ד‪ .‬האוושה הטיפוסית רמה לכל אורך הסיסטולה‬
‫תשובות אפשריות‪:‬‬
‫ב‬ ‫‪81‬‬ ‫‪61‬‬ ‫ב‬ ‫‪41‬‬ ‫ה‬ ‫‪21‬‬ ‫א‬ ‫‪1‬‬
‫ב‬ ‫‪82‬‬ ‫‪62‬‬ ‫ב‬ ‫‪42‬‬ ‫ה‬ ‫‪22‬‬ ‫ב‬ ‫‪2‬‬
‫ד‬ ‫‪83‬‬ ‫‪63‬‬ ‫ב‬ ‫‪43‬‬ ‫ה‬ ‫‪23‬‬ ‫ב‬ ‫‪3‬‬
‫ה‬ ‫‪84‬‬ ‫ב‬ ‫‪64‬‬ ‫ג‬ ‫‪44‬‬ ‫ב‬ ‫‪24‬‬ ‫ג‬ ‫‪4‬‬
‫‪85‬‬ ‫ה‬ ‫‪65‬‬ ‫ה‬ ‫‪45‬‬ ‫ה‬ ‫‪25‬‬ ‫ג‬ ‫‪5‬‬
‫ב‬ ‫‪86‬‬ ‫ב‬ ‫‪66‬‬ ‫ד‬ ‫‪46‬‬ ‫ג‬ ‫‪26‬‬ ‫ב‬ ‫‪6‬‬
‫א‬ ‫‪87‬‬ ‫ג‬ ‫‪67‬‬ ‫ג‬ ‫‪47‬‬ ‫ד‬ ‫‪27‬‬ ‫ד‬ ‫‪7‬‬
‫ב‬ ‫‪88‬‬ ‫ב‬ ‫‪68‬‬ ‫ג‬ ‫‪48‬‬ ‫‪UIP‬‬ ‫‪28‬‬ ‫ב‬ ‫‪8‬‬
‫ד‬ ‫‪89‬‬ ‫ג‬ ‫‪69‬‬ ‫ב‬ ‫‪49‬‬ ‫ד‬ ‫‪29‬‬ ‫ג‬ ‫‪9‬‬
‫ג‬ ‫‪90‬‬ ‫ה‬ ‫‪70‬‬ ‫בוטל‬ ‫‪50‬‬ ‫‪Lobar‬‬ ‫‪30‬‬ ‫‪10‬‬
‫‪pneumonia‬‬
‫א‬ ‫‪91‬‬ ‫ד‬ ‫‪71‬‬ ‫א‬ ‫‪51‬‬ ‫‪31‬‬ ‫‪11‬‬
‫ב‬ ‫‪92‬‬ ‫ג‬ ‫‪72‬‬ ‫ב‬ ‫‪52‬‬ ‫‪32‬‬ ‫‪12‬‬
‫ה‬ ‫‪93‬‬ ‫‪73‬‬ ‫ד‬ ‫‪53‬‬ ‫‪33‬‬ ‫‪13‬‬
‫התקבלו‬ ‫‪94‬‬ ‫‪74‬‬ ‫ה‬ ‫‪54‬‬ ‫ב‬ ‫‪34‬‬ ‫‪14‬‬
‫כל‬
‫התשובות‬
‫ב‬ ‫‪95‬‬ ‫ד‬ ‫‪75‬‬ ‫א‬ ‫‪55‬‬ ‫ב‬ ‫‪35‬‬ ‫ב‬ ‫‪15‬‬
‫ב‬ ‫‪96‬‬ ‫ד‬ ‫‪76‬‬ ‫ב‬ ‫‪56‬‬ ‫ג‬ ‫‪36‬‬ ‫ג‬ ‫‪16‬‬
‫ד‬ ‫‪97‬‬ ‫ג‬ ‫‪77‬‬ ‫ג‬ ‫‪57‬‬ ‫ד‬ ‫‪37‬‬ ‫ב‬ ‫‪17‬‬
‫ה‬ ‫‪98‬‬ ‫ה‬ ‫‪78‬‬ ‫בוטל‬ ‫‪58‬‬ ‫ב‬ ‫‪38‬‬ ‫ב‬ ‫‪18‬‬
‫ד‬ ‫‪99‬‬ ‫א‪+‬ד‬ ‫‪79‬‬ ‫ב‬ ‫‪59‬‬ ‫ד‪+‬ה‬ ‫‪39‬‬ ‫א‬ ‫‪19‬‬
‫ג‬ ‫‪100‬‬ ‫ב‬ ‫‪80‬‬ ‫ו‬ ‫‪60‬‬ ‫ד‬ ‫‪40‬‬ ‫‪-‬‬ ‫‪20‬‬
‫ד‪+‬ה‬ ‫‪101‬‬
‫ד‬ ‫‪102‬‬
‫ד‬ ‫‪103‬‬
‫ד‬ ‫‪104‬‬
‫ב‬ ‫‪105‬‬
‫מקבץ ‪ 1‬ומבואות ‪ -‬מועד א – ‪2006-2007‬‬

‫פרק מס ‪ :1‬מבואות‬

‫‪.1‬‬
‫כל המשפטים הבאים נכונים לגבי הדמיית ‪ FDG PET‬בחולי סרטן פרט ל‪:‬‬
‫א‪ .‬הדמיה זו מבוססת על דטקציה בו‪-‬זמנית של שתי קרני גאמה הנפלטות ב‪ 180-‬מעלות זו מזו‬
‫לאחר תהליך אניהילציה של פוזיטרון הנפלט מאיזוטופ רדיואקטיבי עם אלקטרון‪.‬‬
‫ב‪ .‬אחד היתרונות בבדיקה הוא שניתן להשתמש ב‪ FDG -‬לא רק להדמייה אלא גם לטיפול‬
‫בגרורות‪.‬‬
‫ג‪ .‬אחד היתרונות בבדיקה הוא האפשרות לסקירת הגוף כולו לאחר הזרקה חד פעמית של‬
‫החומר הרדיואקטיבי‬
‫ד‪ .‬בדיקת ‪ FDG PET‬יכולה לזהות רקמה סרטנית פעילה גם בהעדר ממצאים פתולוגים ב‪.CT -‬‬

‫‪.2‬‬
‫כל המשפטים הבאים לגבי מנגנוני הלוקליזציה של החומרים בהם משתמשים ברפואה גרעינית נכונים‬
‫פרט ל‪:‬‬
‫א‪ .‬מיפוי עצם מבוסס על ספיחה כימית על גבי גבישי הדרוקסיאפטיטי של העצם‬
‫ב‪ .‬מיפוי ריאות מבוסס על דיפוזיה פשוטה של החומר לריאות‪.‬‬
‫ג‪ .‬מפוי בלוטת התריס מבוסס על קליטה אקטיבית נגד מפל ריכוזים‬
‫ד‪ .‬מיפוי ‪ FDG PET‬מבוסס על מטבוליזם תוך תאי‪.‬‬

‫‪.3‬‬
‫כל המשפטים הבאים לגבי מיפוי כליות נכונים פרט ל‪:‬‬
‫א‪ .‬ניתן לאבחן באמצעות המיפוי דחייה של כלייה מושתלת הן מקרוב משפחה והן מהמת‪.‬‬
‫ב‪ .‬על מנת ללמוד במיפוי על חסימה בניקוז הכלייתי ניתן להשתמש בתרופה כמו קפוטן‪.‬‬
‫ג‪ .‬מיפוי כליות הינו מיפוי דינמי‪.‬‬
‫ד‪ .‬במיפוי כליות ‪ DMSA‬ניתן לאבחן דלקות כליתיות‪.‬‬

‫‪.4‬‬
‫כל המשפטים הבאים לגבי רפואה גרעינית נכונים פרט ל‪:‬‬
‫א‪ .‬הדמיה ברפואה גרעינית מבוססת על הזרקת חומר רדיואקטיבי ובדיקת תהליכים פיזיולוגים‬
‫בגוף‪.‬‬
‫ב‪ .‬כדי שאטום רדיואקטיבי יהיה מתאים לשמוש בהדמייה ברפואה גרעינית עליו להיות בעל זמן‬
‫מחצית חיים ארוך מאד )מספר ימים לפחות(‪.‬‬
‫ג‪ .‬היתרון בהדמייה משולבת )‪ (PET/CT, SPECT/CT‬הוא ביכולת למקם אנטומית את ממצאי‬
‫המיפוי‪.‬‬
‫ד‪ .‬בדיקת ‪ SPECT‬מציגה את ממצאי המיפוי בשלושה מישורים ויעילה למיקום הממצאים‬
‫באיזורים של אנטומיה מורכבת‪.‬‬

‫‪.5‬‬
‫איזה מהמשפטים הבאים נכון לגבי מיפוי עצם לצורך אבחון שברי מאמץ‪:‬‬
‫א‪ .‬מיפוי עצם יכול להראות את שבר המאמץ גם כאשר צילום רנטגן של האזור תקין‪.‬‬
‫ב‪ .‬מיפוי עצם אינו יעיל לאבחנת שברי מאמץ בנשים‪.‬‬
‫ג‪ .‬מיפוי בכדוריות לבנות מסומנות משמש לעיתים כמיפוי משלים למיפוי העצם לצורך אבחון‬
‫שברי מאמץ‪.‬‬
‫ד‪ .‬הסגוליות )ספציפיות( של מיפוי העצם לאבחון שברי מאמץ הנה גבוהה אולם רגישות הבדיקה‬
‫יחסית נמוכה‪.‬‬

‫עמוד ‪ 1‬מתוך ‪21‬‬


‫‪.6‬‬
‫כל הבאים נכונים לגבי מפוי ריאות פרט ל‪:‬‬
‫א‪ .‬החלקיקים המוזרקים "נתקעים" בפרה‪-‬קפילרות הראתיות ביחס של ‪1:1000‬‬
‫ב‪ .‬מיפוי ריאות יעיל בהדמיית תסחיפים ריאתיים וגידולי ריאה‪.‬‬
‫ג‪ .‬החומר המוזרק במיפוי הינו מאקרו‪-‬אגרגטים של אלבומין‬
‫ד‪ .‬במהלך הבדיקה מוזרק חומר רדיואקטיבי לזרם הדם וניתן גז רדיואקטיבי בשאיפה‪.‬‬

‫‪.7‬‬
‫תהליך האבחון של מחלת היפר‪-‬פאראתירואיד יכול להתחיל כתוצאה מקבלת ממצא אקראי של‬
‫עלייה מעל לתחום הייחוס של‪:‬‬
‫‪ + TSH‬הורמוני יתרת הכליה‬ ‫א‪.‬‬
‫סידן‬ ‫ב‪.‬‬
‫‪EDTA‬‬ ‫ג‪.‬‬
‫פלואוריד‬ ‫ד‪.‬‬
‫אין למעבדה הקלינית תפקיד כל שהוא באבחון מחלות‪.‬‬ ‫ה‪.‬‬

‫‪.8‬‬
‫עמידה של מבחנת דם זמן ממושך לפני העברה למעבדה תגרום לממצא שגוי בצמד הבדיקות הבאות‪:‬‬
‫א‪ .‬נתרן ואשלגן‬
‫ב‪ .‬סידן ופוספט‬
‫ג‪ .‬אשלגן וסידן‬
‫ד‪ .‬גלוקוז ונתרן‬
‫ה‪ .‬אשלגן וגלוקוז‬

‫‪.9‬‬
‫חולה הגיע לחדר מיון עם חום גבוה ותהליך דלקתי סוער והועבר ליחידה לטיפול נמרץ‪ .‬ביחידה נלקחו‬
‫דגימות דם לבדיקות שונות‪ ,‬ובין השאר נמצאה רמה גבוהה ביותר של אשלגן בסרום‪ .‬איזו התיחסות‬
‫יש לתת לממצא זה?‬
‫א‪ .‬יש לו חשיבות רבה בהמשך הטיפול בחולה‬
‫ב‪ .‬יש להתייחס בזהירות רבה לממצא אשר במצב החולה יכול להיות ‪false positive‬‬
‫ג‪ .‬תהליך דלקתי סוער מגביר את רמת כל האלקטרוליטים ועל כן זהו ממצא סביר‬
‫ד‪ .‬יש להתייחס לממצא זה ביחד עם ערך הנתרן בדם בכדי לקבוע טיפול נאות‬

‫‪.10‬‬
‫רמת ייחודיות )‪ (specificity‬גבוהה נדרשת מבדיקה המיועדת לאבחן‪:‬‬
‫א‪ .‬מחלה בעלת שכיחות גבוהה באוכלוסיה‬
‫ב‪ .‬מחלה קשה‪ ,‬אך יש לה טיפול יעיל‬
‫ג‪ .‬מחלה קשה ללא טיפול יעיל‬
‫ד‪ .‬מחלה בעלת מאפיינים שקשה לאבחן בדיקה פיזיקלית‬

‫‪.11‬‬
‫חוסר התקדמות גלי ‪ R‬בחיבורים ‪ .V1-4‬מה הנכון ביותר‪:‬‬
‫א‪ .‬שכיח מאד במרימי משקולות‬
‫ב‪ .‬זהו סימן למחלה ריאתית משמעותית‬
‫ג‪ .‬מרמז עם אוטם קדמי‪-‬מחיצתי קודם‬
‫ד‪ .‬מרמז על בעיה טכנית במיקום אלקטרודות הא‪.‬ק‪.‬ג‬

‫עמוד ‪ 2‬מתוך ‪21‬‬


‫‪.12‬‬
‫השאלה הבאה מתייחסת לתמונה מספר ‪:1‬‬
‫בתרשים שלפניך מהו הציר החשמלי‪:‬‬
‫א‪ .‬מינוס ‪ 45‬מעלות‬
‫ב‪ 0 .‬מעלות‬
‫ג‪ 30 .‬מעלות‬
‫ד‪ 90 .‬מעלות‬

‫‪.13‬‬
‫השאלה הבאה מתייחסת לתמונה מספר ‪2‬‬
‫מה התשובה הנכונה ביותר לגבי התרשים הבא‪:‬‬
‫א‪ .‬לא ניתן לקבוע מיקצב‪ ,‬ציר ‪+ 75‬‬
‫ב‪ .‬מיקצב סינוס‪ ,‬ציר ‪+ 110‬‬
‫ג‪ .‬מיקצב סינוס‪ ,‬מקטע ‪ PR‬ארוך מהתקין‪ ,‬ציר ‪+ 90‬‬
‫ד‪ .‬מיקצב סינוס‪ ,‬מקטע ‪ PR‬תקין‪ ,‬ציר ‪+ 75‬‬

‫‪.14‬‬
‫השאלה הבאה מתייחסת לתמונה מספר ‪3‬‬
‫מה התשובה הנכונה ביותר לגבי התרשים הבא‪:‬‬
‫א‪ .‬סינוס תקין‪ ,‬מקטע ‪ PR‬תקין‪ ,‬ציר תקין‬
‫ב‪ .‬סינוס ברדיקרדיה‪ ,‬מקטע ‪ PR‬תקין‪ ,‬ציר תקין‬
‫ג‪ .‬סינוס ברדיקרדיה‪ ,‬מקטע ‪ PR‬תקין‪ ,‬ציר שמאלי‬
‫ד‪ .‬סינוס ברדיקרדיה‪ ,‬מקטע ‪ PR‬תקין‪ ,‬ציר ימני‬

‫‪.15‬‬
‫השאלה הבאה מתייחסת לתמונה מספר ‪4‬‬
‫לאיזה מהבאים שייך תרשים הא‪.‬ק‪.‬ג הבא‪:‬‬
‫א‪ .‬סטודנט לרפואה שנה ו' בזמן בחינת גמר‪ ,‬הסובל מהתקף חרדה ושחרור מוגבר של אדרנלין‬
‫ב‪ .‬גבר בן ‪ 75‬הסובל שנים רבות מיתר לחץ דם‬
‫ג‪ .‬אישה בת ‪ 23‬הסובלת מיתר לחץ דם ראתי ראשוני‬
‫ד‪ .‬רופאה בת ‪ ,50‬ללא היסטוריה של מחלת לב‬

‫‪.16‬‬
‫השאלה הבאה מתייחסת לתמונה מספר ‪5‬‬
‫לאיזה מהבאים שייך תרשים הא‪.‬ק‪.‬ג הבא‪:‬‬
‫א‪ .‬סטודנט לרפואה שנה ו' בזמן בחינת גמר‪ ,‬הסובל מהתקף חרדה ושחרור מוגבר של אדרנלין‬
‫ב‪ .‬גבר בן ‪ 75‬הסובל שנים רבות מיתר לחץ דם‬
‫ג‪ .‬אישה בת ‪ 23‬הסובלת מיתר לחץ דם ראתי ראשוני‬
‫ד‪ .‬רופאה בת ‪ ,50‬ללא היסטוריה של מחלת לב‬

‫‪.17‬‬
‫תסמונת הנוירון המוטורי העליון כוללת‪:‬‬
‫א‪ .‬ערות החזרים שטחיים‬
‫ב‪ .‬ירידה בהחזרים הגידיים‬
‫ג‪ .‬תגובה פלנטרית פלקסורית‬
‫ד‪ .‬טונוס ספסטי‬
‫ה‪ .‬טונוס פלסידי‬

‫עמוד ‪ 3‬מתוך ‪21‬‬


‫פרק מספר ‪ :2‬ריאות‬

‫‪.18‬‬
‫כל הבאים מקובלים כטיפול בתסמונת דום נשימה בשינה פרט ל‪:‬‬
‫א‪ .‬ירידה במשקל‬
‫ב‪ .‬מכשיר ‪CPAP‬‬
‫ג‪ .‬התקן דנטלי‬
‫ד‪ .‬נתוח חיך ולוע‬
‫ה‪ .‬מתן סטרואידים‬

‫‪.19‬‬
‫מה מבאים יהיה האמצעי היעיל ביותר למניעת העברה של דלקות ריאה הנרכשות בבתי חולים‪:‬‬
‫א‪ .‬מתן אנטיביוטיקה רחבת טווח כפעולה מניעתית‬
‫ב‪ .‬רחיצת ידיים‬
‫ג‪ .‬מתן טיפול קבוע בחוסמי ‪H2‬‬
‫ד‪ .‬השכבת החולההמונשם על הבטן‬
‫ה‪ .‬השכבת החולה במצב ‪supine‬‬

‫‪.20‬‬
‫שיעול הנגרם ע"י ‪ post nasal drip‬מאופיין ע"י כל הבאים פרט ל‪:‬‬
‫א‪ .‬אנמנזה של זיהום דרכי נשימה עליונות לפני תחילת השיעול‬
‫ב‪ .‬שיעול המכיל טיפות דם‬
‫ג‪ .‬ממצא בבדיקה גופנית של גודש בריריות הפה והלוע‬
‫ד‪ .‬שיעול המחמיר לאחר שכיבה‬
‫ה‪ .‬שיעול המוקל לאחר מתן רנטי היסטמינים‬

‫‪.21‬‬
‫מפל חמצן בין אלביאולה לקפילרה ריאתית )‪ (A-a gradient‬סביר שיימצא תקין בחולה הבא‪:‬‬
‫א‪ .‬חולה צעיר ללא מחלה ראתית ידועה אשר התקבל למיון מחוסר הכרה לאחר הזרקת מינון‬
‫יתר של סמים‪.‬‬
‫ב‪ .‬חולה צעיר ללא מחלת ריאות ידועה אשר התקבל למיון עקב חום גבוה‪ ,‬קוצר נשימה ובצילום‬
‫חזה נראה תסנין היכול להתאים לדלקת ריאות‪.‬‬
‫ג‪ .‬חולה מעשן כבר הידוע כסובל מ‪ COPD -‬הגיע למיון עם החמרה במצבו הנשימתי‪.‬‬
‫ד‪ .‬חולה צעיר‪ ,‬ידוע כסובל מאסטמה אשר הגיע למיון עם התקף אסטמה חריף‪.‬‬

‫‪.22‬‬
‫חולה בן ‪ 72‬שוחרר מאשפוז לאחר שבר בירך‪ .‬יומיים לאחר שחרורו הוא מתעורר מהשינה עם כאב‬
‫פלוירלי וקשיי נשימה‪ .‬מה אינו נכון לגבי אבחנתו‪:‬‬
‫א‪ .‬האירוע יכול להיות מסכן חיים‬
‫ב‪ .‬שכיבה יכולה להיות גורם אטיולוגי למצב זה‬
‫ג‪ .‬האירוע מלווה לעיתים בגניחת דם‬
‫ד‪ .‬בכל המקרים של האבחנה הנ"ל קיימת היפוקסמיה‬
‫ה‪ .‬ערך ‪ di-dimer‬שלילי בנוכחות חשד קליני גבוה אינו שולל אבחנה זו‬

‫‪.23‬‬
‫כל הנ"ל הן סיבותשכיחות להמופטיזיס פרט ל‪:‬‬
‫א‪pulmonary embolism .‬‬
‫ב‪chronic bronchitis .‬‬

‫עמוד ‪ 4‬מתוך ‪21‬‬


‫ג‪tuberculosis .‬‬
‫ד‪ .‬גידול ריאתי‬
‫ה‪ .‬אסטמה‬

‫‪.24‬‬
‫הטיפול החשוב ביותר ב‪ COPD -‬הינו‪:‬‬
‫א‪ .‬משאפי סטרואידים‬
‫ב‪ .‬תכשירים אנטיכולינרגים‬
‫ג‪ .‬שיקום נשימתי‬
‫ד‪ .‬מרחיבי סימפונות‬
‫ה‪ .‬הפסקת עישון‬

‫‪.25‬‬
‫נקראת לבדוק חולה עם היפוקסמיה קשה‪ .‬ביקשת לתת חמצן לחולה תוך ניטור סטורציה‪ .‬אילו ערכי‬
‫סטורציה תבקש לשמור‪:‬‬
‫א‪ .‬סטורציה ‪100%‬‬
‫ב‪ .‬סטורציה ‪80%-90%‬‬
‫ג‪ .‬סטורציה ‪90%-92%‬‬
‫ד‪ .‬אף תשובה לא נכונה‬

‫‪.26‬‬
‫נוזל פלוירלי טרנסודטיבי מתאפיין ב‪:‬‬

‫נוזל דמי עם חלבון גבוה‬ ‫א‪.‬‬


‫נוזל צלול עם ‪ LDH‬גבוה‬ ‫ב‪.‬‬
‫נוזל צלול עם חלבון נמוך‬ ‫ג‪.‬‬
‫נוזל עם ‪ pH‬של ‪ 7.12‬ואלבומין גבוה‬ ‫ד‪.‬‬
‫שכיח בפנוימוניות בקטריאליות‬ ‫ה‪.‬‬

‫‪.27‬‬
‫ניתן לזהות בצילום חזה את כל המבנים הנ"ל פרט ל‪:‬‬
‫א‪ .‬קשת האאורטה‬
‫ב‪ .‬קנה‬
‫ג‪ .‬צלעות‬
‫ד‪ .‬מסתם לא מסוייד‬
‫ה‪ .‬פטמת שד‬

‫‪.28‬‬
‫חולה מעשן התקבל למיון‪ .‬בצילום חזה נמצאה הצללה שלמה של הריאה הימנית עם סטייה בולטת‬
‫של המדיאסטינום לצד ימין‪ .‬האבחנה הסבירה היא‪:‬‬
‫א‪ .‬תמט של הריאה הימנית‬
‫ב‪ .‬נוזל פלוירלי‬
‫ג‪ .‬דלקת ריאה מימין‬
‫ד‪ .‬שינויים מתאימים ל‪COPD -‬‬
‫ה‪ .‬דקסטרוקרדיה‬

‫עמוד ‪ 5‬מתוך ‪21‬‬


‫‪.29‬‬
‫באיזה סוג של סרטן ריאה הטיפול מבוסס על כימותרפיה והקרנה‪:‬‬
‫א‪ .‬סרטן קשקשי‬
‫ב‪ .‬סרטן מסוג תאים קטנים‬
‫ג‪ .‬סרטן בלוטי‬
‫ד‪ .‬סרטן מסוד תאים גדולים‬
‫ה‪ .‬כל התשובות נכונות‬

‫‪.30‬‬
‫חולה בן ‪ 54‬מעשן שבעברו עסק בחיתוך אסבסט מגיע עם מחלת ריאות אינטרסטיציאלית‪.‬‬
‫סביר שתמצא את כל הממצאים הבאים פרט ל‪:‬‬
‫א‪ .‬היפוקסמיה‬
‫ב‪ .‬היפוקפניה‬
‫ג‪ A-a gradient .‬נמוך במאמץ‬
‫ד‪ .‬טכיפניאה‬
‫ה‪ .‬דיפוזיה נמוכה‬

‫‪.31‬‬
‫טיפול במרחיבי סימפונות מסוד בטא אגוניסטים כרוך בכל הבאים פרט ל‪:‬‬
‫א‪ .‬הורדת רמת האשלגן בדם‬
‫ב‪ .‬הפחתה משמעותית של התגובה הדלקתית בדרכי האויר‬
‫ג‪ .‬טכיקרדיה‬
‫ד‪ .‬רלקסציה של שריר חלק‬
‫ה‪ .‬שיפעול ‪adenylase cyclase‬‬
‫‪.32‬‬
‫איזו מהקביעות הבאות נכונה לגבי ‪ emphysema‬של הריאות‪:‬‬
‫א‪ .‬ההפרעה התפקודית היא בעיקר רסטרקטיבית )‪(restrictive‬‬
‫ב‪ .‬פיברוזיס ריאתית משמעותית היא אחד המרכיבים של התמונה הפתולוגית‬
‫ג‪ .‬חוסר שיווי משקל בין פרוטאזה ואנטי‪-‬פרוטאזה נחשב גורם אטיולוגי מרכזי‬
‫ד‪ .‬במרבית מקרי האמפיזמה הנגעים ממוקמים בעיקר בבסיסי הריאות‬
‫ה‪ .‬יש עליה ב‪ elastic recoil -‬של הריאה‬
‫‪.33‬‬
‫חולה בן ‪ 18‬שנה הידוע כסובל מאסטמה מגיע למיון בשל התקף אסמטי קשה‪.‬‬
‫כל ההגדים הבאים נכונים לגבי חולה זה פרט ל‪:‬‬
‫א‪ .‬ההתקף יכול להגרם ע"י חשיפה לאלרגנים‬
‫ב‪ .‬ההתקף יכוללהגרם ע"י מתן חוסמי בטא‬
‫ג‪ .‬מתן אנטילויקוטריאנים הינה אופציה יעילה לטיפול בחולה זה‬
‫ד‪ .‬יש לתת סטרואידים סיסטמיים לחולה זה‬
‫ה‪ .‬מרחיבי סימפונות וחמצן הינם הטיפולים בעלי ההשפעה המהירה ביותר‬
‫‪.34‬‬
‫כל הבאים מאפיינים ל )‪ , Idiopathic Pulmonary Fibrosis (IPF‬פרט ל‪:‬‬
‫א‪ .‬נוכחות פיברוזיס בגילאים שונים )‪(Temporal hetrogeneity‬‬
‫ב‪ .‬היווצרות גרנולומות‬
‫ג‪Fibroblastic foci .‬‬
‫ד‪ .‬היפרפלזיה של שריר חלק באינטרסטיציום‬
‫ה‪ .‬דלקת כרונית‬

‫עמוד ‪ 6‬מתוך ‪21‬‬


‫‪.35‬‬
‫איזו מהתכונות הבאות אינה מאפיינת סרקואידוזיס?‬
‫א‪ .‬פיזור גרנולומות לאורך הלימפתיים‬
‫ב‪ .‬גרנולומות פריברונכיאליות‬
‫ג‪ .‬תאי ענק מסוג לנגהנס‬
‫ד‪ .‬נמק נרחב בתוך גרנולומות‬
‫ה‪ .‬מערבת רקמות שונות )איברים שונים(‬

‫‪.36‬‬
‫מה נכון לגבי ‪?Cryptogenic organizing pneumonia‬‬
‫א‪ .‬הפרוגנוזה דומה ל‪IPF-‬‬
‫ב‪ .‬אתיולוגיה זיהומית‬
‫ג‪ .‬אתיולוגיה לא ברורה‬
‫ד‪ .‬בדרך כלל מלווה בפיברוזיס קשה‬
‫ה‪ .‬מלווה בממברנות היאליניות‬

‫‪.37‬‬
‫מה נכון לגבי ‪?Hypersensitivity pneumonitis‬‬
‫א‪ .‬דלקת אינטרסטציאלית‬
‫ב‪ .‬פיברוזיס באינטרסטיציום בשלבים המאוחרים‬
‫ג‪ .‬נוכחות גרנולומות‬
‫ד‪ .‬תהליך מופיע בעיקר מסביב לברונכיולות‬
‫ה‪ .‬כולם נכונים‬

‫‪.38‬‬
‫תכשיר מספר ‪ 2) 1‬שאלות(‬
‫חום‪ ,‬שיעול‪ ,‬תסנינים מפוזרים בגבר בן ‪ .28‬המחלה שבתכשיר מספר ‪) 1‬מה הנכון(‬
‫א‪ .‬גידולית וקשורה לעישון‬
‫ב‪ .‬גידולית וקשורה לחשיפה לאסבסט‬
‫ג‪ .‬זיהומית וצביעת ‪ Ziehl-Nielsen‬יכולה לזהות את הגורם‬
‫ד‪ .‬זיהומית וצביעת כסף יכולה לזהות את הגורם‬
‫ה‪ .‬זיהומית וצביעת גרם או ‪ PAS‬יכולה לזהות את הגורם‬
‫‪.39‬‬
‫המחלה שבתכשיר מספר ‪ 1‬מופיעה בדרך כלל‪:‬‬
‫א‪ .‬כסיבוך של אספירציה‬
‫ב‪ .‬בקרום הריאה‬
‫ג‪ .‬כגוש באיזור ההילוס בצד אחד‬
‫ד‪ .‬בחולה עם דיכוי מערכת החיסון‬
‫‪.40‬‬
‫תכשיר מספר ‪ 2) 2‬שאלות(‬
‫חולה בת ‪ 38‬הגיעה עם שיעול בצילום חזה נרה תסנין באונת ריאה‪ .‬לדבריה בשלוש שנים אחרונות‬
‫סבלה מדלקות ריאה חוזרות באונה ימנית‪ .‬בתכשיר שלפניך נראה‪:‬‬
‫א‪ .‬גידול שפיר‬
‫ב‪ .‬גידול ממאיר )מדרגת ממאירות נמוכה(‬
‫ג‪ .‬גידול ממאיר )מדרגת ממאירות גבוהה מאוד(‬
‫ד‪ .‬תהליך גרנולומטוטי‬
‫ה‪ .‬ברונכואקטזיס‬

‫עמוד ‪ 7‬מתוך ‪21‬‬


‫‪.46‬‬
‫כל הגורמים הבאים ידועים כגורמי פנוימוניה בעיקר אצל מדוכאי חיסון‪ .‬איזה מהבאים אינו שייך‬
‫לקבוצה זו?‬
‫א‪Mucor mycosis .‬‬
‫ב‪Pneumocystic carinii .‬‬
‫ג‪Mycoplasma .‬‬
‫ד‪Aspergillus .‬‬
‫בשאלות ‪ 47-52‬התאם בין ההיגדים הבאים לבין האבחנות )א‪-‬ג(‪ .‬כל היגד יכול להופיע יותר מפעם‬
‫אחת או לא להופיע כלל‪.‬‬

‫א‪ .‬ברונכיטיס כרונית‬


‫ב‪ .‬אסטמה‬
‫ג‪ .‬אמפיזמה‬

‫‪.47‬‬
‫קיימת הסננה אאוזינופילית‬

‫‪.48‬‬
‫חסר ‪ α1 anti trypsin‬יכול להוות אטיולוגיה‬

‫‪.49‬‬
‫‪Blue blotter‬‬

‫‪.50‬‬
‫תגובה מרשימה למשאפי סטרואידים‬

‫‪.51‬‬
‫‪Pink puffer‬‬
‫‪.52‬‬
‫אין התאלות הציפורניים )‪(clubbing‬‬
‫פרק מספר ‪ :3‬קרדיולוגיה‬
‫‪.53‬‬
‫מה מופיע ב‪?Rheumatic carditis-‬‬
‫א‪ .‬בלונינג )‪ (balooning‬של שסתום מיטרלי‬
‫ב‪ .‬נוגדנים )‪ (cross reactiong‬ל‪staphylococeus aurus-‬‬
‫ג‪ .‬הסננה בעיקר ע"י פולימורפונוקלארים בשריר הלב‬
‫ד‪ .‬התהליך יכול לפגוע בכל שכבות הלב‬
‫ה‪ .‬שינויים ניוונים בלי תגובה דלקתית‬
‫‪.54‬‬
‫מה נכון לגבי התא בתמונה המצורפת? )תמונה מספר ‪) (1‬תאים אלה מופיעים באחת ממחלות הלב(‬
‫א‪ .‬זה תא ענק רב גרעיני‬
‫ב‪ .‬התאים מסוג זה מצטברים מסביב נמק גבינתי‬
‫ג‪ .‬זה תא אפופטוטי‬
‫ד‪ .‬בתוך גרעין התא נמצאה רצועת כרומטין מסולסלת‬
‫ה‪ .‬בתוך גרעין התא נמצא גרעינון בולט‬

‫עמוד ‪ 8‬מתוך ‪21‬‬


.55
?Infective endocarditis ‫ אצל חולים עם‬Glomerulonephritis ‫איזה מנגנון מעורב בהתפתחות של‬
Antigen-antibody complex .‫א‬
Antibody mediated reaction .‫ב‬
Cytotoxic T8 lymphocytic reaction .‫ג‬
T4 lymphocytes mediated reaction .‫ד‬
‫ התהליך לא קשור למנגנון אימוני‬.‫ה‬

.56
?Nonbacterial thrombotic endocarditis ‫מה מופיע בפתוגנזה של‬
Antiphospholipid syndrome .‫א‬
(Fibrinoid necrosis) ‫ נמק פיברינואידי‬.‫ב‬
Hypercoagulability .‫ג‬
‫ התנקבות בשסתום‬.‫ד‬
Immune complex .‫ה‬
.57
:‫( מתאימה ל‬2 ‫התמונה המיקרוסקופית של וגטציות במסתם )תמונה מספר‬
Rheumatic verrucae .‫א‬
Infective endocarditis .‫ב‬
Nonbacterial thromboendocarditis .‫ג‬
Libman-Sack’s endocarditis .‫ד‬
Mitral valve prolaps .‫ה‬

.58
?‫ מה נכון‬.‫מרכיבים של בליטה אתרוסקלרוטית‬
‫ גרנולומות‬.‫א‬
‫ תאי ענק‬.‫ב‬
‫ תאי שריר משורטט‬.‫ג‬
‫ תאי שריר חלק‬.‫ד‬
Amyloid ‫ משקעי‬.‫ה‬

.59
?‫מה לא שייך לבליטה אתרוסקלרוטית‬
Fibrous cap .‫א‬
Neovascularization .‫ב‬
Medionecrosis .‫ג‬
Foam cells .‫ד‬
Central necrotic care .‫ה‬

.60
?‫איזה מנגנון גורם להצטברות שומנים בציטופלזמה של תאי קצף שנמצאים בליטה אתרומטוטית‬
‫ פגוציטוזיס‬.‫א‬
‫ אנדוציטוזיס רגיל‬.‫ב‬
‫ ניוון שומני‬.‫ג‬
‫ אגירה כתוצאה מחוסר אנזים ספציפי‬.‫ד‬
(Scavenger receptors ‫ אמדוציטוזיס באמצעות רצפטורים )דרך‬.‫ה‬

21 ‫ מתוך‬9 ‫עמוד‬
‫‪.61‬‬
‫איזה הסננה מופיעה באיזור הנמק של אוטם הלב ביום השלישי מהתחלת האירוע?‬
‫א‪ .‬אאוזינופילים‬
‫ב‪ .‬פולימורפונוקלארים‬
‫ג‪ .‬מאקרופגים‬
‫ד‪ .‬לימפוציטים‬
‫ה‪ .‬תאי קצף‬

‫‪.62‬‬
‫איזה סיבוך עלול להיות בשלב של צלקת אחרי ?‪?Transmural MI‬‬
‫א‪ .‬קרע ב‪Papillary muscle-‬‬
‫ב‪ .‬טמפונדה‬
‫ג‪ .‬פריקרדיטיס פיברינוטי‬
‫ד‪ .‬הצטלקות והיצרות של מסתם מיטרלי‬
‫ה‪Ventricular aneurysm .‬‬

‫‪.63‬‬
‫מה לא נכון לגבי הסיבוך של ‪ MI‬שרואים בתמונה )תמונה מספר ‪?(3‬‬
‫א‪ .‬הסיבוך מלווה ביצירת צלקת ו‪Ventricular aneurysm-‬‬
‫ב‪ .‬המצב מופיע בד"כ ‪ 3-7‬ימים מהתחלת ‪Transmural MI‬‬
‫ג‪ .‬הסיבוך קשור ל‪Transmural MI-‬‬
‫ד‪ .‬הסיבוך לפעמים מתפתח במחיצה בין החדרים של הלב וגורם ל‪ shunt-‬משמאל לימין‬

‫‪.64‬‬
‫מהו המידגם ההיסטולוגי הספציפי שמופיע ב‪?Hypertrophic cardiomyopathy-‬‬
‫א‪ Wavy pattern .‬של מיוציטים‬
‫ב‪Contraction band .‬‬
‫ג‪Mycotic disarray .‬‬
‫ד‪Myocytolysis .‬‬
‫ה‪ .‬שינויים ניווניים במיוציטים‬

‫‪.65‬‬
‫בתמונה )תמונה מספר ‪ 4‬סלייד היסטולוגי של שריר הלב עם צביעה חיובית לברזל‪ .‬איזה תופעה‬
‫מתאימה למצב הזה?‬
‫א‪ .‬היפרטרופיה בלתי סימטרית של דופן הלב‬
‫ב‪ .‬התרחבות נכרת של חלל הלב‬
‫ג‪ Hypercontraction .‬של שריר הלב‬
‫ד‪Subaortal stenosis .‬‬
‫ה‪ .‬ירידה בגמישות שריר הלב‬

‫‪.66‬‬
‫המום השכיח ביותר בתסמונת ‪ Williams‬הינו?‬
‫א‪Ventricular septal defect .‬‬
‫ב‪Atrioventricular septal defect .‬‬
‫ג‪Coarctation of aorta .‬‬
‫ד‪Tetralogy of Fallot .‬‬
‫ה‪Supravalvular aortic stenosis .‬‬

‫עמוד ‪ 10‬מתוך ‪21‬‬


‫‪.67‬‬
‫ב‪ Sinus venosus type-‬ה"חור" במחיצה בין העליות נמצא?‬
‫א‪ .‬גבוה במחיצה‬
‫ב‪ .‬באמצע )מרכז( המחיצה‬
‫ג‪ .‬בחלק התחתון של המחיצה‬
‫ד‪ .‬מדובר בחור שתופס את כל המחיצה‬
‫ה‪ .‬אף אחד אינו נכון‬

‫‪.68‬‬
‫איזה מהבאים הינו מרכיב של ‪?Tetralogy of Fallot‬‬
‫א‪Left ventricular hyperplasia .‬‬
‫ב‪Aortic stenosis .‬‬
‫ג‪Right ventricular hypertrophy .‬‬
‫ד‪Atrioventricular septal defect .‬‬
‫ה‪Patent ductus arteriosus .‬‬
‫‪.69‬‬
‫האבחנה בסלייד מספר ‪ 3‬היא‪_______________ :‬‬

‫‪.70‬‬
‫השינויים ב סלייד מספר ‪:3‬‬
‫א‪ .‬הם רברסביליים לאחר טיפול ברוב החולים‬
‫ב‪ .‬הם רברסביליים לאחר טיפול במיעוט החולים‬
‫ג‪ .‬אופייניים לימים ‪ 10‬עד ‪ 14‬של המחלה‬
‫ד‪ .‬האזור הנגוע נראה בהיר‪-‬צהבהב )מאקרוסקופית(‬
‫ה‪ .‬איבוד גליקוגן ונפיחות המיטוכונדריה הם ממצאים אופייניים בתאים שבסלייד‬
‫‪.71‬‬
‫בחרו תשובה אחת נכונה‪:‬‬
‫א‪ .‬כדי לאבחן תעוקת חזה עלינו לאשש האבחנה בבדיקת מאמץ‬
‫ב‪ .‬אבחנה של תעוקת חזה נעשית על סמך האנמנזה בלבד‬
‫ג‪ .‬חולה עם תעוקת חזה במנוחה אינו זקוק לאשפוז‬
‫ד‪ .‬אבחנה של תעוקת חזה לא יציבה מחייבת אשוש באמצעות אקג‬
‫ה‪ .‬תעוקת חזה )‪ (Angina pectoris‬צמיד מופיעה במרכז החזה‬
‫‪.72‬‬
‫אוטם שריר הלב עשוי להתבטא ב?‬
‫א‪ .‬אי ספיקת לב פתאומית‬
‫ב‪ .‬כאב ממושך בקדמת החזה )מעל ‪ 40‬דקות(‬
‫ג‪ .‬איבוד הכרה פתאומי‬
‫ד‪ .‬כל התשובות נכונות‬
‫‪.73‬‬
‫איזה מהמשפטים הבאים נכון‪:‬‬
‫א‪ .‬כדי להימנע מטעויות הרופא מבסס את הנחת העבודה על ממצאים באנמנזה ובבדיקה גופנית‬
‫ב‪ .‬כדי להימנע מטעויות מאשש הרופא את הנחת העבודה בבדיקות מעבדה‬
‫ג‪ .‬הרופא קובע את הפרוגנוזה של החולה לפני התחלת הטיפול‬
‫ד‪ .‬בדיקות מעבדה אינן תחליף לאנמנזה קפדנית ובדיקה גופנית קפדנית‬
‫ה‪ .‬כל התשובות נכונות‬

‫עמוד ‪ 11‬מתוך ‪21‬‬


‫‪.74‬‬
‫תסמונת תעוקת חזה לא יציבה היא‪:‬‬
‫א‪ .‬תעוקת חזה במנוחה‬
‫ב‪ .‬תעוקת חזה מתקדמת )‪(crescendo‬‬
‫ג‪ .‬תעוקת חזה חדשה‬
‫ד‪ .‬תעוקת חזה לאחר אוטם‬
‫ה‪ .‬כל התשובות נכונות‬

‫‪.75‬‬
‫איזה מהכללים הבאים נכון‪:‬‬
‫א‪ .‬חולה עם תעוקת חזה לא יציבה יש לאשפז‬
‫ב‪ .‬חולה בן ‪ 60‬שנה‪ ,‬מעשן ועם היפרליפדמיה הסובל מכאב עז וממושך בקדמת החזה )מעל ‪30‬‬
‫דקות( סובל כנראה מאוטם שריר הלב‬
‫ג‪ .‬דיסקציה של אבי העורקים‪ ,‬פריקרדיטיס‪ ,‬אוטם ריאתי וחזה אוויר יכולים לגרום לכאב עז‬
‫בחזה‬
‫ד‪ .‬אבחנה של תעוקת חזה נעשית באנמנזה בלבד ואין צורך בבדיקות מעבדה לאשש האבחנה‬
‫ה‪ .‬כל התשובות נכונות‬
‫‪.76‬‬
‫במחלת סוכרת הפגיעה בלב מתבטאת ב‪:‬‬
‫א‪ .‬ירידה בקצב הלב‬
‫ב‪ .‬עליה בתפוקת הלב‬
‫ג‪ .‬שיפור בהתכווצות שריר הלב‬
‫ד‪ .‬ירידה בתפקוד חדר שמאל ושכיחות גבוהה של מחלה טרשתית כלילית‬
‫ה‪ .‬עליה בלחץ דם סיסטולי‬

‫‪.77‬‬
‫הטיפול בתרופות ממשפחת הסטטינים‪:‬‬
‫א‪ .‬מוריד כמות רצפטורים ל‪ LDL-‬על פני הפטוציטים‬
‫ב‪ .‬מקטין ספיגת שומנים מהדיאטה בפעולה במעי‬
‫ג‪ .‬מגדיל הפרשת כולסטרול במלחי המרה ומונע ספיגתם‬
‫ד‪ .‬מקטין ייצור ‪ LDL‬על ידי הפטוציטים‬
‫ה‪ .‬מקטין שכיחות אוטמים רק בחולים שעברו אוטם בעבר‬
‫‪.78‬‬
‫תלונות שכיחות בהיצרות מסתם מיטרלי‪:‬‬
‫א‪ .‬קוצר נשימה‬
‫ב‪ .‬דופק לא סדיר‬
‫ג‪ .‬תסחיפים‬
‫ד‪ .‬כל התשובות נכונות‬

‫‪.79‬‬
‫בבדיקתו של חולה עם קרדיומיופטיה היפרטרופית‪:‬‬
‫א‪ .‬גל הדופק איטי וממושך‬
‫ב‪ .‬לחץ הדם נמוך באופן טיפוסי‬
‫ג‪ .‬יש אי ספיקה של מסתם אבי העורקים‬
‫ד‪ .‬בחלק מהחולים יש אוושה סיסטולית המתגברת בעמידה‬

‫עמוד ‪ 12‬מתוך ‪21‬‬


‫‪.80‬‬
‫בבדיקתו של חולה עם אי ספיקת מסתם אבי העורקים‪:‬‬
‫א‪ .‬הדופק הוא איטי וממושך‬
‫ב‪ .‬הדופק מכה )מהיר וגבוה(‬
‫ג‪ .‬לחץ הדם הדיאסטולי גבוה בצורה טיפוסית‬
‫ד‪ .‬האוושה הטיפוסית רמה לכל אורך הדיאסטולה‬

‫פרק מספר ‪ :4‬נוירולוגיה‬

‫‪.81‬‬
‫כל הבאים הם סימני ‪ ,Viral encephalitis‬פרט ל‪:‬‬
‫א‪Pperivascular lymphocytic infiltrates .‬‬
‫ב‪Microglial nodules .‬‬
‫ג‪Neuronophagia .‬‬
‫ד‪Neurofibrillary tangles .‬‬
‫ה‪Inclusion bodies .‬‬
‫‪.82‬‬
‫כל המשפטים הבאים לגבי ‪ Meduloblastoma‬נכונים פרט ל‪:‬‬
‫א‪ .‬גידול גליאלי‪ ,‬שמקורו באסטרוציטים‬
‫ב‪ .‬גידול של ילדים עד גיל ‪16‬‬
‫ג‪ .‬גידול של מוחון )‪(cerebellum‬‬
‫ד‪ .‬הגידול בנוי מתאים קטנים ומאורכים‬
‫ה‪ .‬חמש שנים לאחר טיפול‪ ,‬היום חיים כ‪ 50-70%-‬מהחולים‬

‫‪.83‬‬
‫גבר בן ‪ 58‬נותח בגלל גידול גדול ב‪ Frontal lobe-‬בבדיקה פתולוגית נראית אטיפיה‪ ,‬מיטוזות‪ ,‬נמק‬
‫ופרוליפרציה של אנדותל של כלי הדם בגידול‪ .‬הצביעה ל‪ GFAP-‬חיובית בתאי הגידול‪ .‬מה האבחנה‬
‫שלך?‬
‫א‪Astrocytoma grade 2 .‬‬
‫ב‪Glioblastoma multiforme .‬‬
‫ג‪Meduloblastoma .‬‬
‫ד‪Malignant meningioma .‬‬
‫ה‪ .‬גרןרה של ‪carcinoma‬‬

‫‪.84‬‬
‫כל הבאים אופייניים למחלת ‪ ,Alzheimer‬פרט ל‪:‬‬
‫א‪Senile (neuritic) plaques .‬‬
‫ב‪Neurofibrillary tangles .‬‬
‫ג‪Spongiform change .‬‬
‫ד‪Amyloid angiopathy .‬‬
‫ה‪Granulovscular degeneration .‬‬

‫‪.85‬‬
‫סמן מה נכון לגבי מחלת ‪?Alzheimer‬‬
‫א‪ Aβ-peptide .‬הינו תוצר של פעילות האנזים ‪reductase‬‬
‫ב‪ – Tau .‬חלבון פתולוגי‪ ,‬שלא נמצא בתא תקין‬
‫ג‪ .‬למולקולה של ‪ APP‬יש חלק תוך וחוץ תאי וטרנסמברנלי‬

‫עמוד ‪ 13‬מתוך ‪21‬‬


‫ד‪ Amyloid plaques .‬לא מכילים אקסונים‬
‫ה‪ .‬מספר ה‪ Amyloid plaques-‬במוח מראה קורלציה הכי טובה עם דרגת ה‪Dementia-‬‬

‫‪.86‬‬
‫אישה בת ‪ ,56‬סובלת מיתר לחץ דם‪ ,‬הגיעה לחדר מיון במצב של תרדמת שהתפתח תוך זמן קצר מאוד‬
‫בעבודה‪ .‬ב‪ CT-‬נראה שטף דם סובארכנואידלי נרחב בבסיס המוח‪ .‬מה הסיבה הסבירה ביותר למצב‬
‫זה?‬
‫א‪Acute bacterial meningitis .‬‬
‫ב‪Tuberculous meningitis .‬‬
‫ג‪Amyloid angiopathy .‬‬
‫ד‪Rupture aneurysm .‬‬
‫ה‪Vasculitis .‬‬
‫‪.87‬‬
‫סמן‪ ,‬מה נכון לגבי ‪?CMV encephalitis‬‬
‫א‪ .‬שכיח אצל ילדים בני ‪5-10‬‬
‫ב‪ .‬גורם לנמק המורגי מסביב לחדרים של המוח אצל חולים עם מערכת חיסון מדוכאת‬
‫ג‪ .‬במוח נראו גרנולומות עם תאי ענק‬
‫ד‪ .‬לא מערב את חוט השדרה והשורשים‬
‫ה‪ .‬האבחנה אפשרית בדרך כלל רק באמצעות ‪PCR‬‬

‫‪.88‬‬
‫מה מבדיל בין ‪ Astrocytoma grade III‬ל‪? Glioblastoma-‬‬
‫_____ _______________________‬

‫‪.89‬‬
‫סלייד מספר ‪:4‬‬
‫_____________________________‬

‫‪.90‬‬
‫תמונה מספר ‪ .5‬גבר בן ‪ 75‬עם דמנציה‪ .‬מהו השינוי בתמונה?‬
‫_____________________________‬

‫‪.91‬‬
‫תמונה מספר ‪ .6‬ילד בן ‪ 6‬עם גידול במוחון‪ .‬לפניך צביעה חיובית ל‪ .Synaptophysin-‬מה האבחנה?‬
‫_____________________________‬

‫‪.92‬‬
‫תמונה מספר ‪ .7‬בת ‪ 18‬עם מחלה נוירולוגית ממושכת‪ .‬לפניך חתך דרך גזע המוח )צביעת ‪taxol-fast‬‬
‫‪ (blue‬מהו השינוי שבולט בתמונה?‬
‫_____________________________‬

‫‪.93‬‬
‫תמונה מספר ‪ .8‬חולה עם דמנציה‪ .‬איך נקרא המבנה שרואים בציטופלזמה של הנוירון?‬
‫_____________________________‬

‫‪.94‬‬
‫תמונה מספר ‪ .9‬חולה בן ‪ 10‬עם נגעים בריאות ובמוח‪ .‬לפניך ביופסיה מרקמת מוח‪ .‬מה אבחנתך?‬
‫_____________________________‬

‫עמוד ‪ 14‬מתוך ‪21‬‬


‫‪.95‬‬
‫פלקסיה של הצוואר בחולה ‪ MS‬עלולה לגרום ל‪:‬‬
‫א‪ .‬תנועה דיסטונית של הגפיים‬
‫ב‪ .‬הופעת כאב בפנים‬
‫ג‪ .‬תחושת זרם חשמלי המתפשטת לאורך עמ"ש‬
‫ד‪ .‬ריקון פתולוגי של שלפוחית השתן‬
‫ה‪ .‬החמרה של הסימנים הצרבליים הקיימים‬

‫‪.97‬‬
‫בבדיקת שדות ראיה בקונפרונטציה‪ ,‬בן ‪ 76‬אינו מזהה גירויים המוצגים בשדה הראייה השמאלי‬
‫למרות שבבדיקה שדה הראיה של כל עין בנפרד תקין‪ .‬מקום הנזק‪:‬‬
‫א‪ .‬אונה פריאטלית ימנית‬
‫ב‪ .‬אונה פרונטלית ימנית‬
‫ג‪ .‬אונה פריאטלית שמאלית‬
‫ד‪ .‬אונה פרונטלית שמאלית‬
‫ה‪ .‬אונה טמפורלית‬

‫‪.98‬‬
‫סמן את המבנה האנטומי אשר אינו מעורב ביצירת הזיכרון‪:‬‬
‫א‪Mamillary bodies .‬‬
‫ב‪Fornix .‬‬
‫ג‪Basal forebrain .‬‬
‫ד‪Putamen .‬‬
‫ה‪Thalamus .‬‬
‫‪.99‬‬
‫‪ MRI‬משמש לאיבחון גידולים‪:‬‬
‫א‪ .‬בשל רגישותו הגבוהה לדימום תת חריף‬
‫ב‪ .‬בשל רגישותו הרבה לשוני בין רקמות‬
‫ג‪ .‬הודות ליכולתו לזהות ולמפות כלי דם‬
‫ד‪ .‬בשל יכולתו לדייק יותר מ‪ CT-‬במידת התפשטות הגידול‬
‫ה‪ .‬כל ‪ 4‬האפשרויות הראשונות‬
‫ו‪ .‬אף לא אחת מהאפשרויות‬

‫‪ .100‬ממצאים אופייניים בבדיקת ‪ NCS-EMG‬בנוירופתיה דה‪-‬מיילנטיבית‪:‬‬


‫א‪ .‬היחידות המוטוריות בבדיקת ‪ EMG‬מראות פוטנציאלים גבוהים‬
‫ב‪ .‬תיתכן ירידה בגיוס יחידות מוטוריות‬
‫ג‪ .‬מהירויות ההולכה תקינות‬
‫ד‪ .‬משרעות המוטוריות נמוכות‬
‫ה‪ .‬פעילות ספונטנית בשרירים‬
‫‪.101‬‬
‫חולה בת ‪ 40‬מגיעה למרפאה עקב כאבים בעין‪ .‬בבדיקה – עפעף צנוח‪ ,‬אישון מגיב לאור‪ .‬מה האבחנה‬
‫המתאימה ביותר?‬
‫א‪ .‬גידול באונה הפריאטלית עם לחץ מוגבר על גבול הרניאציה דרך הטנטוריום‬
‫ב‪ .‬מפרצת עורקית הלוחצת על העצב השלישי‬
‫ג‪ .‬נוירופתיה של העצב השלישי‬
‫ד‪ .‬נוירופתיה של העצב השישי‬

‫עמוד ‪ 15‬מתוך ‪21‬‬


‫‪.102‬‬
‫חומרי אילחוש מקומי )כמו לידוקאין( מפסיקים כאב חריף על ידי‪:‬‬
‫א‪ .‬מניעת יצירת פרוסטגלנדינים‬
‫ב‪ .‬חסימת ההולכה בסיבי העצב הדקים‬
‫ג‪ .‬חסימת ההולכה בסיבי העצב העבים‬
‫ד‪ .‬שיחרור אנדורפינום‬
‫ה‪ .‬אף תשובה נכונה‬
‫‪.103‬‬
‫הדרך הנכונה ביותר להעריך עוצמת כאב הינה‪:‬‬
‫א‪ .‬אומדן באמצעות סרגל עוצמות כאב‬
‫ב‪ .‬לצפות בחולה ולשפוט עפ"י התנהגותו‬
‫ג‪ .‬לעקוב אחר סימנים אוטונומיים כמו לחץ דם דופק וקצב נשימה‬
‫ד‪ .‬אין צורך לאמוד כאב משום שהוא ממילא סוביקטיבי‬
‫‪.104‬‬
‫כל המשפטים הבאים בנושא ‪ Tremor‬נכונים‪ ,‬פרט ל?‬
‫א‪ .‬תנועה בלתי רצונית שאינה ריטמית ולא רגולרית והשכיחה ביותר מהתנועות הבלתי רצוניות‬
‫ב‪ Midbrai, tremor (rubral tremor) .‬הוא רעד גס‪ ,‬שכיח במחלות וסקולריות של גזע המוח ו‪-‬‬
‫‪MS‬‬
‫ג‪ E.T (Essential tremor) .‬הוא רעד בד"כ משפחתי )ב‪ 70%-‬מהמקרים(‬
‫ד‪ .‬הטיפול ברעד הוא סימפטומטי ע"י דרלין ופרמידון והוא בד"כ יעיל‬
‫ה‪ 2 .‬שיאים להופעת מחלת ה‪ – E.T-‬הראשון בגיל ההתבגרות והשני בגיל המבוגר‪late ) .‬‬
‫‪(adulthood‬‬
‫‪.105‬‬
‫כל המשפטים הבאים נכונים בקשר ל‪ ,Subacute combined degeneration (SACD) -‬פרט ל?‬
‫א‪ .‬בעיקר הפרעה בתחושה עמוקה‬
‫ב‪ .‬בדרך כלל ביטוי קליני אסימטרי‬
‫ג‪ .‬חוסר ויטמין ‪ B12‬מהווה את הסיבה לתסמונת זו‬
‫ד‪ .‬ערות החזרים עם בבינסקי חיובי ואטקסיה סנסורית הם סימנים שכיחים‬
‫ה‪ .‬הטיפול הוא ‪ replacement‬של ויטמין ‪ B12‬ע"י זריקת ‪ IM‬בד"כ‬
‫‪.106‬‬
‫חולה בן ‪ 30‬מתקבל לחדר מיון בשל פרכוס כללי‪ .‬מעברו הרפואי‪ ,‬היסטוריה של כאבי ראש ב‪10-‬‬
‫החודשים האחרונים‪ ,‬כן ארוע של איבוד הכרה ככל הנראה אפילפטי‪ .‬איזו מבין האבחנות הבאות ניתן‬
‫בברור לשלול על הסף?‬
‫א‪Glioblastoma .‬‬
‫ב‪Meningioma .‬‬
‫ג‪Oligodendroglioma .‬‬
‫ד‪low grade Astrocytome .‬‬
‫ה‪Pituitary adenoma .‬‬
‫‪.107‬‬
‫אילו מבין החולים הבאים יוגדר כסובל מאפילפסיה?‬
‫א‪ .‬חולה שסבל מהתקף אפילפטי ממושך )סטטוס אפילפטיקוס( לראשונה בחייו‬
‫ב‪ .‬חולהשסבל מ‪ 8-‬התקפים אפילפטים בחודש האחרון על רקע נטילת התרופה האנטי‬
‫פסיכוטית קלוזפין‬
‫ג‪ .‬חולה שסבל מ‪ 3-‬התקפים אפילפטיים בשנה האחרונה שהתבטאו בפרכוס בכף יד שמאל ללא‬
‫שינוי במצב ההכרה‬

‫עמוד ‪ 16‬מתוך ‪21‬‬


‫ד‪ .‬חולה הסובל מכאבי ראש תכופים )‪ 1-2‬לשבוע( ובדיקת ‪ EEG‬הדגימה הפרעות אפילפטיות‬
‫מובהקות‬
‫ה‪ .‬אף אחד מהחולים הנ"ל לא יוגדר כסובל מאפילפסיה‬
‫‪.108‬‬
‫חולה שפנה לחדר המיון בשל התקף טוני קלוני כללי שני בחייו באנמנזה מפורטת עולה שהחולה סבל‬
‫בעבר ממיוקלוניות ואבסנסים‪ .‬איזה תרופה תבחר לתת לחולה זה?‬
‫א‪ .‬קרבמזפין )טגרטול(‬
‫ב‪ .‬קלונזפם )קלונקס(‬
‫ג‪ .‬חומצה ולפרואית )דפלפט(‬
‫ד‪ .‬סומטריפטן )אימיטרקס(‬
‫ה‪ .‬פנוברביטל )לומינל(‬
‫‪.109‬‬
‫איזה מהמשפטים הבאים אינם נכונים לגבי הטיפול של מחלת פרקינסון עם ‪?L-Dopa‬‬
‫א‪ L-Dopa .‬אינו מונע מוות של נוירונים דופמינרגיים‬
‫ב‪ .‬רמות של ‪ L-Dopa‬בפלזמה מושפעות מכמות החלבון בתזונה‬
‫ג‪ L-Dopa .‬חודר את המחסום דם‪-‬מוח‬
‫ד‪ L-Dopa .‬רינו מפורק בפריפריה של הגוף‬
‫ה‪ .‬המשפטים א‪+‬ב‬
‫‪.110‬‬
‫הצביע על תופעת הלוואי הנפוצה ביותר עם טיפול ממושך )שנים( ב‪: L-Dopa-‬‬
‫א‪ .‬אריתמיה לבבית‬
‫ב‪ .‬תנועות בלתי רצוניות )‪(Dyskinesia‬‬
‫ג‪ .‬בחילה‬
‫ד‪ .‬לחץ דם נמוך כשקמים )‪(Orthostatic hypotension‬‬
‫ה‪ .‬כל התשובות אינן נכונות‬
‫‪.111‬‬
‫‪ Carbidopa‬מפחיתה את‪:‬‬
‫א‪ .‬הפעילות של ‪ Dopa-Decarboxylase‬במערכת העצבים המרכזית‬
‫ב‪ .‬המינון של ‪ L-Dopa‬הנחוץ לקבלת השפעה תרפויטית‬
‫ג‪ .‬החמרה של ה‪ Dyskinesia-‬המופיע עם טיפול ב‪L-Dopa-‬‬
‫ד‪ .‬הזמן המתנה עד לקבלת ההשפעות התרפויטיות של ‪L-Dopa‬‬
‫ה‪ .‬כל התשובות אינן נכונות‬
‫‪.112‬‬
‫סמן את התשובה הלא נכונה‪:‬‬
‫א‪ .‬אגוניסט דופמינרגי בשפ ברומוכריפטין יכול לגרום לעליה בלחץ דם‬
‫ב‪ Amantadine .‬יכולה לשמש כתרופה למחלת פרקינסון משום שהיא גורמת לעליה של דופאמין‬
‫במוח‬
‫ג‪ .‬תרופות נוגדות דיכאון כגון ‪ serotonin reuptake inhibitors‬יכולות לגרום לסימפטומים של‬
‫פרקינסון‬
‫ד‪ .‬קיימים מחקרים שטוענים שמעכבי ‪ MAOB‬יכולים להוריד התמותה של נוירונים במחלת‬
‫פרקינסון‬
‫ה‪ .‬התרופות מהסוג אנטי‪-‬כולינרגי גורמות לתופעות לוואי מועטות‪ .‬עובדה זו מאפשרת‬
‫להשתמש בהם לטיפול במגוון רב של אוכלוסיות של חולי פרקינסון‬

‫עמוד ‪ 17‬מתוך ‪21‬‬


‫תרשים מספר ‪:1‬‬

‫תרשים מספר ‪:2‬‬

‫עמוד ‪ 18‬מתוך ‪21‬‬


‫תמונה מספר ‪:3‬‬

‫תמונה מספר ‪:4‬‬

‫עמוד ‪ 19‬מתוך ‪21‬‬


‫תמונה מספר ‪:5‬‬

‫עמוד ‪ 20‬מתוך ‪21‬‬


‫תשובות אפשריות‪:‬‬

‫ב‬ ‫‪87‬‬ ‫תשובות‬ ‫תשובות מספר‬ ‫תשובות מספר‬ ‫מספר‬


‫פרוליפרציה‬ ‫אפשריות‬ ‫אפשריות שאלה‬ ‫אפשריות שאלה‬ ‫שאלה‬
‫של אנדותל‬ ‫ב‬ ‫‪29‬‬ ‫מבואות‬
‫כלי דם‪,‬‬ ‫‪88‬‬ ‫ד‬ ‫‪58‬‬ ‫ג‬ ‫‪30‬‬ ‫ב‬ ‫‪1‬‬
‫נמק‪,‬‬ ‫ג‬ ‫‪59‬‬ ‫ב‬ ‫‪31‬‬ ‫‪2‬‬
‫______‬ ‫ה‬ ‫‪60‬‬ ‫ג‬ ‫‪32‬‬ ‫ב‬ ‫‪3‬‬
‫‪89‬‬ ‫ב‬ ‫‪61‬‬ ‫ג‬ ‫‪33‬‬ ‫ב‬ ‫‪4‬‬
‫‪active‬‬ ‫ה‬ ‫‪62‬‬ ‫ב‬ ‫‪34‬‬ ‫א‬ ‫‪5‬‬
‫‪90‬‬
‫‪plaque‬‬ ‫א‬ ‫‪63‬‬ ‫ד‬ ‫‪35‬‬ ‫‪6‬‬
‫‪91‬‬ ‫ג‬ ‫‪64‬‬ ‫ג‬ ‫‪36‬‬ ‫‪7‬‬
‫‪92‬‬ ‫‪65‬‬ ‫ד‬ ‫‪37‬‬ ‫‪8‬‬
‫‪NK‬‬ ‫‪93‬‬ ‫ה‬ ‫‪66‬‬ ‫ד‬ ‫‪38‬‬ ‫‪9‬‬
‫?‪TB‬‬ ‫‪94‬‬ ‫א‬ ‫‪67‬‬ ‫ד‬ ‫‪39‬‬ ‫ג‬ ‫‪10‬‬
‫ג‬ ‫‪95‬‬ ‫ג‬ ‫‪68‬‬ ‫ב‬ ‫‪40‬‬ ‫ג‬ ‫‪11‬‬
‫ד‬ ‫‪96‬‬ ‫‪MI‬‬ ‫‪69‬‬ ‫‪41‬‬ ‫ד‬ ‫‪12‬‬
‫א‬ ‫‪97‬‬ ‫ד‬ ‫‪70‬‬ ‫‪42‬‬ ‫ד‬ ‫‪13‬‬
‫ד‬ ‫‪98‬‬ ‫ב‬ ‫‪71‬‬ ‫‪43‬‬ ‫ג‬ ‫‪14‬‬
‫ה‬ ‫‪99‬‬ ‫ד‬ ‫‪72‬‬ ‫‪44‬‬ ‫ד‬ ‫‪15‬‬
‫ב‬ ‫‪100‬‬ ‫ה‬ ‫‪73‬‬ ‫‪45‬‬ ‫ג‬ ‫‪16‬‬
‫ג‬ ‫‪101‬‬ ‫ה‬ ‫‪74‬‬ ‫ג‬ ‫‪46‬‬ ‫ד‬ ‫‪17‬‬
‫ב‬ ‫‪102‬‬ ‫ב‬ ‫‪75‬‬ ‫ב‬ ‫‪47‬‬ ‫ריאות‬
‫א‬ ‫‪103‬‬ ‫ד‬ ‫‪76‬‬ ‫‪48‬‬ ‫ה‬ ‫‪18‬‬
‫א‬ ‫‪104‬‬ ‫ד‬ ‫‪77‬‬ ‫‪49‬‬ ‫ב‬ ‫‪19‬‬
‫ב‬ ‫‪105‬‬ ‫ד‬ ‫‪78‬‬ ‫‪50‬‬ ‫ב‬ ‫‪20‬‬
‫ה‬ ‫‪106‬‬ ‫ד‬ ‫‪79‬‬ ‫‪51‬‬ ‫א‬ ‫‪21‬‬
‫ג‬ ‫‪107‬‬ ‫ב‬ ‫‪80‬‬ ‫ג‪+‬ב‬ ‫‪52‬‬ ‫ד‬ ‫‪22‬‬
‫ג‬ ‫‪108‬‬ ‫נוירולוגיה‬ ‫לב‬ ‫ה‬ ‫‪23‬‬
‫ד‬ ‫‪109‬‬ ‫ד‬ ‫‪81‬‬ ‫ד‬ ‫‪53‬‬ ‫ה‬ ‫‪24‬‬
‫ב‬ ‫‪110‬‬ ‫א‬ ‫‪82‬‬ ‫ד‬ ‫‪54‬‬ ‫ג‬ ‫‪25‬‬
‫ב‬ ‫‪111‬‬ ‫ב‬ ‫‪83‬‬ ‫א‬ ‫‪55‬‬ ‫ג‬ ‫‪26‬‬
‫ה‬ ‫‪112‬‬ ‫ג‬ ‫‪84‬‬ ‫ג‬ ‫‪56‬‬ ‫ד‬ ‫‪27‬‬
‫ג‬ ‫‪85‬‬ ‫ב‬ ‫‪57‬‬ ‫א‬ ‫‪28‬‬
‫ד‬ ‫‪86‬‬

‫עמוד ‪ 21‬מתוך ‪21‬‬


‫‪ CIP‬קרדיולוגיה‬
‫מקבץ ‪I‬‬
‫‪ CIP‬קרדיולוגיה ‪2007-2008‬‬

‫‪VI‬‬ ‫‪V‬‬ ‫‪IV‬‬ ‫‪III‬‬ ‫‪II‬‬ ‫‪I‬‬ ‫אבחנה‬


‫טיפול‬ ‫פתולוגיה‬ ‫הדמיה‬ ‫מעבדה‬ ‫בדיקה‬ ‫אנמנזה‬
‫ומהלך‬ ‫גופנית‬
‫‪Hypertrophic‬‬
‫‪Obstructive‬‬
‫‪Cardiomyopathy‬‬
‫אי ספיקה מיטרלית‬
‫עם אנדוקרדיטיס‬
‫‪Aortic Stenosis‬‬
‫‪Unstable Angina‬‬
‫אוטם שריר הלב‬

‫‪ - I‬אנמנזה‬
‫א‪ .‬חולה בן ‪ 46‬עם כאב לוחץ בחזה שנמשך ‪ 10‬דקות במנוחה‬
‫ב‪ .‬חולה בן ‪ 30‬אשר הגיע לחדר המיון בזמן משחק כדורסל‬
‫ג‪ .‬חולה בן ‪ 78‬עם היסטוריה של ניתוח מעקפים‪ ,‬אשר אושפז עקב כאב בחזה שנמשך ‪40‬‬
‫דקות במנוחה ולווה בקוצר נשימה‬
‫ד‪ .‬חולה בת ‪ 35‬אשר אושפזה עקב חום שנמשך שבועיים וקוצר נשימה‬
‫ה‪ .‬חולה בן ‪ 65‬אשר סובל מקוצר נשימה והרגשת מחנק בהליכה בחודשים האחרונים‪ ,‬ללא‬
‫גורמי סיכון למחלת לב כלילית‬

‫‪ - II‬בדיקה גופנית‬
‫א‪ .‬בבדיקתה מצב כללי בינוני ‪ ,‬ל"ד ‪ 110/60‬ממ"כ‪ .‬חום ‪ ,38‬בבדיקת חוד הלב מורחב‪.‬‬
‫בהאזנה קול שלישי ואוושה סיסטולית רמה‬
‫ב‪ .‬בבדיקה גופנית מצב כללי טוב‪ .‬סימני שפשוף על הפנים )עקב נפילה(‪ .‬גל דופק מכה‪ ,‬נשמע‬
‫קול רביעי ואוושה סיסטולית רמה המתגברת בעמידה‬
‫ג‪ .‬בבדיקה גופנית מצב כללי טוב‪ ,‬שוכב פרקדן‪ ,‬גל דופק איטי ובעל נפח נמוך‪ .‬בהאזנה נשמע‬
‫קול רביעי‪ ,‬אוושה סיסטולית בבסיס הלב רמה בצורת יהלום‬
‫ד‪ .‬בבדיקה גופנית מצב כללי טוב‪ ,‬ל"ד ‪ 150/70‬ממ"כ‪ .‬לא נמושו דפקים בטיבאליס האחורי‬
‫בשתי הרגליים‬
‫ה‪ .‬בבדיקה גופנית מצב כללי בינוני‪ ,‬חוור ומזיע‪ ,‬צלקת ניתוחית על פני הסטרנום‬

‫‪ - III‬מעבדה‬
‫א‪ .‬ב‪ DNA-‬שבודד מדם ורידי נמצאה מוטציה בשרשרת כבדה של מיוזין‬
‫ב‪ ECG .‬תקין‬
‫ג‪ .‬סימני היפרטרופיה של חדר שמאל ב ‪ECG‬‬
‫ד‪ .‬תרשים ‪ ECG‬מס' ‪1‬‬
‫ה‪ .‬בתרביות דם צמח סטרפטוקוקוס וירידנס‬

‫‪ - IV‬הדמיה‬
‫א‪ .‬באקו היפרטרופיה סימטרית של חדר שמאל ובצנתור מפל לחצים ממוצע של ‪ 55‬ממ"כ‬
‫בין אבי העורקים לחדר שמאל בזמן הסיסטולה‬
‫ב‪ .‬באקו היפרטרופיה אסימטרית של חדר שמאל‬
‫ג‪ .‬באקו דופלר זרימה סיסטולית מחדר שמאל לעליה שמאלית‬
‫ד‪ .‬באנגיוגרפיה כלילית חסימה מלאה של העורק הקדמי היורד‬
‫ה‪ .‬באנגיוגרפיה כלילית היצרות של ‪ 95%‬בעורק הקדמי היורד הרחוקני‬
‫‪ - V‬פתולוגיה‬
‫א‪ .‬היפרטרופיה של חדר שמאל עם ספטום מאוד מעובה ועליה שמאלית מוגדלת‬
‫ב‪ .‬הסתיידות של המסתם האאורטלי‬
‫ג‪ .‬אזור של נמק בדופן הקדמית של חדר שמאל‬
‫ד‪ .‬מסתם מעובה ובצקתי עם ממצא של חיידקים‬
‫ה‪ .‬קרע של טרשת בעורק הקדמי היורד עם קריש דם לא חוסם‬

‫‪ - VI‬טיפול ומהלך‬
‫א‪ .‬טופל ב‪ TPA-‬אך עקב כאב בחזה ול"ד נמוך עבר אנגיוגרפיה כלילית ופתיחה באמצעות בלון‬
‫ותומך של העורק הקדמי היורד‬
‫ב‪ .‬טיפול אנטיביוטי של ‪ 6‬שבועות‪ .‬מעכבי ‪ ACE‬עקב אי ספיקת הלב‬
‫ג‪ .‬ביום השני לאשפוז התלונן על כאב בחזה ותוך שתי דקות איבד הכרה ובאק"ג הקצב‬
‫בתרשים מס' ‪.2‬‬
‫ד‪ .‬עבר ניתוח להחלפת מסתם‬
‫ה‪ .‬ביום השלישי לאשפוזו הותקן מכשיר דה‪-‬פיברילטור‬

‫תשובות אפשריות‪:‬‬

‫‪VI‬‬ ‫‪V‬‬ ‫‪IV‬‬ ‫‪III‬‬ ‫‪II‬‬ ‫‪I‬‬ ‫אבחנה‬


‫טיפול‬ ‫פתולוגיה‬ ‫הדמיה‬ ‫מעבדה‬ ‫בדיקה‬ ‫אנמנזה‬
‫ומהלך‬ ‫גופנית‬
‫‪Hypertrophic‬‬
‫ה‬ ‫א‬ ‫ב‬ ‫א‬ ‫ב‬ ‫ב‬ ‫‪Obstructive‬‬
‫‪Cardiomyopathy‬‬
‫אי ספיקה מיטרלית‬
‫ב‬ ‫ד‬ ‫ג‬ ‫ה‬ ‫א‬ ‫ד‬ ‫עם אנדוקרדיטיס‬
‫ד‬ ‫ב‬ ‫א‬ ‫ג‬ ‫ג‬ ‫ה‬ ‫‪Aortic Stenosis‬‬
‫ג‬ ‫ה‬ ‫ה‬ ‫ב‬ ‫ד‬ ‫א‬ ‫‪Unstable Angina‬‬
‫א‬ ‫ג‬ ‫ד‬ ‫ד‬ ‫ה‬ ‫ג‬ ‫אוטם שריר הלב‬
‫מקבץ ‪II‬‬
‫מקבץ ‪2007-2008 - II‬‬
‫פרק מספר ‪ : 1‬עיכול‪-‬כבד‬
‫‪ .1‬סיבוכי מחלת הצליאק כוללים את כל הבאים פרט ל‪:‬‬
‫א‪ .‬ממאירות‬
‫ב‪ .‬ירידה בצפיפות העצם‬
‫ג‪ .‬הפרעות במתן שתן‬
‫ד‪ .‬הפרעות בפוריות‬

‫‪ .2‬בת ‪ ,18‬סובלת מקוליטיס כיבית )קוליטיס אולצרוזה( מזה שנתיים‪ .‬במהלך השנתיים‬
‫סבלה משתי התלקחויות‪ ,‬שהתבטאו בשלשול דמי והצריכו מתן סטרואידים תוך ורידי‪.‬‬
‫מטופלת ברפסל ופורינטול מאז האירוע השני‪ .‬פנתה בשל שלשול דמי עד ‪ 8‬פעמים ביום‬
‫ומבקשת טיפול בסטרואידים שעזרו לה בעבר‪ .‬מה דעתך ?‬
‫א‪ .‬נראה שמדובר בהתלקחות של המחלה ולכן כדאי מתן סטרואידים‬
‫ב‪ .‬נראה שקיימת אתיולוגיה שונה לשלשול כעת ויש צורך בבירור‬
‫ג‪ .‬בשלב ראשון כדאי להעלות את מינון ה‪ ASA-5-‬למינון מקסימאלי ולעקוב‬
‫ד‪ .‬כדאי לבדוק רמות מטבוליטים של ‪ MP-6‬ולפי התוצאות לקבל החלטה טיפולית‬
‫ה‪ .‬קורס שלישי של סטרואידים אינו מומלץ ולכן כדאי לשקול טיפול ברמיקייד‬

‫‪ .3‬אחד מהמצבים הבאים מחייב הספקת טיפול תמידית ב‪ 6-‬מרקפטופורין או אימורן‪:‬‬


‫א‪ .‬פנקריאטיטיס‬
‫ב‪ .‬לויקופניה‬
‫ג‪ .‬תרומבוציטופניה‬
‫ד‪ .‬הפרעה בתפקודי הכבד‬
‫ה‪ .‬הופעת בחילה‬

‫‪ .4‬מחלת קרון יכולה להסתמן כ‪:‬‬


‫א‪ .‬כמו אפנדיציטיס‬
‫ב‪ .‬כפרפורציה של מעי‬
‫ג‪ .‬אבצס פרי‪-‬אנלי‬
‫ד‪ .‬ככאבים בבטן ימנית תחתונה מלווים בחום וירידה במשקל‬
‫ה‪ .‬כל התשובות נכונות‬

‫‪ .5‬ההסתמנות השכיחה של מחלת כיב היא בד"כ‪:‬‬


‫א‪ .‬כאב סביב הטבור הקורן לגב ומלווה בהקאה‬
‫ב‪ .‬כאב אפיגסטרי המוקל על ידי אוכל‪ ,‬מחריף ברעב ומעיר בלילה משינה‬
‫ג‪ .‬כאב מפלח כסכין ומלווה ברגישות רבה ובטן חריפה‬
‫ד‪ .‬דמם מדרכי העיכול‬

‫‪ .6‬גבר בן ‪ 55‬הסובל מצרבת שנים רבות‪ ,‬מפתח לאחרונה דיספאגיה‪ .‬מהו הצעד הנכון‬
‫ביותר?‬
‫א‪ .‬להעלות את מינון התרופות לצרבת ולהוסיף טיפול פרוקינטי‬
‫ב‪ .‬צריך לשנות את אורח החיים‪ :‬ארוחות קטנות‪ ,‬הפסקת עישון וירידה במשקל‬
‫ג‪ .‬גסטרוסקופיה‬
‫ד‪ .‬מעקב בלבד‬
‫‪ .7‬אישה בת ‪ 50‬עם דיספפסיה וירידה במשקל‪ .‬מה הנכון לגבי הצעד הבא בברור ?‬
‫א‪ .‬מבחן נשיפה לחיידק הליקובקטר פילורי וטיפול בהתאם‬
‫ב‪ .‬טיפול אמפירי לחיסול החיידק הליקובקטר פילורי‬
‫ג‪ .‬מתן תרופות מסוג ‪proton pump inhibitors‬‬
‫ד‪ .‬גסטרוסקופיה‬

‫‪ .8‬מה מבין הבאים יכול להיות תוצאה של ‪:(GERD) Gastrointestinal reflux disease‬‬
‫א‪ .‬מחלת ריאות חסימתית )אסטמה(‬
‫ב‪ .‬מחלת שיניים‬
‫ג‪ .‬צרידות ו‪posterior laryngitis-‬‬
‫ד‪ .‬כל הנ"ל‬

‫‪ .9‬איזו מבדיקות ההדמיה הבאות אינה מתאימה לאבחנה‪:‬‬


‫א‪ .‬חסימת מעי דק ‪ -‬צילום בטן ריקה‬
‫ב‪ US - Acute cholecystitis .‬בטן עליונה‬
‫ג‪ - Acute diverticulitis .‬צילום בטן ריקה‬
‫ד‪ - Perforated duodenal ulcer .‬צילום בטן ריקה‬
‫ה‪ CT - Acute appendicitis .‬בטן‬

‫‪ .10‬איזו מההצהרות הבאות נכונה לגבי דיברטיקולוזיס ?‬


‫א‪ .‬מעל גיל ‪ 70‬התופעה השכיחה יותר אצל גברים‬
‫ב‪ .‬השמנה מהווה גורם סיכון לדיברטיקולוזיס של הגיל הצעיר‬
‫ג‪ .‬דימום מדיברטיקולי מתבטא בד"כ באנמיה ובדיקות דם סמוי חיובית‬
‫ד‪ .‬האזור הכי שכיח להופעת דיברטיקולוזיס הינו במעי הגס הרוחבי‬
‫ה‪ .‬האבחנה המדויקת ביותר של דיברטיקוליטיס הינה בקולונוסקופיה‬

‫‪ .11‬כל המצבים הבאים מתאפיינים בכאבים בבטן ימנית עליונה‪ ,‬פרט ל‪:‬‬
‫א‪Hepatitis .‬‬
‫ב‪Cholecystitis .‬‬
‫ג‪Cholangitis .‬‬
‫ד‪Right lower lobe pneumonia .‬‬
‫ה‪Crohn's disease .‬‬

‫‪ .12‬חום מאפיין במיוחד שלשול‪:‬‬


‫א‪ .‬זיהומי חיידקי‬
‫ב‪ .‬דלקתי כלשהו‬
‫ג‪ .‬פרזיטרי‬
‫ד‪ .‬אוסמוטי‬

‫‪ .13‬מי מהבאים מהווה המזהם החושב ביותר בחולים מאושפזים ?‬


‫א‪ .‬סלמונלה‬
‫ב‪ .‬שיגלה‬
‫ג‪ .‬פרזיטים שונים‬
‫ד‪ .‬קלוסטרידיום דפיצילה‬
‫‪ .14‬מה בדרך כלל הורם המשמעותי ביותר להחלטה על מתן טיפול אנטיביוטי לחולה הסובל‬
‫משלשול על רקע זיהומי ?‬
‫א‪ .‬סוג המזהם‬
‫ב‪ .‬מצבו הקליני של החולה‬
‫ג‪ .‬מספר הימים שהשלשול נמשך‬
‫ד‪ .‬מספר הלויקוציטים בספירת הדם‬

‫‪ .15‬תת תזונה משפיעה על מהלך האשפוז בעיקר לגבי‪:‬‬


‫א‪ .‬משך האשפוז‬
‫ב‪ .‬סיכויי החלמה‬
‫ג‪ .‬עלות אשפוז‬
‫ד‪ .‬סיכוי לסיבוכים‬
‫ה‪ .‬כל הנ"ל‬

‫‪ .16‬דימום ממע' העיכול העליונה סביר שיבוא לידי ביטוי בכל אחד מהבאים פרט ל‪:‬‬
‫א‪ .‬המטמזיס‬
‫ב‪ .‬המטוכזיה‬
‫ג‪ .‬מלנה‬
‫ד‪coffee ground .‬‬

‫‪ .17‬חיידק ההליקובקטר פילורי קשור לפתוגנזה של כל המחלות הבאות פרט ל‪:‬‬


‫א‪ .‬כיב פפטי בקיבה‬
‫ב‪ .‬לימפומה של הקיבה‬
‫ג‪ .‬אדנוקרצינומה של הקיבה‬
‫ד‪ .‬ליומיומה של הקיבה‬

‫‪ .18‬מבחן אוריאז לזיהוי הליקובקטר פילורי מסתמך על‪:‬‬


‫א‪ .‬נשיפה‬
‫ב‪ .‬ריאקצית צבע‬
‫ג‪ .‬ריאקצית דם‬
‫ד‪ .‬בדיקה היסטולוגית‬
‫‪ .19‬לחולה עם סרטן המעי הגס יש גרורות רק בריאות‪ .‬מיקום הגידול הראשוני היה כנראה‬
‫ב‪:‬‬
‫רקטום‬ ‫א‪.‬‬
‫סיגמה‬ ‫ב‪.‬‬
‫מעי רוחבי‬ ‫ג‪.‬‬
‫צקום‬ ‫ד‪.‬‬
‫‪ .20‬שכיחות סרטן המעי הגס היא‪:‬‬
‫פחותה אחרי גיל ‪ 60‬יחסית לגיל ‪50-60‬‬ ‫א‪.‬‬
‫פחותה בקרובי משפחה של חולי סרטן המעי‬ ‫ב‪.‬‬
‫שווה בין גברים ונשים‬ ‫ג‪.‬‬
‫יורדת ב‪ 80%-‬בחולה שאוכל הרבה סובין‬ ‫ד‪.‬‬
‫‪ .21‬מה הגורם העיקרי ל‪? Acute cholesystitis-‬‬
‫‪E.coli infection‬‬ ‫א‪.‬‬
‫‪Ascending cholangitis‬‬ ‫ב‪.‬‬
‫‪Cholelithiasis‬‬ ‫ג‪.‬‬
‫‪Cholesterolosis‬‬ ‫ד‪.‬‬
‫‪Drugs‬‬ ‫ה‪.‬‬
‫‪ .22‬באיזה מהמצבים קיים סיכון מוגבר להתפתחות סרטן של כיס המרה ?‬
‫א‪Xanthogranumatous cholecystitis .‬‬
‫ב‪Srrawberry gall bladder .‬‬
‫ג‪Rokintansky-aschoff sinuses .‬‬
‫ד‪Porcelain gall bladder .‬‬
‫ה‪Cholesterolosis .‬‬

‫‪ .23‬לגבי ‪ Carcinoid tumor‬הכל נכון פרט ל‪:‬‬


‫א‪ .‬התאים דומים זה לזה‬
‫ב‪ Carcinoid syndrome .‬שכיח בקרצינואיד של מערכת העיכול )נמצא ברב המקרים(‬
‫ג‪ Carcinoid .‬בתוספתן מתגלה ברב המקרים באופן מקרים‬
‫ד‪ .‬לגידולים פחות מ‪ 2-‬ס"מ יש פרוגנוזה טובה‬
‫ה‪ .‬ברב המקרים של ‪ gastrointestinal carcinoid tumor‬יש גרורות בכבד‬

‫‪ Adenocarcinoma .24‬של הקולון מתפתחת מ‪:‬‬


‫א‪Carcinoid tumor .‬‬
‫ב‪Lymphoid polyp .‬‬
‫ג‪Juvenile polyp .‬‬
‫ד‪Peutz jegher's polyp .‬‬
‫ה‪Adenoma .‬‬
‫‪ .25‬מה נכון לגבי סלייד מספר ‪:1‬‬
‫א‪Colloid adenocarcinoma .‬‬
‫ב‪Carcinoid tumor .‬‬
‫ג‪Juvenile polyp .‬‬
‫ד‪Pseudopolyp .‬‬
‫ה‪ .‬אף חד מהנ"ל‬

‫‪ .26‬לגבר בן ‪ 31‬יש כאבי בטן‪ steatorrhea ,‬ואנמיה‪ .‬בביופסיה של המעי הדק רואים השטחת‬
‫הוילי‪ ,‬דלקת וריבוי לימפוציטים בין תאי האפיתל‪ .‬האבחנה המועדפת היא‪:‬‬
‫א‪Crohn's disease .‬‬
‫ב‪Celiac disease .‬‬
‫ג‪Whipple disease .‬‬
‫ד‪Lymphoma .‬‬

‫‪ .27‬זיהום על‪-‬ידי הליקובקטר פילורי קשור לכל הבאים פרט ל‪:‬‬


‫א‪Chronic gastritis .‬‬
‫ב‪Gastric lymphoma .‬‬
‫ג‪Chronic peptic ulcer .‬‬
‫ד‪Acute gastric ulceration .‬‬
‫ה‪Gastric adenocarcinoma .‬‬

‫‪ .28‬כל הבאים מאפיינים ‪ Squamous cell carcinoma‬של הוושט פרט ל‪:‬‬


‫א‪ .‬מתפתח מוושט על שם ‪Barrett‬‬
‫ב‪ .‬המיקום הכי שכיח הוא בשליש האמצעי של הוושט‬
‫ג‪ .‬הפרוגנוזה גרועה ברב הגדול של המקרים‬
‫ד‪ .‬יותר שכיח בגברים‬
‫ה‪ .‬הצורה ה‪ Macroscopic-‬הכי שכיחה היא ‪Exophytic or polypoid‬‬
‫‪ .29‬מהי האבחנה שמופיעה בתמונה מספר ‪? 1‬‬
‫א‪Gastric hyperplastic polyp .‬‬
‫ב‪Fundic gland polyp .‬‬
‫ג‪Helicobacter pylori gastritis .‬‬
‫ד‪Barrett's esophagus .‬‬
‫ה‪Squamous cell carcinoma of esophagus .‬‬

‫‪ .30‬מה נכון לגבי המחלה שרואים בתמונה מס' ‪? 2‬‬


‫א‪ .‬מתאפיינת בנוכחות גרנולומים‬
‫ב‪ .‬מתאפיינת בנוכחות נגעים בצורת פיטריה או "התפרצות הא געש"‬
‫ג‪ .‬מתאפיינת בנוכחות כיבים יחסית שטחיים שאינם עוברים את התת רירית‬
‫ד‪ .‬מתאפיינת בהשטחת הוילי עם ריבוי לימפוציטים בין תאי האפיתל‬
‫ה‪ .‬מתאפיינת בנוכחות כיבים בעלי צוואר צר ובסיס רחב‬

‫‪ .31‬כל הבאים הם סיבה לתסמונת תת ספיגה‪ ,‬פרט ל‪:‬‬


‫א‪ .‬מחלת ‪Crohn‬‬
‫ב‪Chronic pancreatitis .‬‬
‫ג‪Celiac disease .‬‬
‫ד‪Ulcerative colitis .‬‬
‫ה‪Whipple disease .‬‬
‫‪ .32‬הסיבוך הכי שכיח של כיב פפטי כרוני הוא‪:‬‬
‫א‪ .‬דימום‬
‫ב‪ .‬השתנות ממאירה‬
‫ג‪ .‬התנקבות )‪(perforation‬‬
‫ד‪Peritonitis .‬‬
‫ה‪ .‬חסימה‬

‫‪ .33‬באיזה מהבאים קיים סיכון מוגבר ל‪? Small bowel lymphoma-‬‬


‫א‪Whipple disease .‬‬
‫ב‪Celiac disease .‬‬
‫ג‪Crohn's disease .‬‬
‫ד‪Ulcerative colitis .‬‬
‫ה‪Viral enteritis .‬‬
‫‪ .34‬המחלה שבסלייד מס' ‪ 5‬מתאפיינת ב‪:‬‬
‫א‪ .‬פוגעת תמיד ברקטום והכיבים אינם עמוקים משכבת התת רירית‬
‫ב‪ .‬רואים קטעים בריאים בין קטעים חולים ונוכחות גרנולומים בדופן האיבר‬
‫ג‪ .‬הסננה על‪-‬ידי ‪Signet ring cell‬‬
‫ד‪ .‬נגעים דמויי "התפרצות הר געש" או פיטריה‬
‫ה‪ .‬הסננה על‪-‬ידי בלוטות אטיפיות בכל שכבות דופן האיבר‬

‫‪ .35‬הממצא בתמונה מס' ‪ 3‬אופייני ל‪:‬‬


‫א‪Primary biliary cirrhosis .‬‬
‫ב‪Primary sclerosing cholangitis .‬‬
‫ג‪Alcoholic hepatitis .‬‬
‫ד‪Alcoholic cirrhosis .‬‬
‫ה‪Chronic pancreatitis .‬‬
:‫ פרט ל‬Acute hepatitis-‫ כל הבאים אופייניים ל‬.36
Lobar disarray .‫א‬
Mallory bodies .‫ב‬
Acidophil bodies .‫ג‬
Ballooning degeneration .‫ד‬
Hypertrophy of Kuppfer cells .‫ה‬

:‫ פרט ל‬Primary biliary cirrhosis-‫ כל הבאים קשורים ל‬.37


Mallory bodies .‫א‬
Granulomas .‫ב‬
Common bile duct obstruction .‫ג‬
Florid duct lesion .‫ד‬
Cholestasis .‫ה‬
‫ רואים הסננה מונונוקלארית עם רוב של תאי פלזמה‬40 ‫ בביופסית כבד אצל אישה בת‬.38
?‫ מה האבחנה הסבירה ביותר‬.‫המסננים את הרווחים הפורטליים ומתפשטים לתוך האוניות‬
Autoimmune hepatitis .‫א‬
Primary biliary cirrhosis .‫ב‬
Acute viral hepatitis .‫ג‬
Chronic viral hepatitis .‫ד‬
Alcoholic hepatitis .‫ה‬

? ‫ בדם‬Alpha feto protein ‫ באיזה מהבאים אופיינית עליית‬.39


Hepatocellular adenoma .‫א‬
Focal nodular hyperplasia .‫ב‬
Cholangio carcinoma .‫ג‬
Liver metastasis .‫ד‬
Hepatoblastoma .‫ה‬

? ‫ ולהבדיל אותו מכבד תקין‬Hepatocellular adenoma ‫ איזה מהבאים מאפשר לאבחן‬.40


‫ העדר רווחים פורטליים‬.‫א‬
‫ הפרשת מרה‬.‫ב‬
‫ העדר סיבי רטיקולין‬.‫ג‬
Mallory ‫ נוכחות גופיפי‬.‫ד‬
‫ נוכחות שינוי שומני‬.‫ה‬

:‫ הכל נכון פרט ל‬Adenocarcinoma of pancreas ‫ לגבי‬.41


(ducts) ‫ ברב המקרים מתפתח מתאים דוקטליים‬.‫א‬
Pseudocyst ‫ אחד הסיבוכים שלו הינו הופעת‬.‫ב‬
‫ ברב המקרים מופיע בראש הלבלב‬.‫ג‬
Perineural invasion-‫ יש לו נטייה ל‬.‫ד‬
‫ יש לו נטייה לחדור בשלבים יחסית מוקדמים לרקמות בסמוכות ללבלב‬.‫ה‬

? 4 '‫ מה האבחנה האפשרית בתמונה מס‬.42


Chronic pancreatitis .‫א‬
Carcinoma of pancreas .‫ב‬
Islet cell tumor of pancreas .‫ג‬
Acute pancreatitis .‫ד‬
Pseudocyst of pancerase .‫ה‬
‫‪ .43‬כל הבאים יכולים לגרום למחלה שבסלייד מס' ‪ 4‬פרט ל‪:‬‬
‫א‪Chronic viral hepatitis .‬‬
‫ב‪Primary sclerosing cholangitis .‬‬
‫ג‪Wilson's disease .‬‬
‫ד‪Hepatic vein thrombosis .‬‬
‫ה‪Galactosemia .‬‬

‫‪ .44‬המרכיבים הקליניים של דרוג ה‪ Child pugh's-‬כוללים‪:‬‬


‫א‪ .‬בצקת היקפית‪ ,‬כאבי בטן‬
‫ב‪ .‬אנצפלופתיה‪ ,‬מיימת‬
‫ג‪ .‬כאבי בטן‪ ,‬דלדול שרירים‬
‫ד‪ .‬קפוט מדוזה‪ ,‬מיימת‬
‫ה‪ .‬ירידה במסת השריר‪ ,‬צהבת‬

‫‪ .45‬המרכיב החדש שנכנס לדרוג ‪ MELD‬הוא‪:‬‬


‫א‪ .‬קריאטנין‬
‫ב‪ .‬אלפא‪-‬פטו פרוטאין‬
‫ג‪ .‬רמת קלציום בדם‬
‫ד‪ .‬רמת הנתרן בדם‬
‫ה‪ .‬המוגלובין‬
‫‪ .46‬בחולה עם מחלה כולסטטית מתקדמת ונזק לכבד‪ ,‬הבדיקות שיבחנו בצורה הטובה ביותר את‬
‫תפקוד הכבד‪:‬‬
‫א‪ALT / AST .‬‬
‫ב‪ALKP / GGT .‬‬
‫ג‪ .‬בילירובין‪ ,‬אלבומין‪INR ,‬‬
‫ד‪ ,α1At ,ALP .‬רמות נחושת‬

‫‪ .47‬בן ‪ ,20‬רמות ‪ ALT‬ו‪ AST-‬מוגברות פי שנים מהנורמה וביחס ‪ .1:1‬מי מהבדיקות הבאות‬
‫תקדמנה את האבחנה ?‬
‫א‪Anti-HCV .‬‬
‫ב‪HBsAg .‬‬
‫ג‪CPK .‬‬
‫ד‪ .‬רמות נחושת בשתן‬
‫ה‪ .‬כל הבדיקות הנ"ל תקדמנה את האבחנה‬

‫‪ .48‬מי מהמחלות הבאות גורמות לעליה בפוספטזה בסיסית ?‬


‫א‪(PBC) Primary biliary cirrhosis .‬‬
‫ב‪ .‬מחלת כבד גרנולומטוטית‬
‫ג‪ .‬העלמות אידיופתית של צינורות המרה‬
‫ד‪(PSC) Primary sclerosing cholangitis .‬‬
‫ה‪ .‬כל התשובות נכונות‬

‫‪ .49‬תרופת הבחירה לטיפול בחולה עם שחמת כבד‪ HBV ,‬ומיימת )דקומפנסציה( תהיה‪:‬‬
‫א‪Interpheron .‬‬
‫ב‪ .‬פג‪-‬אינטרפרון‬
‫ג‪ .‬טיפול תומך בלבד‬
‫ד‪Nucleoside analogue .‬‬
‫ה‪ .‬כל התשובות נכונות‬
‫‪ .50‬מבין הצירופים הבאים איזה צרוף סרולוגי מתבטא החלמה מ‪? HBV-‬‬
‫א‪IgG anti HBc, IgG anti HBs, IgG anti HBe .‬‬
‫ב‪IgG anti HBe, IgG anti HBc, HBsAg .‬‬
‫ג‪IgG anti HBc, IgG anti HBe, HBV-DNA 3000cpm/ml .‬‬
‫ד‪(+) IgG anti HBs, (+) IgG anti HBc, (-) IgM anti HBc .‬‬

‫‪ .51‬מה נכון לגבי זיהום בנגיף הפטיטיס ‪? C‬‬


‫א‪ .‬שיעור התגובה לטיפול אנטי ויראלי בגנוטיפ ‪ 1‬הוא ‪20%‬‬
‫ב‪ .‬ברב החולים עם הפטיטיס ‪ C‬מחלת הכבד מתקדמת עם מעבר לצירוזיס‬
‫ג‪ .‬העומס הויראלי אינו גורם בעל חשיבות בקביעת התגובה לטיפול‬
‫ד‪ .‬השמנה ויסצרלית מהווה גורם המשפיע על קצב התקדמות לפיברוזיס ו‪/‬או צירוזיס‬

‫‪ .52‬בחולה אסימפטומטית בת ‪ 37‬נמצאו הערכים הבאים‪:‬‬


‫‪ ,AST-230 ,ALT-280‬גלובולין ‪ 4.8‬גר‪ ,%‬אלבומין ‪ 3.8‬גר‪ ,%‬בילירובין ו‪ PT-‬תקינים‪.‬‬
‫כמו כן נמצאו נוגדנים חיוביים כנגד מרכיבי הגרעין )‪ .(ANA‬כל האבחנות הבאות אפשריות‬
‫פרט ל‪:‬‬
‫א‪ .‬הפטיטיס אוטואימונית‬
‫ב‪non alcoholic steato-hepatitis – NASH .‬‬
‫ג‪ .‬המוכרומטוזיס‬
‫ד‪ .‬הפטיטיס ‪ C‬כרוני‬
‫ה‪ .‬הפטיטיס מתרופות )‪(drug induced hepatitis‬‬

‫‪ .53‬אישה מכורה לאלכוהול נטלה ‪ 8‬טבליות אקמול )סה"כ ‪ 4‬גר'( והגיעה לחדר מיון‪ .‬בבדיקה‬
‫ראשונית לאחר ‪ 6‬שעות נמצאו הממצאים הבאים‪ :‬אנזימי כבד תקינים‪ ,‬תפקוד סינטטי תקין‬
‫)‪ PT‬ואלבומין(‪ ,‬רמת פאראצטמול )אקמול( בדם גבוהה‪ .‬איזו מהגישות הבאות תהיה‬
‫המתאימה ביותר למצבה?‬
‫א‪ .‬הסיכוי לנזק כבדי קטן ולכן אין צורך להמשך אשפוז‬
‫ב‪ .‬יש להתחיל ‪ ,N-acetyl cystein‬ולעקוב אחרי רמות אקמול ואנזימי כבד‬
‫ג‪ .‬יש לבצע שטיפת קיבה ומעקב אמבולטורי של אנזימי כבד‬
‫ד‪ .‬במקרה זה מנון האקמול אינו הפטו‪-‬טוקסי אבל יש צורך במעקב תכוף של אנזימי כבד‬

‫שאלות ‪ 54-56‬מתייחסות לקטע הבא‪:‬‬


‫התאם את תיאור המקרה לאבחנה המתאימה ביותר אצל חולה השותה אלכוהול בכמות גדולה‪.‬‬
‫)ערכים תקינים ראה בסוף השאלה(‪.‬‬

‫‪Alcoholic cirrhosis‬‬ ‫א‪.‬‬


‫‪Alcoholic fatty liver‬‬ ‫ב‪.‬‬
‫‪Acute alcoholic hepatitis‬‬ ‫ג‪.‬‬
‫‪Alcoholic gastritis‬‬ ‫ד‪.‬‬

‫ערכי מעבדה ‪ -‬טווח תקין‪:‬‬


‫‪WBC:‬‬ ‫‪5000-10,000/ml‬‬
‫‪AST:‬‬ ‫‪10-40 U/L‬‬
‫‪ALT:‬‬ ‫‪10-40 U/L‬‬
‫‪Bilirubin: 0.2-1.0 mg%‬‬
‫‪PT:‬‬ ‫)‪75%-100% (INR 0.9-1.1‬‬
‫‪Albumin: 3.8-4.5 gr%‬‬
‫‪ .54‬חום ‪ 38‬מעלות‪ ,‬לויקוציטוזיס )‪ , 120-ALT , 270-AST ,(WBC:18,000/ml‬בילירובין ‪12 -‬‬
‫מ"ג‪ ,( 2-INR ) 36% - PT ,%‬אלבומין ‪ 2.8‬גר'‪.%‬‬

‫‪ .55‬מיימת‪ ,‬אנצפליטיס‪ ,‬לויקופניה )‪ , 120-ALT , 270-AST ,(WBC:2,200/ml‬בילירובין ‪8 -‬‬


‫מ"ג‪ ,38% - PT ,%‬אלבומין ‪ 2.8‬גר'‪.%‬‬

‫‪ .56‬ספירת דם תקינה‪ , 120-ALT , 270-AST ,‬בילירובין ‪ 3 -‬מ"ג‪ ,100% - PT ,%‬אלבומין ‪3.9‬‬


‫גר'‪.%‬‬

‫פרק מספר ‪ :2‬נפרולוגיה‬


‫‪ .57‬לגבי סלייד מס' ‪ :3‬מהם הממצאים במיקרוסקופ אלקטרוני במחלה זו ?‬
‫א‪ .‬משקעים סובאנדותליאליים גדולים‬
‫ב‪ .‬משקעים סובאפיתליאליים קטנים עם גדילה של ‪ GBM‬ביניהם‬
‫ג‪ .‬משקעים סובאפיתליאליים גדולים ללא גדילה של ה‪ GBM-‬ביניהם‬
‫ד‪ .‬משקעים קטנים במזנגיום רק בחלק מהמקרים‬
‫ה‪ .‬אין משקעים‬

‫‪ .58‬לגבי סלייד מס' ‪ :3‬מהי הקליניקה של החולה ?‬


‫א‪Glomerulonephritis and acute renal failure .‬‬
‫ב‪Glomerulonephritis and chronic renal failure .‬‬
‫ג‪Nephrotic syndrome and acute renal failure .‬‬
‫ד‪Nephrotic syndrome and chronic renal failure .‬‬
‫ה‪Malignant hypertension .‬‬

‫‪ .59‬תמונה מס' ‪ – 5‬שלושה סוגי אלו של צביעת ‪ IgG‬באימונופלורסנציה יכולים להופיע ב‪:‬‬
‫א‪SLE .‬‬
‫ב‪Cresentic glomerulonephritis .‬‬
‫ג‪Post-infectious glomerulonephritis .‬‬
‫ד‪IgA nephropathy .‬‬
‫ה‪Hepatitis B nephropathy .‬‬

‫‪ .60‬תמונה מס' ‪ – (EM) 6‬מהי התמונה הקלינית של החולה ?‬


‫א‪RBS (Erythrocyte) casts .‬‬
‫ב‪Rapidly progressive glomerulonephritis .‬‬
‫ג‪Recurrent hematuria .‬‬
‫ד‪Nephrotic syndrome .‬‬
‫ה‪Diabetes .‬‬

‫‪.61‬מהי התמונה הקלינית האופיינית המתאימה לתמונה מס' ‪? 7A‬‬


‫א‪ .‬ילד עם גלילים המטיים‪ ,‬יתר לחץ דם‪ ,‬הפרעה בתפקוד הכלייתי‬
‫ב‪ .‬ילד עם בצקות‪ ,‬היפואלבומינמיה‪ ,‬הרבה חלבון בשתן‬
‫ג‪ .‬נער עם המטוריה חוזרת )ללא גלילים( בעקבות זיהומים‬
‫ד‪ .‬מבוגר עם התדרדרות מהירה בתפקוד הכלייתי‬
‫ה‪ .‬מבוגר עם יתר לחץ דם ואי ספיקת כליות‬
‫‪ .62‬תמונה מס' ‪ 7B‬ו‪) 7C-‬ימנית עליונה ‪ -‬מיקרוסקופ אור‪ ,‬שמאלית תחתונה ‪ -‬מיקרוסקופ‬
‫אלקטרוני( הן מאותו חולה‪ .‬מה אופייני למחלה בבדיקת דם ?‬
‫א‪ ANCA .‬גבוה‬
‫ב‪ Anti-DNA .‬גבוה‬
‫ג‪ Anti-streptolysin-O .‬גבוה‬
‫ד‪ Anti-GBM .‬גבוה‬
‫ה‪ IgA .‬גבוה‬

‫‪ .63‬תמונה מס' ‪ 8‬מראה תאי ענק סביב ל‪ Cast-‬בטובולי ואטרופיה טובולרית‪ .‬התמונה אופיינית‬
‫ל‪:‬‬
‫א‪Diabetes .‬‬
‫ב‪Scleroderma .‬‬
‫ג‪Myeloma .‬‬
‫ד‪Analgesic nephropathy .‬‬
‫ה‪SLE .‬‬

‫‪ .64‬כל הבאים נכונים לגבי ‪ Renal cell carcinoma‬פרט ל‪:‬‬


‫א‪ .‬התאים בהירים בגלל הצטברות של מיטוכונדריה‬
‫ב‪ .‬נוטה לחדור לורידים‬
‫ג‪ .‬התפשטות לשומן הפרינפרי מעלה את ה‪stage-‬‬
‫ד‪ .‬קשורה עם מוטציה ב‪Von-Hippel-Lindau gene-‬‬
‫ה‪ .‬יכולה להראות גוון צהבהב במאקרוסקופיה‬

‫‪ Hyperplastic Arthritis (onion skinning) .65‬אופייני ל‪:‬‬


‫א‪ .‬סוכרת‬
‫ב‪SLE .‬‬
‫ג‪RPGN .‬‬
‫ד‪Anaplastic nephropathy .‬‬
‫ה‪ .‬אף אחד מהנ"ל‬

‫‪ .66‬חולה בן ‪ 50‬מגיע עם ‪ .Chronic renal failure‬בבדיקתו ‪ -‬כליות גדולות מאוד עם ציסטות‬


‫מרובות בגדלים שונים‪ .‬מה נכון ?‬
‫א‪ .‬המחלה ‪Autosomal recessive‬‬
‫ב‪ .‬המחלה מלווה לעיתים ב‪congenital hepatic fibrosis-‬‬
‫ג‪ .‬המחלה יכולה להיות גם אונילטרלית‬
‫ד‪ .‬המחלה קשורה למוטציה בגן בכרומוזום ‪16‬‬
‫ה‪ .‬המחלה מלווה לעיתים קרובות בתסמונת נפרוטית‬

‫‪ .67‬חלבון ‪ Nephrin‬הינו‪:‬‬
‫א‪ .‬מרכיב של הממברנה הבזאלית בגלומרולי‬
‫ב‪ .‬מרכיב של ה‪slit diaphragm-‬‬
‫ג‪ .‬קושר את רגלית הפודוציט לממברנה הבזאלית‬
‫ד‪ .‬מרכיב של המטריקס המזנגיאלי‬

‫‪ .68‬כל הבאים נכונים לגבי עמילוידוזיס של הכליה‪ ,‬פרט ל‪:‬‬


‫א‪ .‬ניתן לראות סיבים אופייניים במיקרוסקופ אלקטרוני‬
‫ב‪ .‬ניתן להוכיח שקיעתו בצביעת קונגו‪-‬רד‬
‫ג‪ .‬ניתן לראותו בצביעה אימונופלורסצנטית לאימונוגלובולינים )‪(IgG, IgM, IgA‬‬
‫‪ .69‬סליד מס' ‪ - 2‬מי מהבאים קשור לאבחנה שבסלייד ?‬
‫א‪Hepatitis B .‬‬
‫ב‪ANCA .‬‬
‫ג‪Anti-GBM .‬‬
‫ד‪Polymyalgia rheumatica .‬‬
‫ה‪ C3 .‬נמוך‬

‫‪ ANCA .70‬גבוה בדם מופיע בכל הבאים‪ ,‬פרט ל‪:‬‬


‫א‪Microscopic polyangiitis .‬‬
‫ב‪Idiopathic cresentic glomerulonephritis .‬‬
‫ג‪Churg-Strauss .‬‬
‫ד‪Goodpasture syndrome .‬‬
‫ה‪Wegener's granulamatosis .‬‬

‫‪ Palpable purpura .71‬הוא ממצא אופייני ל‪:‬‬


‫א‪Acute leukocytoclastic vasculitis .‬‬
‫ב‪Giant cell arthritis .‬‬
‫ג‪Polyarteritis nodosa .‬‬
‫ד‪Takayasu arthritis .‬‬
‫ה‪Kawasaki disease .‬‬

‫‪ .72‬בן ‪ ,70‬נבדק בחדר מיון עקב חום גבוה‪ ,‬שיעול וחולשה כללית‪ .‬בדיקות המעבדה הראו את‬
‫התוצאות הבאות‪:‬‬
‫‪Glucose=90mg/dl‬‬
‫‪BUN=56mg/dl‬‬
‫‪Sodium=150meq/l‬‬
‫מה תהיה האוסמולליות בפלסמה של החולה הנ"ל ?‬
‫א‪325 .‬‬
‫ב‪280 .‬‬
‫ג‪180 .‬‬
‫ד‪275 .‬‬

‫‪ .73‬בן ‪ ,30‬סובל מזה ‪ 25‬שנה מסכרת מסוג ‪ ,(type-I DM) 1‬ומטופל באינסולין‪ .‬כעת הופיעו‬
‫בצקות‪ ,‬רטינופתיה פרוליפורטיבית עם ירידה בראיה‪ ,‬ניורופתיה פריפרית‪ .‬ערכי לחץ דם‬
‫‪ ,140/90‬קריאטינין בסרום ‪ .2.5mg/dl‬מה לדעתך כמות החלבון בשתן שאתה מצפה למצוא‬
‫באיסוף שתן ‪ 24‬שעות ?‬
‫א‪Protein of 6500mg/24 hr .‬‬
‫ב‪Protein of 800mg/24 hr .‬‬
‫ג‪Protein of 70mg/24 hr .‬‬
‫ד‪Microalbumin of 120mg/day .‬‬

‫‪ .74‬איזה מהמשפטים הבאים מתאים ל‪? IgA nephropathy-‬‬


‫א‪ .‬מחלה גנטית העוברת בתורשה אוטוזומלית דומיננטית‬
‫ב‪ 50% .‬מהמקרים מתפתחים לאי‪-‬ספיקת כליות סופנית תוך עשר שנים‬
‫ג‪ .‬שכיחותה באוכלוסיה הינה ‪ ,25%‬אך היא לא מאובחנת מספיק מהר‬
‫ד‪ .‬לא חוזרת בכליה המושתלת‪ ,‬כמו שאר המחלות התורשתיות‬
‫ה‪ .‬מתבטאת בהמטוריה מיקרוסקופית ומאובחנת ע"י ביופסית כליה‬
‫‪ .75‬בן ‪ ,70‬הובא לחדר מיון בגלל כאבי בטן ושלשולים מימיים מזה מספר שבועות‪ .‬מעברו ידוע על‬
‫אוטם בשריר הלב בדופן הסרעפתי מלפני ‪ 10‬שנים‪ .‬בבדיקה פיזיקלית‪ :‬משקל ‪ 70‬ק"ג‪ ,‬לחץ‬
‫דם ‪ ,100/65‬דופק ‪ 108‬לדקה ויובש בריריות‪ .‬בדיקות המעבדה מראות‪ :‬גלוקוז ‪ 90‬מ"ג‪/‬ד"ל‪,‬‬
‫‪ 25 - BUN‬מ"ג‪/‬ד"ל‪ ,‬קריאטינין ‪ 1‬מ"ג‪/‬ד"ל‪ ,‬אלבומין ‪ 3.4‬גר'‪/‬ד"ל‪ ,‬גלובולין ‪ 3‬גר'‪/‬ד"ל‪,‬‬
‫כולסטרול ‪ 190‬מ"ג‪/‬ד"ל‪ ,‬טריגליצרידים ‪ 200‬מ"ג‪/‬ד"ל‪ ,‬נתרן ‪ 121‬מא"ק‪/‬ל‪ ,‬אשלגן ‪ 4‬מא"ק‪/‬ל‪.‬‬
‫מה נכון לגבי החולה הנ"ל ?‬
‫א‪ .‬ה‪ circulatory effective volume-‬נמוך ולכן הראבסורבציה הפרוקסימלית נמוכה‬
‫ב‪ .‬החולה זקוק להגבלת מים בכדי להעלות את רמת הנתרן הנמוכה בפלסמה‬
‫ג‪ .‬החולה זקוק לטיפול בתמיסת ‪Saline 3%‬‬
‫ד‪ .‬מתן עירוי ע"י ‪ Saline 0.9%‬יגרום לתיקון ההיפונתרמיה‬

‫‪ .76‬טיפול מומלץ מיידי בחולה המתקבל עם היפרקלצמיה סימפטומטית‪ ,‬חוסר תאבון‬


‫ופוליאוריה‪:‬‬
‫א‪2L Ringer Lactate .‬‬
‫ב‪ .‬מתן משתנים מסוג ‪ Furosemide‬בכדי להוריד ספיגת סידן בתאי הלולאה העולה ע"ש‬
‫"הנלי"‬
‫ג‪ .‬הידרציה ע"י העמסת נוזלים מסוג ‪(0.9% NaCl) Saline‬‬
‫ד‪ .‬העמסת כל סוג של נוזלים בשילוב עם ‪Furosemide‬‬
‫ה‪ .‬מתן ‪ Alendronate‬המעכב את פעילות ה‪ Osteoclasts-‬בעצם‬

‫‪ .77‬בן ‪ ,23‬סטודנט למתמטיקה‪ ,‬ביקש יעוץ שני )‪ (second opinion‬עקב מיקרואלבומינוריה ולחץ‬
‫דם תקין‪ .‬ידוע כסובל מסכרת סוג ‪ (type I) 1‬במשך ‪ 9‬שנים‪ .‬מטופל באינסולין‪ ,‬דיאטה‬
‫והליכה‪ .‬אין פגיעה באיברי מטרה אחרים‪ .‬ל"ד בבית ‪ .130/85‬בבדיקתו במשרד היועץ‪ :‬משקל‪-‬‬
‫‪ 62‬ק"ג‪ ,‬ל"ד‪ ,HbA1c-5.8% ,128/82-‬תפקודי כליה תקינים‪ ,‬אלקטרוליטים תקינים‪ ,‬שתן‬
‫לכללית ללא ממצא‪ .‬רופא המשפחה המליץ להתחיל טיפול אנטי‪-‬היפרטנסיבי‬
‫למיקרואלבומינוריה שנתגלתה לאחרונה‪ .‬החולה מודאג‪ .‬מה מבין ההמלצות של היועץ‬
‫הנפרולוגי היא נכונה ?‬
‫א‪ .‬התקדמות לכיוון ‪ Overt nephropathy‬קורה ביותר מ‪ 70%-‬מהחולים עם ‪type I DM‬‬
‫ומיקרואלבומינוריה‬
‫ב‪ .‬טיפול ב‪ Angiotensin converting enzyme inhibitor (ACEI)-‬יכול להוריד‬
‫מיקרואלבומינוריה ומפחית סיבוכים להתקדמות המחלה ל‪ overt nephropathy-‬עם‬
‫חולים בסכרת סוג ‪ 1‬ומיקרואלבומינוריה‬
‫ג‪ .‬מתן ‪ ACEI‬יכול להחמיר את הפרוטאינוריה ומומלץ לתת משתנים מסוג ‪Dizothiazide‬‬
‫בלבד‪.‬‬
‫ד‪ .‬אין קשר בין רמת סוכר ו‪ HbA1c-‬והופעת מיקרואלבומינוריה‬

‫‪ .78‬איזה מהבאים נמצא בסיכון לפתח ‪? Contrast Nephropathy‬‬


‫א‪ .‬חולה עם ‪ ,Chronic Kidney Disease – stage V‬תחת טיפול בהמודיאליזה‬
‫ב‪ .‬חולה עם ‪Malignant melanoma‬‬
‫ג‪ .‬בן ‪ ,60‬שבועיים לאחר ‪ Acute Extensive Anterior Wall M.I‬ועם אורטופניה ו‪PND-‬‬
‫ד‪ .‬בן ‪ 45‬עם יתר לחץ דם תחת טיפול ע"י מינון גבוה של ‪B-Blockers‬‬
‫ה‪ .‬בן ‪ ,21‬מרים משקולות‪ ,‬עם קריאטינין ‪1.2mg%‬‬
‫‪ .79‬בת ‪ ,34‬הופנתה לחדר מיון עקב נפיחות ברגליים‪ .‬לאחרונה אובחנה כסובלת מ‪ .HIV-‬בבדיקה‪:‬‬
‫ללא חום‪ ,‬נשימות ‪ 14‬לדקה‪ ,‬ל"ד ‪ ,126/78‬בצקת גומתית ‪ ,++‬שאר הבדיקה הפיזיקלית תקינה‪.‬‬
‫במעבדה‪:‬‬
‫‪Hb=12gr/dl‬‬
‫‪Plasma creatinine=1.8mg/dl‬‬
‫‪Serum total complement=normal‬‬
‫‪Antisterptolysin-O=0‬‬
‫בשתן לכללית‪ :‬חלבון‪ ,+‬אריתרוציטים רבים‪ ,‬גלילים היאליניים‪ .‬באיסוף שתן ‪ 24‬שעות לחלבון‬
‫כמותי=‪.16gr‬‬
‫באיזו מהבדיקות היית מתחיל את הבירור לקראת אבחנה‪:‬‬
‫א‪ .‬בדיקת על קולית של הכליות )‪(US‬‬
‫ב‪Intravenous Pyelography (IVP) .‬‬
‫ג‪ .‬אנגיוגרפיה כליתית‬
‫ד‪ .‬אקו‪-‬קרדיאוגרפיה‬
‫ה‪Retrograde Pyelopathy .‬‬
‫‪.80‬ציין את המשפט הנכון‪:‬‬
‫א‪ .‬מידת ההפרשה של זרחן בשתן עוזרת בד"כ באבחנה מבדלת של היפופוספטמיה‬
‫ב‪ Calcium sensing receptor .‬הינו קולטן ספציפי המופעל אך ורק ע"י ריכוז סין מיונן‬
‫גבוה בפלזמה‬
‫ג‪ .‬שימוש בחוקנים להכנה לבדיקת קולונוסקופיה עלול להסתבך בהיפונתרמיה‪,‬‬
‫היפוקלצמיה והיפופוספטמיה חריפה‬
‫ד‪ .‬השימוש הבלעדי במשתנים הינו לצורך טיפול במחלות של אגירת נוזלים במדור החוץ‪-‬‬
‫תאי בגוף‬
‫ה‪ .‬בהיפוקלצמיה חריפה מרווח ‪ QT‬מתקצר בתרשים א‪.‬ק‪.‬ג‬

‫‪ .81‬קונטרה‪-‬אינדיקציה להשתלת כליה‪:‬‬


‫א‪ .‬היסטוריה של סכרת‬
‫ב‪ .‬החולה מעל גיל ‪60‬‬
‫ג‪ .‬הפטיטיס ‪ C‬אנטיגנמיה‬
‫ד‪ .‬טיפול למחלה ממאירה בשנה האחרונה‬
‫‪ .82‬כל הסימנים הבאים נגרמים ע"י היפרקלמיה‪ ,‬פרט ל‪:‬‬
‫א‪ .‬השטחה גל גלי ‪T‬‬
‫ב‪ .‬הרחבה של קומפלקס ‪QRS‬‬
‫ג‪ .‬השטחה של גלי ‪P‬‬
‫ד‪ .‬קצב לב איטי‬
‫ה‪ PR interval .‬מאורך‬

‫‪ .83‬בן ‪ ,65‬נבדק בחדר המיון עקב התדרדרות כללית במצבו ובלבול‪ .‬לציין שהחולה סובל מגידול‬
‫בלבלב‪ .‬בבדיקה פיזיקלית‪ :‬משקל ‪ 70‬ק"ג‪ ,‬לחץ דם ‪ ,120/80‬דופק ‪ 80‬לדקה‪ ,‬ללא יובש‬
‫בריריות‪ ,‬ריאות נקיות‪ ,‬בטן וגפיים ללא ממצא פתולוגי‪ .‬בדיקות מעבדה‪ :‬גלוקוז ‪ 77‬מ"ג‪/‬ד"ל‪,‬‬
‫‪ 11 - BUN‬מ"ג‪/‬ד"ל‪ ,‬קריאטינין ‪ 0.8‬מ"ג‪/‬ד"ל‪ ,‬אלבומין ‪ 4.8‬גר'‪/‬ד"ל‪ ,‬כולסטרול ‪ 256‬מ"ג‪/‬ד"ל‪,‬‬
‫טריגליצרידים ‪ 249‬מ"ג‪/‬ד"ל‪ ,‬נתרן ‪ 110‬מא"ק‪/‬ל‪ ,‬אשלגן ‪ 4‬מא"ק‪/‬ל‪.‬‬
‫החולה התאשפז במחלקה הפנימית לטופל ע"י ‪ Saline 3%‬למשך ‪ 10‬שעות בקצב של ‪ 80‬סמ"ק‬
‫לשעה‪ .‬מצב החולה לא השתפר ואף הורע‪ .‬מה לדעתך הסיבה להרעה במצב החולה ?‬
‫א‪ .‬הטיפול בחולה היה איטי‬
‫ב‪ .‬הטיפול בחולה היה מהיר מידי‬
‫ג‪ .‬אין קשר בין ההרעה במצב החולה והטיפול שניתן‬
‫ד‪ .‬הטיפול בחולה צריך להיות הגבלת מים בלבד‬
‫‪ .84‬בן ‪ 60‬עם יתר לחץ דם גבוה ביותר ולא מאוזן‪ ,‬אסימפטומטי עם קריאטינין ‪ ,4.8‬הפרשת חלבון‬
‫בשתן היא ‪ 4.2‬גרם ליום‪ ,‬בדיקות סרולוגיות שליליות‪ ,‬בדיקת ‪ US‬מראה כליות בגודל ‪ 6.8‬ס"מ‬
‫סימטריות‪ .‬התערבות האבחנתית או הטיפולית המתאימה ביותר כוללת‪:‬‬
‫א‪ .‬ביופסית כליה מיידית‬
‫ב‪ .‬דיאטה מוגבלת ל‪ 40-‬גרם חלבון ליום‬
‫ג‪ .‬טיפול ביתר לחץ‪-‬דם ליעד של לחץ דם בסביבות ‪) 125/75‬והכנה לטיפול בדיאליזה או‬
‫השתלה בעתיד(‬
‫ד‪ .‬בדיקה אנגיוגרפית של עורקי הכליות עם הזרקת חומר ניגוד‬
‫‪ .85‬אילו מהתוצאות הבאות רומזות על מחלה כליתית כרונית כסיבה לקריאטינין גבוה‪ ,‬במטופל‬
‫שמגיע למרפאה לבדיקה תקופתית ?‬
‫א‪ .‬כליות דו צדדיות סימטריות באורך ‪ 8‬ס"מ‬
‫ב‪ .‬בבדיקת דם לפני חצי שנה ערכים נורמאליים של קריאטינין‬
‫ג‪ CK .‬של ‪40,000‬‬
‫ד‪ .‬ערכי זרחן ‪ 3.6‬וסידן ‪ 9.8‬בפלסמה‬
‫ה‪ .‬המוגלובין נורמאלי‬

‫‪ .86‬המשתן הידרוכלורתיאזיד גורם בד"כ ל‪:‬‬


‫א‪ .‬דיכוי הציר רנין‪-‬אנגיוטנסין‬
‫ב‪ .‬היפואלדוסטרוניזם‬
‫ג‪ .‬עיכוב הנשא נתרן כלוריד בצד הלומינלי של ה‪Distal convoluted tubules-‬‬
‫ד‪ .‬עיכוב הנשא נתרן כלוריד בצד הבזולטרלי של ה‪Distal convoluted tubules-‬‬
‫ה‪ .‬עיכוב הנשא נתרן כלוריד בצד הלומינלי של ה‪Collecting ducts-‬‬

‫‪ .87‬בן ‪ 63‬הסבול מזה שנים רבות מ‪ ,COPD-‬פיתח ‪ Cor pulmonale‬עם חמרה של הבצקות‬
‫הפריפריות‪ .‬כתוצאה מכך טופל ע"י הגבלת מלח בדיאטה‪ ,‬דיגוקסין ו‪ .Loop Diuretics-‬מס'‬
‫ימים לאחר הטיפול אושפז בבי"ח עקב חולשה כללית וירידה במצב ההכרה‪ .‬בקבלתו לחץ דם‬
‫נמוך‪ ,‬לא היו בצקות ברגליים‪ ,‬והיו הפרעות בקצב הלב‪ .‬הבדיקות הראשונות שבוצעו הראו‪:‬‬
‫‪,Urine Na=2 ,K=2.5,Cl=74 ,PO2=50 ,PCO2=70 ,PH=7.43 ,HCO3=46 ,Na=140‬‬
‫‪.Urine Cl=1‬‬
‫כל האבחנות הבאות אפשריות פרט ל‪:‬‬
‫א‪ .‬חמצת מטבולית‬
‫ב‪ .‬חמצת נשימתית‬
‫ג‪ .‬בססת מטבולית‬
‫ד‪ .‬בססת נשימתית‬

‫‪ .88‬הערך הנמוך מאוד של כלור בשתן בחולה הנ"ל )שאלה קודמת( מעיד ל כך ש‪:‬‬
‫א‪ .‬הוא כעת איננו תחת השפעת ‪Loop Diuretics‬‬
‫ב‪ .‬מתאים עם מצב של בססת מטבולית‬
‫ג‪ .‬מתאים עם מצב של ירידה בנפח נוזלי הגוף‬
‫ד‪ .‬כל התשובות )א‪+‬ב‪+‬ג( נכונות‬
‫ה‪ .‬אף תשובה אינה נכונה‬

‫‪ .89‬הטיפול המתאים לחולה הנ"ל כולל‪:‬‬


‫א‪ .‬מתן נוזלים עם תוספת ‪ KCl‬והפסקת דיגוקסין‬
‫ב‪ .‬משך מתן פוסיד‬
‫ג‪ .‬מתן חמצן בזרימה גבוהה‬
‫ד‪ .‬אינטובציה מיידית והנשמה מלאכותית‬
‫ה‪ .‬המודיאליזה מיידית‬
‫‪ .90‬בן ‪ 70‬שקול ‪ 70‬ק"ג‪ .‬הובא לחדר מיון בגלל שיעול מזה מספר חודשים וירידה במשקל של ‪5‬‬
‫ק"ג בחודשיים האחרונים‪ .‬מעברו עישון כבד ומחלה וסקולרית פריפרית‪.‬‬
‫בבדיקה פיזיקלית‪ :‬משקל ‪ 70‬ק"ג‪ ,‬לחץ דם ‪ ,125/85‬דופק ‪ 84‬לדקה‪ ,‬ללא יובש בריריות‪,‬‬
‫ריאות‪-‬כניסת אוויר מופחתת לריאה ימנית‪ ,‬בטן וגפיים ללא ממצא פתולוגי‪.‬‬
‫בבדיקות המעבדה מראות‪ :‬גלוקוז ‪ 94‬מ"ג‪/‬ד"ל‪ BUN-12 ,‬מ"ג‪/‬ד"ל‪ ,‬קריאטינין ‪ 0.6‬מ"ג‪/‬ד"ל‪,‬‬
‫אלבומין ‪ 4.5‬גר'‪/‬ד"ל‪ ,‬גלובולין ‪ 3.5‬גר'‪/‬ד"ל‪ ,‬כולסטרול ‪ 242‬מ"ג‪/‬ד"ל‪ ,‬טריגליצרידים ‪231‬‬
‫מ"ג‪/‬ד"ל‪ ,‬נתרן ‪ 126‬מא"ק‪/‬ל‪ ,‬אשלגן ‪ 4‬מא"ק‪/‬ל‪.‬‬
‫מה נכון לגבי החולה הנ"ל‪:‬‬
‫א‪ .‬האוסמולליות של השתן תהיה נמוכה יחסית לאוסמולליות בפלסמה‬
‫ב‪ .‬רמת ה‪ ADH-‬בפלסמה גבוהה כתוצאה מהפרשה מוגברת בגרעיני ההיפותלמוס‬
‫ג‪ .‬החולה זקוק לטיפול בתמיסה היפרטונית )‪(Saline 3%‬‬
‫ד‪ .‬האבחנה המבדלת תכלול אפשרות של קרצינומה של הריאות‬

‫‪ .91‬מאלו תרופות יש צורך להסס ולהיזהר בטיפול מחשש להיפרקלמיה בחולים עם כשל כליתי ?‬
‫א‪ .‬חסמי בטא )‪ (Beta blockers‬לא סלקטיביים‬
‫ב‪ .‬מעכבי ‪ ACE‬וחוסמי קולטני אנגיוטנסין )‪(ARB‬‬
‫ג‪ .‬תרופות ממשפחת האנטי‪-‬דלקתיות הלא סטרואידיות )‪(NSAIDS‬‬
‫ד‪ .‬נוגדי פעולת אלדוסטרון‪ ,‬ספירונולקטון )‪(Spironolactone‬‬
‫ה‪ .‬כל התשובות נכונות‬

‫‪ .92‬ריכוז זרחן נמוך בנסיוב יכול להיגרם מ‪:‬‬


‫א‪ .‬רמה גבוהה של ‪ 1,25 (OH)2 Vitamin D3‬בנסיוב‬
‫ב‪ .‬תנגודת לפעילות של ויטמין ‪ D‬ברקמות‬
‫ג‪ .‬הפרשה נמוכה של ‪Parathyroid hormone‬‬
‫ד‪ .‬מוטציה המעכבת פעילות ‪FGF 6‬‬
‫ה‪ .‬חמצת מטבולית‬

‫‪ .93‬בת ‪ 68‬סובלת מגלומרולונפריטיס כרונית‪ ,‬תפקוד כלייתי מוערך כ‪ GFR~19ml/min-‬ונמצאת‬


‫במעקב של מרפאה פרדיאליטית‪ .‬הגיעה עם תלונות של גרד וכאבים קלים בשוקיים‪ .‬בבדיקת‬
‫דם נמצאה רמת ‪) PTH-233‬גבוהה(‪ ,‬סידן של ‪ 8mg/dl‬וזרחן של ‪ .7mg/dl‬איזה מהמשפטים‬
‫הבאים מתאר את ההפרעה המטבולית הנ"ל‪:‬‬
‫א‪Primary Hyperparathyroidism .‬‬
‫ב‪ .‬לתפקוד תקין של מערכות הגוף בחולה‪ ,‬חשובה מאוד רמת הסידן הכללית שנמדדה ואין‬
‫להתייחס לרמת הזרחן החופשית‪.‬‬
‫ג‪ .‬רב הסידן‪-‬זרחן הגוף נמצא במדור התוך תאי ועיקר בשריר הסקלטלי ולכן היא סובלת‬
‫מכאבים ברגליים‬
‫ד‪ .‬מתואר ‪ Secondary Hyperparathyroidism‬קל ואינו מסביר את התלונות‪ ,‬ויש צורך‬
‫לחפש נוכחות ‪PTHrP‬‬
‫ה‪ Hyperparathyroidism .‬משני לאי ספיקת כליות כרונית הגורמת ליצור מופחת של‬
‫ויטמין ‪ D‬פעיל בכליה‪ ,‬היפרפוספטמיה והיפוקלצמיה‬

‫‪ .94‬המקור העיקרי לאיבוד האשלגן בעקבות הקאות מרובות או שימוש ממושך בזונדה‪:‬‬
‫א‪ .‬בקיא עצמו‬
‫ב‪ .‬במעבר של האשלגן למדור התאי‬
‫ג‪ .‬בשתן‬
‫ד‪ .‬בזיעה‬
‫ה‪ .‬אין איבוד אשלגן ‪ -‬האיבוד העיקרי הוא של יוני מימן‬

‫‪ .95‬סיבה אפשרית ל‪ Rapidly progressive glomerulonephritis-‬יכול להיות‪:‬‬


‫א‪Post-streptococcal GN .‬‬
‫ב‪IgA nephropathy .‬‬
‫ג‪ANCA – vasculitis .‬‬
‫ד‪Goodpasteur's syndrome .‬‬
‫ה‪ .‬כל התשובות נכונות‬

‫‪ .96‬גורלם של רב החולים עם אי ספיקת כליות כרונית‪:‬‬


‫א‪ .‬התאוששות ספונטנית עד לתפקוד כליה תקין‬
‫ב‪ .‬התדרדרות עד לדיאליזה או השתלה‬
‫ג‪ .‬מוות ממחלה קרדיווסקולרית עוד לפני שמגיעים לאי‪-‬ספיקת כליות סופנית‬
‫ד‪ .‬אף תשובה אינה נכונה‬

‫פרק מספר ‪ :3‬ראומטולוגיה‬


‫‪ .97‬חולה בן ‪ ,27‬מתלונן על כאבי פרקים וקשיון בוקר ממושך מזה ‪ 3‬חודשים‪ .‬בבדיקה‪ :‬נפיחות‬
‫של ברך ימין ונפיחות של המפרקים האינראלפנגיאליים המקורבים ‪ +‬המפרקים‬
‫האינטראפלנגיאליים המרוחקים )‪ .(PIPs+DIPs‬האבחנה הסבירה ביותר בחולה זה תהיה‪:‬‬
‫א‪ .‬קשחת השלד )‪(Ankylosing Spondylitis‬‬
‫ב‪ .‬דלקת פרקים ראומטואידית )‪(Rheumatoid Arthritis‬‬
‫ג‪ .‬אוסטאוארטרוזיס )‪(Osteoarthrosis‬‬
‫ד‪ .‬אטרופתיה פסוריאטית )‪(Psoriatic arthropathy‬‬

‫‪ .98‬חיילת בת ‪ ,19‬בריאה בד"כ‪ ,‬מתלוננת על כאבים ברגליים עם קשיון בוקר ממושך מזה ארבעה‬
‫ימים‪ .‬מציינת שלפני שבועיים סבלה ממחלת שלשול חריפה‪ .‬בבדיקתה‪ :‬טנדיניטיס אכילס‬
‫משמאל וסינוביטיס בברך ימין‪ .‬איזה טיפול תציע לחולה בשלב זה ?‬
‫א‪ .‬מתן אנטיביוטיקה למשך שבועיים‬
‫ב‪ .‬תרופות אנטי‪-‬דלקתיות לא סטרואידליות )‪(NSAIDS‬‬
‫ג‪ .‬מטוטרקסט בזריקות‬
‫ד‪ .‬תרופות מסוג אנטי‪TNF-‬‬

‫‪ .99‬ניקור מפרק ובדיקה של נוזל סינוביאלי מוצדקים‪:‬‬


‫א‪ .‬רק במפרקים גדולים‬
‫ב‪ .‬אם יש חשד לזיהום‬
‫ג‪ .‬אם יש חשד למחלת גבישים‬
‫ד‪ .‬רק במקרה שיש סינוביטיס )‪ (synovitis‬כרונית‬
‫ה‪ .‬רק במקרה של חשד לזיהום או למחלת גבישים‬

‫‪ .100‬אישה בת ‪ ,26‬חודשיים לאחר לידה‪ ,‬פונה לבדיקת רופא בשל חום ‪ 38°C‬מזה חודש‪ ,‬הרגשת‬
‫תשישות‪ ,‬פריחה בפנים‪ ,‬כאבים ונפיחות בכפות הידיים‪ ,‬וכאבים בחזה‪ .‬האבחנה הסבירה‬
‫ביותר היא‪:‬‬
‫א‪ .‬אוסטאוארטריטיס )‪(Osteoarthritis‬‬
‫ב‪ .‬שיגדון )‪(Gout‬‬
‫ג‪ .‬זאבת אדמנתית מערכתית )‪(Systemic Lupus Erythematosus‬‬
‫ד‪ .‬דלקת פרקים שגרונתית )‪(Rheumatoid Arthritis‬‬
‫ה‪ .‬קדחת שגרונית )‪(Rheumatic fever‬‬

‫‪ .101‬בדיקות מעבדה של החולה הנ"ל )שאלה ‪ ,(100‬יראו קרוב לוודאי‪:‬‬


‫א‪ .‬לויקופניה וטרומבוציטופניה‬
‫ב‪ .‬גורם ראומטואידי בטיטר גבוה )‪(Rheumatoid factor‬‬
‫ג‪ .‬רמה גבוהה של חומצה אורית בדם‬
‫ד‪ .‬רמת משלים ‪ C4‬גבוהה בדם‬
‫ה‪ .‬שקיעת‪-‬דם תקינה‬
‫‪ .102‬חולה בן ‪ ,60‬לפני כחודשיים אובחן כסובל מגידול ממאיר בנזו‪-‬פרינקס‪ .‬מזה שבועיים סובל‬
‫ממחלת חום‪ ,‬פריחה לא מגרדת בפנים ובקדמת החזה‪ ,‬כאבי מפרקים‪ ,‬כאבי שרירים וחולשה‬
‫ניכרת‪ .‬מתקשה להתרומם מהכיסא‪ .‬מזה כשבועיים הופיעו שיעול יבש וקושי בנשימה‪.‬‬
‫כל המשפטים הבאים נכונים ביחס לחולה זה פרט ל‪:‬‬
‫א‪ .‬קיים קשר נסיבתי בין שתי המחלות‬
‫ב‪ .‬בבדיקת שרירים ניתן לזהות חולשה של השרירים הפרוקסימליים‬
‫ג‪ .‬בחלק מהחולים מופיעים סימני תסמונת‪-‬רנו‬
‫ד‪ .‬בביופסיית שריר ניתן לראות סימני דלקת עם הסננה לימפוציטרית ונמק של סיבי השריר‬
‫ה‪ .‬זיהום בדרכי הנשימה הוא ביטוי עיקרי למעורבות ריאות במחלת דרמטומיוזיטיס‬
‫‪ .103‬בת ‪ ,42‬התחילה לסבול מכאבים ונפיחות במפרקי כפות הידיים‪ ,‬התקפים חולפים של כיחלון‬
‫וחיוורון באצבעות הידיים‪ ,‬צרבות והפרעות בליעה‪ .‬בבדיקה נמצאו פצע איסכמי באגודל של‬
‫יד ימין‪ ,‬דפקים היקפיים תקינים‪ ,‬עיבוי של העור בחזה‪ ,‬בגב‪ ,‬בזרועות‪ ,‬בכפות ידיים ורגליים‪.‬‬
‫בבדיקות דם נמצאו נוגדנים מסוג ‪ ANA‬חיובי‪ ,‬ו‪ anti-SCL70-‬חיובי‪.‬‬
‫מה נכון לגבי חולה זו ?‬
‫א‪ .‬מדובר במחלה מסוג ‪Limited systemic sclerosis‬‬
‫ב‪ .‬הטיפול המקובל בצרבות בחולה זו הוא דיאטה‬
‫ג‪ .‬שיעול בחולה זו מעיד על ברונכיטיס‬
‫ד‪ .‬קיים סיכון גבוה למחלת ריאות אינטרסטיציאלית בחולה זו‬
‫ה‪ .‬לא צפויים סיבוכים כילייתיים אצל חולה זו‬
‫‪ .104‬הערכת יעילות של טיפול בדלקת פרקים ראומטואידית מתבצעת ע"י‪:‬‬
‫א‪ .‬בדיקת רמת גורם ראומטואידי בנסיוב‬
‫ב‪ .‬הערכה של מספר הפרקים הרגישים והנפוחים‬
‫ג‪ .‬רמת אלבומין ופריטין בנסיוב‬
‫ד‪ .‬הערכת רמת כאב בסרגל ‪ 150‬ממ'‬
‫ה‪ .‬מדידת חום הגוף וספירת תאים בנוזל סינוביאלי‬
‫‪ .105‬דפוס האימונוגלובולינים בדם היקפי במחלות ראוימטיות הוא בד"כ‪:‬‬
‫א‪ .‬היפרגאמאגלובולינמיה מונוקלונלית‬
‫ב‪ .‬היפרגאמאגלובולינמיה פוליקלונלית‬
‫ג‪ .‬היפוגאמאגלובולינמיה פוליקלונלית‬
‫ד‪ .‬גמופתיה מונוקלונלית‬

‫‪ .106‬אבחנה של מחלה שבבסיסה וסקוליטיס נקבעת על סמך‪:‬‬


‫א‪ .‬סימנים קליניים‬
‫ב‪ .‬בדיקות מעבדה‬
‫ג‪ .‬תוצאות ביופסיה‬
‫ד‪ .‬בדיקות הדמיה‬
‫ה‪ .‬כל המדדים הנ"ל‬
‫ו‪ .‬בדרך כלל ניתן להסתפק ב‪ -‬א‪+‬ב‬

‫‪ .107‬איזה מבין התרופות הבאות יעילה ביותר כטיפול כרוני במחלות ראומטיות‪:‬‬
‫א‪Corticotrophin releasing hormone (CRH) .‬‬
‫ב‪Adrenocorticotrophic hormone (ACTH) .‬‬
‫ג‪ .‬קורטיזול ‪Cortisol -‬‬
‫ד‪ .‬פרדניזון ‪Prednisone -‬‬
‫ה‪ .‬דקסאמתזון ‪Dexamethasone -‬‬
‫‪ .108‬שילוב הבדיקות האופייני לאוסטיאוארתרוזיס הוא‪:‬‬
‫א‪ .‬שקיעת דם מוחשת‪ CRP ,‬תקין‪ ,‬עלייה בסידן בנסיוב וזרחן תקין‬
‫ב‪ .‬שקיעת דם מוחשת‪ CRP ,‬גבוה‪ ,‬סידן וזרחן תקינים‬
‫ג‪ .‬סידן גבוה בדם ובשתן‪ ,‬זרחן נמוך‪ ,‬שקיעת דם תקינה ו‪ CRP-‬גבוה‬
‫ד‪ .‬שקיעת דם תקינה‪ CRP ,‬תקין‪ ,‬עלייה בטרנסאמינזות ללא עליה בפוספטאזה בסיסית‬
‫ה‪ .‬אף תשובה אינה נכונה‬

‫‪ .109‬מה נכון לגבי שיגדון )‪? (Gout‬‬


‫א‪ HLA-B27 .‬תמיד חיובי‬
‫ב‪ .‬דיורטיקה היא גורם סיכון חשוב‬
‫ג‪ .‬שכיחות מירבית בנשים בגיל הפוריות‬
‫ד‪ .‬טיפול הבחירה בהתקף הוא ע"י מתן קולכיצין תוך ורידי‬
‫ה‪ .‬אבחון קשה שכן הקליניקה מקדימה את הופעת ההיפראוריצמיה‬
‫‪ .110‬מבין המשפטים הבאים סמן את המשפט הנכון‪:‬‬
‫א‪ .‬בכל מקרה של היפראוריצמיה יש לטפל באלופורינול‬
‫ב‪ .‬דיאטה היא טיפול חובה בכל מקרה של גאוטף שכן ‪ 1/3‬מהפורינים מקורם במזון‬
‫ג‪ .‬בשילוב של גאוט ואוסטיאוארטרוזיס אין לטפל באלופורינול‬
‫ד‪ .‬משפטים א ו‪-‬ג נכונים‬
‫ה‪ .‬אף משפט אינו נכון‬

‫‪ .111‬מבין הבאים‪ ,‬מי יותר אבחנתי ל‪:Systemic Lupus Erythematosus-‬‬


‫א‪ .‬נוכחות של ‪ Lupus Erythematosus Bodies‬בנוזל פלאורלי‬
‫ב‪ .‬של ‪ Lupus Erythematosus Bodies‬בנוזל פריקרדיאלי‬
‫ג‪ .‬נוכחות מסוג ‪anti-double-stranded DNA‬‬
‫ד‪ Lupus band test .‬חיובי‬

‫‪ .112‬אילו נוגדנים אופייניים ל‪? Diffuse Systemic Sclerosis-‬‬


‫א‪Anticentromere antibodies .‬‬
‫ב‪Anti-DNA topoisomerase antibodies .‬‬
‫ג‪Antihistone antibodies .‬‬
‫ד‪Anti-double-stranded DNA antibodies .‬‬

‫‪ .113‬באיזה חלק של מערכת העיכול השינויים של ‪ Systemic Sclerosis‬יותר בולטים‪:‬‬


‫א‪ .‬חלל הפה‬
‫ב‪ .‬וושט‬
‫ג‪ .‬קיבה‬
‫ד‪ .‬מעי דק‬
‫ה‪ .‬מעי גס‬

‫‪ .114‬מה נכון לגבי ‪? Rheumatoid Arthritis‬‬


‫א‪ .‬הסחוס נהרס ע"י ‪CD8+Lymphocytes‬‬
‫ב‪ .‬גברים חולים במחלה הזאת יותר מנשים‬
‫ג‪ .‬הנוזל הסינוביאלי מכיל מוגלה‬
‫ד‪ .‬הסחוס נהרס ע"י פנוס )‪(Pannus‬‬
‫‪ .115‬כל הבאים קשורים בדרך כלל ל‪ Reactive arthritis-‬פרט ל‪:‬‬
‫א‪Chlamydia .‬‬
‫ב‪Shigella .‬‬
‫ג‪Yersinia .‬‬
‫ד‪Campylobacter .‬‬
‫ה‪Staphylococcus .‬‬

‫‪ .116‬סמן מה נכון לגבי ‪? Paget's disease‬‬


‫א‪ .‬ברב המקרים רק עצם אחת מעורבת‬
‫ב‪ .‬מחלה ויראלית חריפה עם מהלך מהיר‬
‫ג‪ .‬על רקע מחלת ‪ Paget's‬יכולים להתפתח גידולים ממאירים ושפירים בעצם‬
‫ד‪ .‬רב החולים מגיעים לטיפול כירורגי בגלל זיהומים בעצם‬
‫ה‪ .‬המנגנון העיקרי של המחלה ‪ -‬איסכמיה ונמק של העצם‬

‫‪ .117‬חולה בן ‪ 16‬הגיע לבית חולים עם כאבי ברך‪ .‬בצילום רגיל גילו גידול מטאפיזרי בירך עם‬
‫מעורבות של רקמות רכות‪ .‬ב‪ CT-‬חזה גילו גוש בריאה שמאלית‪ .‬מה האבחנה הסבירה‬
‫ביותר?‬
‫א‪Chondrosarcoma .‬‬
‫ב‪Osteoma .‬‬
‫ג‪ .‬גרורה של סרטן ריאה‬
‫ד‪Osteosarcoma .‬‬
‫ה‪Giant cell tumor .‬‬

‫‪ .118‬כיום ידוע כי מנגנון הפעולה המולקולרית של גלוקוקורטיקואידים )‪ (GC‬כולל את כל הבאים‬


‫פרט ל‪:‬‬
‫א‪ .‬קישור ה‪ GC-‬בזיקה גבוהה לרצפטור ציטוזולי ל‪(GCR) GC-‬‬
‫ב‪ .‬קישור ה‪ GC-‬ל‪ GCR-‬ממברנלי‬
‫ג‪ .‬קישור הקומפלקס }‪ {GCR-GC‬ל‪ GC response element (GRE)-‬ב‪DNA-‬‬
‫ד‪ .‬דימריזציה של הקומפלקס }‪{GCR-GC‬‬

‫‪ .119‬סמן את המצב היחיד‪ ,‬מבין המצבים הבאים‪ ,‬שאינו מהווה התווית נגד )‪(contraindication‬‬
‫בשימוש בגלוקוקורטיקואידים כתרופות אנטידלקתיות בטיפול במצבי דלקת כרונית ?‬
‫א‪ .‬סכרת סמויה‬
‫ב‪ .‬אי ספיקת לב‬
‫ג‪ .‬לחץ דם גבוה‬
‫ד‪ .‬רגישות יתר לתרופות‬
‫ה‪ .‬מצב דכאוני‬
‫תמונות‬
‫תמונה מס' ‪:1‬‬

‫תמונה מס' ‪:2‬‬

‫תמונה מס' ‪:3‬‬

‫תמונה מס' ‪:4‬‬

‫תמונה מס' ‪:5‬‬


‫תמונה מס' ‪:6‬‬

‫תמונה מס' ‪:7‬‬

‫תמונה מס' ‪:8‬‬


‫תשובות אפשריות‪:‬‬

‫א‬ ‫‪101‬‬ ‫ד‬ ‫‪81‬‬ ‫ב‬ ‫‪61‬‬ ‫ב‬ ‫‪41‬‬ ‫ג‬ ‫‪21‬‬ ‫ג‬ ‫‪1‬‬
‫ה‬ ‫‪102‬‬ ‫א‬ ‫‪82‬‬ ‫ג‬ ‫‪62‬‬ ‫ד‬ ‫‪42‬‬ ‫ד‬ ‫‪22‬‬ ‫ב‬ ‫‪2‬‬
‫ד‬ ‫‪103‬‬ ‫ב‬ ‫‪83‬‬ ‫ג ‪,‬ד‬ ‫‪63‬‬ ‫ד‬ ‫‪43‬‬ ‫ב‬ ‫‪23‬‬ ‫א‬ ‫‪3‬‬
‫ב‬ ‫‪104‬‬ ‫ג‬ ‫‪84‬‬ ‫א‬ ‫‪64‬‬ ‫ב‬ ‫‪44‬‬ ‫ה‬ ‫‪24‬‬ ‫ה‬ ‫‪4‬‬
‫ב‬ ‫‪105‬‬ ‫א‬ ‫‪85‬‬ ‫ה‬ ‫‪65‬‬ ‫א‬ ‫‪45‬‬ ‫ה‬ ‫‪25‬‬ ‫ב‬ ‫‪5‬‬
‫ו‬ ‫‪106‬‬ ‫ג‬ ‫‪86‬‬ ‫ד‬ ‫‪66‬‬ ‫ג‬ ‫‪46‬‬ ‫ב‬ ‫‪26‬‬ ‫ג‬ ‫‪6‬‬
‫ד‬ ‫‪107‬‬ ‫ד‬ ‫‪87‬‬ ‫ב‬ ‫‪67‬‬ ‫ה‬ ‫‪47‬‬ ‫ד‬ ‫‪27‬‬ ‫ד‬ ‫‪7‬‬
‫ה‬ ‫‪108‬‬ ‫ד‬ ‫‪88‬‬ ‫ג‬ ‫‪68‬‬ ‫ה‬ ‫‪48‬‬ ‫א‬ ‫‪28‬‬ ‫ד‬ ‫‪8‬‬
‫ב‬ ‫‪109‬‬ ‫א‬ ‫‪89‬‬ ‫ד‬ ‫‪69‬‬ ‫ד‬ ‫‪49‬‬ ‫ב‬ ‫‪29‬‬ ‫ג‬ ‫‪9‬‬
‫ה‬ ‫‪110‬‬ ‫ד‬ ‫‪90‬‬ ‫ד‬ ‫‪70‬‬ ‫א‬ ‫‪50‬‬ ‫ג‪,‬א‬ ‫‪30‬‬ ‫ב‬ ‫‪10‬‬
‫ג‬ ‫‪111‬‬ ‫ה‬ ‫‪91‬‬ ‫א‬ ‫‪71‬‬ ‫ד‬ ‫‪51‬‬ ‫ד‬ ‫‪31‬‬ ‫ה‬ ‫‪11‬‬
‫ב‬ ‫‪112‬‬ ‫ב‬ ‫‪92‬‬ ‫א‬ ‫‪72‬‬ ‫ג‬ ‫‪52‬‬ ‫א‬ ‫‪32‬‬ ‫ב‬ ‫‪12‬‬
‫ב‬ ‫‪113‬‬ ‫ה‬ ‫‪93‬‬ ‫א‬ ‫‪73‬‬ ‫ב‬ ‫‪53‬‬ ‫ב‬ ‫‪33‬‬ ‫ד‬ ‫‪13‬‬
‫ד‬ ‫‪114‬‬ ‫ג‬ ‫‪94‬‬ ‫ה‬ ‫‪74‬‬ ‫ג‬ ‫‪54‬‬ ‫ד‬ ‫‪34‬‬ ‫ב‬ ‫‪14‬‬
‫ה‬ ‫‪115‬‬ ‫ה‬ ‫‪95‬‬ ‫ד‬ ‫‪75‬‬ ‫א‬ ‫‪55‬‬ ‫ב‬ ‫‪35‬‬ ‫ה‬ ‫‪15‬‬
‫ג‬ ‫‪116‬‬ ‫ג‬ ‫‪96‬‬ ‫ג‬ ‫‪76‬‬ ‫ב‬ ‫‪56‬‬ ‫ב‬ ‫‪36‬‬ ‫ב‬ ‫‪16‬‬
‫ד‬ ‫‪117‬‬ ‫ד‬ ‫‪97‬‬ ‫ב‬ ‫‪77‬‬ ‫ה‬ ‫‪57‬‬ ‫ג‬ ‫‪37‬‬ ‫ד‬ ‫‪17‬‬
‫ד‬ ‫‪118‬‬ ‫ב‬ ‫‪98‬‬ ‫ג‬ ‫‪78‬‬ ‫ד‬ ‫‪58‬‬ ‫א‬ ‫‪38‬‬ ‫ב‪,‬א‬ ‫‪18‬‬
‫ד‬ ‫‪119‬‬ ‫ה‬ ‫‪99‬‬ ‫א‬ ‫‪79‬‬ ‫ב‬ ‫‪59‬‬ ‫ה‬ ‫‪39‬‬ ‫א‬ ‫‪19‬‬
‫ג‬ ‫‪100‬‬ ‫א‬ ‫‪80‬‬ ‫ד‬ ‫‪60‬‬ ‫א‬ ‫‪40‬‬ ‫ג‬ ‫‪20‬‬
‫)עיכול‪ ,‬כבד‪ ,‬כליות וראומטולוגיה(‬
‫מקבץ ‪ – 2‬מועד א' ‪2007‬‬

‫עיכול‬

‫התאם בין הגידול ומאפייניו ) כל גידול יגול‬


‫להופיע פעם אחת או יותר או לא להופיע כלל(‪.‬‬
‫‪ .9‬להלן תמונה מס' ‪ ,2‬מהי אבחנתך?‬ ‫הגידולים‪:‬‬
‫א‪ .‬המרטומה‬
‫ב‪Vilous Adenoma .‬‬
‫ג‪Tubular Adenoma .‬‬
‫ד‪Hyperplastic Polyp .‬‬
‫המאפיינים‪:‬‬
‫‪ .1‬הגידול הגדול ביותר‬
‫‪ .2‬גידול אפיתל נאופלסטי‬
‫‪ .3‬חלק מ‪Familial Adenomatous -‬‬
‫‪Poliposys‬‬
‫‪Peutz Jegher .4‬‬
‫‪ .5‬הכי פחות ממאיר‬
‫‪ .10‬את כל הבאים נראה במחלה האופיינית‬
‫לתמונה זו‪ ,‬פרט ל‪:-‬‬ ‫‪ .6‬להלן תמונה מס' ‪ ,1‬מהי אבחנתך?‬
‫א‪ .‬גרנולומות‬ ‫‪.‬‬
‫ב‪ .‬בהכרח מעורבות של הרקטום‬
‫ג‪ .‬פגיעה טרנסמורלית‬
‫‪ .11‬תמונה מס' ‪ ,3‬ציין שני מאפיינים שנראים‬
‫בתמונה ואופיינים ל ‪ HEPATITIS‬על רקע‬
‫‪?HCV‬‬

‫‪ .7‬תמונה ‪ 1‬נגרמת עקב‪:‬‬


‫א‪GERD .‬‬
‫ב‪ .‬כיב פפטי‬
‫ג‪ .‬ירידה במשקל וחוסר תיאבון‬
‫ד‪ .‬סרטן הקולון‬
‫‪ .12‬אילו תאים נראים להלן בתמונה מס' ‪? 4‬‬
‫‪ .8‬שינוי זה )תמונה ‪ (1‬יכול לגרום ל‪:-‬‬
‫א‪Adenocarcinoma .‬‬
‫ב‪Squamous cell carcinoma .‬‬
‫ג‪gastritis .‬‬
‫ד‪ .‬כיב פפטי‬
‫‪ .17‬איזה ממצא מצפים למצוא בביופסית כבד‬
‫של חולה בן ‪ ,35‬שסובל מ‪ ,UC-‬ומפתח הפרעה‬
‫כולסטטית‪:‬‬
‫א‪HCC .‬‬
‫ב‪ .‬שינוי שומני קשה‬
‫ג‪ .‬גופיפי מלורי‬
‫ד‪Ground glass hepatocytes .‬‬
‫ה‪ Onion skin fibrosis .‬סביב צינורות מרה‬

‫‪ .18‬כל הבאים אופיינים להפטיטיס אלכוהולי‬


‫פרט ל‪:‬‬
‫א‪ .‬הסננה ע"י ניוטרופילים‬
‫ב‪Ground glass hepatocyte .‬‬ ‫‪ .13‬כל הבאים נכונים לגבי המחלה בה‬
‫ג‪ .‬פיברוזיס‬ ‫מופיעים תאים אלו‪ ,‬חוץ מ‪:-‬‬
‫א‪ .‬גבולות הגידול מטושטשים‬
‫ד‪Ballooning of hepatocytes .‬‬ ‫ב‪ .‬השכיחות לא ירדה בחמישים השנים‬
‫ה‪.Malory bodies .‬‬ ‫האחרונות‬
‫ג‪ .‬השכיחות שווה בין המינים‬
‫‪ .19‬כל הבאים אופיינים להפטיטיס אקוטית‬ ‫ד‪ .‬קשור לגסטריטיס כרונית ומטפלזיה‬
‫פרט ל‪:‬‬ ‫אינטסטינלית‬
‫א‪Ballooning degeneration .‬‬
‫ב‪Lobular disarray .‬‬
‫ג‪ .‬פיסמיל נקרוזיס‬
‫ד‪Acidophil bodies .‬‬ ‫‪ .14‬מהו הגיל שצריך לעשות בדיקה לגילוי‬
‫ה‪ .‬היפרטרופיה של תאי קופפר‬ ‫סרטן המעי הגס בגברים ללא סיכון?‬
‫א‪30 .‬‬
‫‪ .20‬כל הבאים לגבי הפטובלסטומה נכונים‬ ‫ב‪40 .‬‬
‫פרט ל‪:‬‬ ‫ג‪50 .‬‬
‫א‪ .‬אופייני אצל ילדים‬ ‫ד‪65 .‬‬
‫ב‪ .‬יכול להכיל מוקדים של רקמת סחוס או‬
‫עצם‬ ‫‪ .15‬אישה בת ‪ 22‬עם ‪ 2-3‬יציאות ביום כל‬
‫ג‪ .‬מהווה גידול שפיר‬ ‫החיים‪ ,‬איזו בדיקה תמליץ?‬
‫ד‪ .‬יכול להכיל אזורי דימום נמק והיסתידויות‪.‬‬ ‫א‪ .‬גסטרוסקופיה‬
‫ה‪ .‬לחולה יש ברוב המקרים רמה גבוה של‬ ‫ב‪ .‬דם סמוי בצואה‬
‫ג‪ .‬קולונוסקופיה‬
‫חלבון עוברי )‪(alpha fetoprotein‬‬ ‫ד‪ .‬בדיקות דם‬
‫ה‪ .‬ביופסיה‬
‫‪ .21‬הגידול השכיח ביותר בכבד מבין הבאים‬ ‫ו‪ .‬אף תשובה אינה נכונה‬
‫הוא‪:‬‬
‫א‪cholangio carcinoma .‬‬ ‫‪ .16‬איזה מהממצאים הבאים יותר אופייני ל‪-‬‬
‫ב‪HCC .‬‬ ‫‪:Inactive/healed UC‬‬
‫ג‪Angio sarcoma .‬‬ ‫א‪ .‬גרנולומים‬
‫ד‪Hepato cellular adenoma .‬‬ ‫ב‪ .‬התכייבויות‬
‫ה‪ .‬גרורה‬ ‫ג‪crypt abscesses .‬‬
‫ד‪Distorsion of architecture .‬‬
‫‪ .22‬איזה מהשינויים הבאים מאפיינים את‬ ‫ה‪ .‬צברים לימפטים בכל דופן המעי‪.‬‬
‫השלב ההתחלתי של התפתחות סרטן הלבלב?‬
‫א‪ .‬מוטציה בגן ‪P53‬‬ ‫כבד‬
‫ב‪ .‬מוטציה בגן ‪.BRCA2‬‬
‫ג‪ .‬קיצור טלומר‬
‫ד‪ .‬אף תשובה אינה נכונה‬
‫ה‪ .‬הביטוי הקליני שלה‪Renovascular -‬‬ ‫ה‪ .‬כל התשובות נכונות‬
‫‪Hypertension‬‬
‫‪ .23‬מה נכון לגבי ‪ pseudo cyst‬בלבלב?‬
‫‪ .30‬בדיקות הדם שיכולות לכוון לאבחנה‬ ‫א‪ .‬אחד מהסיבוכים של ‪acute pancriatitis‬‬
‫ספציפית של סלייד מס' ‪:1‬‬ ‫ב‪ .‬אחד הסיבוכים של ‪chronic pancriatitis‬‬
‫א‪ C3 .‬נמוך‬ ‫ג‪ .‬נגע ציסטי השכיח ביותר בלבלב‬
‫ב‪ANCA .‬‬ ‫ד‪ .‬כל התשובות נכונות‬
‫ג‪ .‬אנטי ‪GBM‬‬ ‫ה‪ .‬אף תשובה אינה נכונה‬
‫ד‪ .‬כל התשובות נכונות‬
‫‪ .24‬איזה מהנגעים הבאים מהווה סיכון גבוה‬
‫‪ .31‬בתמונה מס' ‪ 5‬במיקרוסקופ אלקטורוני‬ ‫להתפתחות סרטן כיס המרה?‬
‫)‪:(Minimal Change Disease‬‬ ‫א‪cholesterolosis .‬‬
‫א‪ .‬בן ‪ ,40‬גלילים המטיים‪ ,‬פרוטאינוריה קלה‬ ‫ב‪Porcelain gall bladder .‬‬
‫והדרדרות מהירה בתיפקוד כליה‬ ‫ג‪ .‬פפילומטוזיס‬
‫ב‪ .‬בן ‪ ,5‬עם פרוטאינוריה מאסיבית‪,‬בצקות‪,‬‬ ‫ד‪ .‬אף תשובה‬
‫היפואלבומינמיה‬ ‫ה‪ .‬כל התשובות‬
‫ג‪ .‬בן ‪ 60‬עם פרוטאינוריה מאסיבית‪,‬בצקות‪,‬‬
‫היפואלבומינמיה על רקע ‪HBV‬‬ ‫‪ .25‬מה הגורם העיקרי ל‪Acute -‬‬
‫ד‪ .‬בן ‪ 60‬עם פרוטאינוריה מאסיבית‪,‬בצקות‪,‬‬ ‫‪?cholecystitis‬‬
‫היפואלבומינמיה על רקע ‪CRF‬‬ ‫א‪ .‬זיהום ע"י סלמונלה טיפי‬
‫ה‪ .‬בן ‪ 60‬עם פרוטאינוריה מאסיבית‪,‬בצקות‪,‬‬ ‫ב‪ .‬וסקוליטיס‬
‫היפואלבומינמיה על רקע עמילואידוזיס‬ ‫ג‪ .‬זיהום ויראלי בדרכי המרה‬
‫ד‪ .‬אבנים בדרכי המרה‬
‫‪ .32‬בתמונה מס' ‪ 6‬במיקרוסקופ אלקטרוני )‬ ‫ה‪ .‬תרופות‬
‫‪ (Membranous Nephropathy‬הממצאים‬
‫האופיינים במיקרוסקופ אור‪:‬‬ ‫כליה‬
‫א‪spikes and domes .‬‬
‫ב‪wire loops .‬‬ ‫‪ .26‬סלייד מס' ‪Acute Pyelonephritis – 1‬‬
‫ג‪Glumerulonephritis .‬‬ ‫‪ .27‬לגבי המחלה שבסלייד מס' ‪:1‬‬
‫ד‪crescents .‬‬ ‫א‪ .‬הביטוי הקליני שלה – ‪CRF‬‬
‫ה‪nodular glumeruloscleriss .‬‬ ‫ב‪ .‬הביטוי הקליני שלה – ‪nephritic‬‬
‫‪ .33‬להלן תמונה מס' ‪:7‬‬ ‫‪syndrome‬‬
‫ג‪ .‬הביטוי הקליני שלה – ‪ARF‬‬
‫ד‪ papillary necrosis .‬ו‪pyonephrosis -‬‬
‫יכולים להיות סיבוך שלה‬
‫ה‪ .‬היא ‪drug induced‬‬

‫‪ .28‬סלייד מס' ‪crescentic GN – 2‬‬

‫‪ .29‬לגבי המחלה שבסלייד מס' ‪:2‬‬


‫א‪ .‬הביטוי הקליני שלה ‪-‬‬
‫‪Glumerulonephritis‬‬
‫ב‪ .‬הביטוי הקליני שלה – ‪RPGN‬‬
‫במחלה שבתמונה נמצא‪:‬‬ ‫ג‪ .‬הביטוי הקליני שלה‪Chronic Renal -‬‬
‫א‪CRF and subarachnoid hemorrhage.‬‬ ‫‪Failure‬‬
‫ד‪ .‬הביטוי הקליני שלה‪Nephrotic -‬‬
‫‪ .34‬טרומבים בקפילרים גלומרולרים יכולים‬ ‫‪Syndrome‬‬
‫להיות ביטוי של כל המחלות הבאות פרט ל‪:-‬‬
‫א‪renal artery stenosis .‬‬
‫‪ .40‬חולה סכרת בן ‪ 65‬מופיע עם תסמונת‬ ‫ב‪DIC .‬‬
‫נפרוטית ואי ספיקת כליות כרונית הכל נכון‬ ‫ג‪hemolytic uremic syndrome .‬‬
‫פרט ל‪:‬‬
‫ד‪TTP .‬‬
‫א‪ .‬ב‪ EM-‬נוכל לראות קומפלקסים אימונים‬
‫סוב אפיתליאלים‬ ‫ה‪malignant hypertension .‬‬
‫ב‪ .‬ב‪ LM-‬נוכל לראות ‪nodular GS‬‬
‫‪ .35‬נמק נרחב של אפיתל טובולרי שפוגע ברוב‬
‫ג‪ .‬ב‪ LM-‬נוכל לראות ארתרוסקלרוזיס‬ ‫הטובולי הפרוקסימלי אופייני ל‪:-‬‬
‫היאליני גם בארתריולה האפרנטית‬
‫ד‪ .‬פפילרי נקרוזיס הוא סיבוך של המחלה‬ ‫א‪drug induced acute interstitial .‬‬
‫ה‪advanced glycation end products .‬‬ ‫‪nephritis‬‬
‫משתתפים בפתוגנזה‬ ‫ב‪contrast medium associated ATN .‬‬
‫ג‪ischemic ATN .‬‬
‫‪ .41‬חולה בת ‪ 22‬עם יתר לחץ דם‪ ,‬משקע שתן‬ ‫ד‪atheroemboli .‬‬
‫גלילים המטים‪ C3 ,‬נמוך‪ ,‬ב‪ EM-‬משקעים‬ ‫ה‪hemolytic uremic syndrome .‬‬
‫סוב אנדותליאלים ו‪ TRI-‬בתאי האנדותל‪.‬‬
‫ב‪ LM-‬נמצא‪:‬‬ ‫‪ .36‬כל הבאים אופיינים ל‪RCC ( renal cell -‬‬
‫א‪Humps .‬‬ ‫)‪ carcinoma‬פרט ל‪:-‬‬
‫ב‪Spikes and domes .‬‬ ‫א‪ .‬תאים בהירים )‪ (clear‬ברוב המקרים‬
‫ב‪ .‬גוון צהבצב לגידול מאקרוסקופי‬
‫ג‪Double contour of GBM .‬‬ ‫ג‪ .‬נטייה לחדור לכלי דם ורידיים‬
‫ד‪Wire loops .‬‬ ‫ד‪ .‬חסר ‪ 3p‬שכיח‬
‫ה‪ .‬נודולר סקלרוזיס‬ ‫ה‪ .‬נטייה לגדולים מרובים ובילטרלים‬
‫‪ .42‬חולה בן ‪ ,70‬פרוטנוריה של ‪ 8‬גרם ליממה‪,‬‬ ‫‪ .37‬מה נכון לגבי ‪? analgetic nephropathy‬‬
‫היפואלבולינמיה ובצקות‪ .‬בגלומרולי‬
‫רואים עיבוי דפנות הקפילרים ללא ריבוי‬ ‫א‪ .‬מתבטאת כ‪acute renal failure -‬‬
‫ב‪ .‬תוצאה של רגישות יתר לתרופות‬
‫תאים‪ .‬ב‪ IF-‬שקיעה גלומרולרית עדינה‪,‬‬ ‫ג‪ .‬פגיעה מתבטאת בנמק פפילרי‬
‫דיפוזית לאורך כל הקיפלרים‬ ‫ד‪ .‬תסמונת נפריטית היא ממצא אופייני‬
‫הגלומרולרים של ‪ IgG‬ללא ‪ IgM‬או ‪.IgA‬‬ ‫בשלבים מאוחרים‬
‫שקיעה חלשה של ‪ .C3‬הרקע למחלה יכול‬ ‫ה‪ .‬כל התשובות נכונות‬
‫להיות הכל פרט ל‪:‬‬
‫א‪ .‬קרצינומה‬ ‫‪ .38‬טרומבי בקפילרים גלומרולריים יכולים‬
‫ב‪SBE .‬‬ ‫להיות ביטוי של כל מחלות הבאות פרט ל‪:-‬‬
‫ג‪ .‬הפטיטיס ‪B‬‬ ‫א‪renal artery stenosis .‬‬
‫ד‪ .‬פניצילאמין‬ ‫ב‪DIC .‬‬
‫ה ‪ .‬תרופה )זהב(‬ ‫ג‪HUS .‬‬
‫ד‪TTP.‬‬
‫‪ .43‬חולה עם הפטיטיס ‪ C‬וקריוגלובולינמיה‬ ‫ה‪Malignant hypertension .‬‬
‫פיתח מחלת כליה עם פרוטינאוריה של ‪4‬‬
‫גרם ליממה‪ ,‬גלילים המטים במשקע‬
‫השתן‪ ,‬יל"ד‪ ,‬ועליה קלה בקריאטינין‪ .‬מה‬
‫נראה בביופסיה‪:‬‬ ‫‪ .39‬כל הבאים נכון לגבי גידולים בכיס השתן‬
‫א‪ .‬איחוי נרחב של רגליות הפודוציטים‬ ‫פרט ל‪:‬‬
‫כממצא בלעדי‬ ‫א‪ .‬רובם גידולים של אפיתל מעבר‬
‫ב‪ .‬מופיע ברוב החולים עם וון היפל לנדו‬
‫ב‪ .‬שקיעה של ‪ IgA‬במזנגיום‬ ‫ג‪ .‬קשורים לחשיפה לבטא‪-‬נפטילמין‬
‫ג‪ .‬סהרונים‬ ‫ד‪ .‬חדירה לשריר הדופן מחייבת כריתה של‬
‫ד‪ .‬ריבוי תאים בגלומרולי עם ‪Humps‬‬ ‫כיס השתן או הקרנות‬
‫ה‪ .‬קונטור כפול של ה‪ GBM-‬עם ‪Mesangial‬‬ ‫ה‪ .‬נוטים לגדול שוב בכיס השתן לאחר‬
‫‪interposition‬‬ ‫הכריתה‬
‫ה‪ .‬רבוי תאים בגלומרולי )פרוליפרציה( עם‬
‫נויטרופילים ו‪ IgG-‬בדפנות הקפילרים‬ ‫‪ .44‬חולה בן ‪ 18‬עם המטוריה חוזרת ללא‬
‫פרוטאינוריה‪ ,‬עם תפקוד תקין של הכליה‪.‬‬
‫‪ C3‬תקין‪ .‬נוכל למצוא בביופסיה כיליתית‬
‫ראומטולוגיה‬ ‫את כל הבאים פרט ל‪:‬‬
‫א‪ GBM .‬דק מהרגיל ב‪EM-‬‬
‫‪ .45‬מה לא נכון לגבי ‪? Ewing’s sarcoma‬‬ ‫ב‪ IgA .‬במזנגיום‬
‫א‪ .‬מופיע הסמן ‪O13‬‬ ‫ג‪ .‬עיבוי עם למלציות ואירגולריות של ה‪-‬‬
‫ב‪ .‬טיפול כירורגי יעזור‬ ‫‪GBM‬‬
‫ג‪ .‬מופיע גם ברקמות רכות‬ ‫ד‪ .‬ריבוי תאי מזנגיום‬
‫ד‪ .‬טרנסלוקציה של ‪11-12‬‬

‫תשובות אפשריות‪:‬‬
‫ב‬ ‫‪.24‬‬ ‫א‪+‬ב‬ ‫‪.1‬‬
‫‪.25‬‬ ‫א‪+‬ד‬ ‫‪.2‬‬
‫‪.26‬‬ ‫ג‬ ‫‪.3‬‬
‫ד‬ ‫‪.27‬‬ ‫א‬ ‫‪.4‬‬
‫‪.28‬‬ ‫א‪+‬ד‬ ‫‪.5‬‬
‫ב‬ ‫‪.29‬‬ ‫‪Intestinal Metaplasia‬‬ ‫‪.6‬‬
‫ד‬ ‫‪.30‬‬ ‫א‬ ‫‪.7‬‬
‫ב‬ ‫‪.31‬‬ ‫א‬ ‫‪8‬‬
‫א‬ ‫‪.32‬‬ ‫‪Fissure Ulcer‬‬ ‫‪.9‬‬
‫א‬ ‫‪.33‬‬ ‫ב‬ ‫‪.10‬‬
‫א‬ ‫‪.34‬‬ ‫א‪steatosis .‬‬ ‫‪.11‬‬
‫ב‪portal lymphoid aggregates .‬‬
‫‪.35‬‬ ‫‪Signet ring cell‬‬ ‫‪.12‬‬
‫ה‬ ‫‪.36‬‬ ‫ד‬ ‫‪.13‬‬
‫ג‬ ‫‪.37‬‬ ‫ג‬ ‫‪.14‬‬
‫א‬ ‫‪.38‬‬ ‫ו‬ ‫‪.15‬‬
‫ב‬ ‫‪.39‬‬ ‫ד‬ ‫‪.16‬‬
‫א‬ ‫‪.40‬‬ ‫ה‬ ‫‪.17‬‬
‫ד‬ ‫‪.41‬‬ ‫ב‬ ‫‪.18‬‬
‫ב‬ ‫‪.42‬‬ ‫ג‬ ‫‪.19‬‬
‫ה‬ ‫‪.43‬‬ ‫ג‬ ‫‪.20‬‬
‫ה‬ ‫‪.44‬‬ ‫ה‬ ‫‪.21‬‬
‫‪.45‬‬ ‫ג‪+‬ה‬ ‫‪.22‬‬
‫ד‬ ‫‪.23‬‬
‫מקבץ ‪III‬‬
‫מקבץ ‪2007-2008 III‬‬
‫מחלות זיהומיות‬
‫‪ .1‬מה נכון לגבי מקרולידים ?‬
‫א‪ .‬אינם פעילים כנגד רוב החיידקים הגראם שליליים והאנאירוביים‬
‫ב‪ .‬חדירות טובה לתאים‬
‫ג‪ .‬עמידים במנגנון ‪Efflux‬‬
‫ד‪ .‬המטבוליזם העיקרי מתרחש בכבד‬
‫ה‪ .‬הכל נכון‬

‫‪ .2‬אין מקום לטיפול אנטיביוטי בחולים עם בקטריאוריה אסימפטומטית‪ ,‬פרט ל‪:‬‬


‫א‪ .‬חולים סכרתיים‬
‫ב‪ .‬חולים קשישים‬
‫ג‪ .‬חולים לפני עקירת שן‬
‫ד‪ .‬נשים בהריון‬
‫ה‪ .‬חולים עם קטטר קבוע בשלפוחית‬

‫‪ .3‬בת ‪ ,30‬בריאה בד"כ‪ ,‬פונה למרפאה עקב צריבה ודחיפות במתן שתן‪ ,‬ללא חום או כאבים‬
‫במותניים‪ ,‬לראשונה בחייה‪ .‬בבדיקת שתן לכללית |‪ pyuria‬וניטרטים חיוביים‪ .‬כיצד תמליץ‬
‫להמשיך ?‬
‫א‪ .‬להתחיל טיפול אנטיביוטי ולא לשלוח לתרבית‬
‫ב‪ .‬לשלוח תרבית ולחכות לתשובה לפני הטיפול המתאים‬
‫ג‪ .‬אין צורך בתרבית או טיפול‪ .‬יש להמליץ על שתייה מרובה‬
‫ד‪ .‬לשלוח תרבית שתן ולהתחיל אנטיביוטיקה‬
‫ה‪ .‬לשלוח תרבית שתן ודם ולהתחיל טיפול אנטיביוטי‬

‫‪ .4‬איזה מהבאים נחשב קריטריון "גדול" של אנדוקרדיטיס‪:‬‬


‫א‪ .‬שתי תרביות דם חיוביות לקנדידה אלביקנס אצל חולת סרטן אחרי טיפול כמותרפי‪,‬‬
‫שנלקחו בהפרש של ‪ 24‬שעות ביניהן‬
‫ב‪ .‬שתי תרביות של ‪ ,Enterococcus Faecalis‬הראשונה נלקחה ביום קבלתו והשנייה ‪24‬‬
‫שעות אח"כ‪ .‬החולה בן ‪ 70‬ומאושפז עם אבחנה של זיהום נרכש בקהילה‬
‫ג‪ .‬שלוש תרביות דם חיוביות לפסאודומונס אצל חולה בטיפול נמרץ עם צנתר מרכזי‬
‫ד‪ .‬שתי תרביות דם חיוביות ל‪ GBS-‬בחולה עם זיהום אופייני ל‪Erysipelas-‬‬
‫ה‪ .‬שתי תרביות דם חיוביות ל‪ Staph. Coagulase negative-‬שנלקחו בהפרש של ‪ 24‬שעות‬
‫מחולה עם זיהום ורידי משני לצנתר‬

‫‪ .5‬לגבי הטיפול באנדוקרדיטיס שנגרם על ידי ‪ Staph.aureus‬במסתם טבעי‪ ,‬איזה מההיגדים‬


‫הבאים הוא הנכון‪:‬‬
‫א‪ .‬אם החיידק רגיש ל‪ ,MSSA-‬תרופת הבחירה היא ‪Ceftriaxone‬‬
‫אם החיידק הוא ‪ ,MSSA‬הטיפול המומלץ הוא ‪ Gentamicin‬ו‪ Cloxacillin -‬במשך ‪4‬‬ ‫ב‪.‬‬
‫שבועות לפחות‬
‫אם החיידק הוא ‪ MSSA‬והחולה אלרגי לפניצילין )תפרחת עורית מגרדת בעבר(‪ ,‬התחליף‬ ‫ג‪.‬‬
‫ל‪ Cloxacillin-‬הוא ‪Cefazolin‬‬
‫אם החיידק עמיד למתיצילין )‪ , (MRSA‬השילוב של ‪ Gentamicin‬עם ‪,Vancomycin‬‬ ‫ד‪.‬‬
‫לאורך כל משך הזמן של הטיפול מאפשר תוצאות טובות יותר )בהשוואה לטיפול עם‬
‫‪ Vancomycin‬בלבד(‪.‬‬
‫ה‪ Gentamicin .‬ו‪ Cloxacillin-‬אינן תרופות סינרגיסטיות נגד ‪Staph.aureus‬‬
‫‪ .6‬מה נכון לגבי גונוראה ?‬
‫א‪ 80% .‬מהנשים הן סימפטומטיות‬
‫ב‪ 80% .‬מהגברים הם סימפטומטיים‬
‫ג‪ .‬טיפול הבחירה הוא פניצילין‬
‫ד‪ .‬טיפול בחולה גונוראה משולב תמיד עם טיפול בהרפס‬
‫ה‪ .‬רוב החולים מפתחים אלח דם‬

‫‪ .7‬ציין מה נכון לגבי דלקת המפרק מסיבה חיידקית )ספטיק ארטריטיס(‬


‫א‪ coagulase negative staph .‬הם הגורמים השכיחים ביותר‬
‫ב‪ .‬זיהום מסטאף אאורוס הוא שכיח רק אצל צרכני סמיםקשים דרך הוריד‬
‫ג‪ .‬קנדידה הנה הסיבה השכיחה ביותר של זיהום חד )‪.(acute infection‬‬
‫ד‪ .‬תרביות דם הינם חיוביים רק במיעוט המקרים‬
‫ה‪ .‬במקרה טיפוסי בדיקת הנוזל הסינוביאלי מגלה נוכחות של כדוריות לבנות מעל ‪50,000‬‬
‫תאים‪/‬מ"מ‪.‬‬

‫‪ .8‬מבין התרופות הבאות‪ ,‬מי התרופה היעילה כנגד פסאודומונס?‬


‫א‪cefuroxime .‬‬
‫ב‪ceftriaxone .‬‬
‫ג‪ceftazimide .‬‬
‫ד‪cefazolin .‬‬
‫ה‪cefotaxime .‬‬

‫‪ .9‬מה נכון לגבי זיהום בדרכי השתן הנרכש בבית החולים?‬


‫א‪ .‬הגורם הוא בדרך כלל חיידקים גרם חיוביים‬
‫ב‪ .‬קנדידה אינו גורם שכיח במצב זה‬
‫ג‪ .‬רוב המקרים קשורים לניתוחים‬
‫ד‪ .‬רוב המקרים קשורים לקטטרים )צנתרים(‬
‫ה‪ .‬כל התשובות נכונות‬

‫‪ .10‬בת ‪ 70‬מאושפזת בגלל דלקת ריאות‪ .‬ביום השמיני של הטיפול הופיעו שלשולים רבים‪ .‬מה הוא‬
‫הפתוגן שבסבירות הגבוהה ביותר גרם לכך?‬
‫א‪rotavirus .‬‬
‫ב‪C. difficile .‬‬
‫ג‪Adenovirus .‬‬
‫ד‪Salmonella .‬‬
‫ה‪Shigella .‬‬

‫‪ .11‬מה נחשב כגורם הסיכון החשוב ביותר לזיהום בזרם הדם )‪ (bloodstream infection‬בחולה‬
‫מאושפז ?‬
‫א‪ .‬צנתר עורקי‬
‫ב‪ .‬צנתר ורידי מרכזי‬
‫ג‪ .‬צנתר ורידי היקפי‬
‫ד‪ .‬לקיחת דם מוריד היקפי‬
‫ה‪ .‬לכל הממצאים הנ"ל אותה מידה של סיכון‬

‫‪ .12‬ציין איזה מבין תופעות הלוואי הבאות היא אופיינית ל‪:Vancomycin-‬‬


‫א‪ .‬תפרחת עור אדומה בחלק העליון של הגוף המוכרת בשם ‪red man syndrome‬‬
‫ב‪ .‬דלקת כבד‬
‫ג‪ .‬דלקת מעי מהחיידק ‪C.defficile‬‬
‫ד‪ .‬תרומבוציטופניה‬
‫ה‪Gray baby syndrome .‬‬
‫‪ .13‬מי מהתרופות הבאות נחשבת כתרופת הבחירה לטיפול בחולה מבוגר הסובל מ‪-‬‬
‫‪) Mediterranean spotted fever‬שנגרם מריקציה( ‪:‬‬
‫א‪Vnacomycin .‬‬
‫ב‪Ceftriaxone .‬‬
‫ג‪Cefazolin .‬‬
‫ד‪Doxycyclin .‬‬
‫ה‪Chloramphenicol .‬‬

‫‪ .14‬באיזה מהמחלות הבאות היית מטפל ב‪? Vancomycin-‬‬


‫א‪ .‬זיהום בזרם הדם בקלבסיאלה‬
‫ב‪ .‬דלקת ריאות מ‪Enterobacter-‬‬
‫ג‪ .‬זיהום של ‪ ventriculo-peritoneal shunt‬שנגרם מ‪coagulase negative -‬‬
‫‪staphylococcus‬‬
‫ד‪ .‬דלקת ריאות מליגיונלה‬

‫‪ Aminoglycosides .15‬יעילים בטיפול בחולים עם זיהומים שנגרמים על‪-‬ידי‪:‬‬


‫א‪Pneumococcus .‬‬
‫ב‪Streptococcus .‬‬
‫ג‪E.coli .‬‬
‫ד‪Bacteroides .‬‬
‫ה‪ .‬כל התשובות נכונות‬

‫‪ .16‬מבין החיידקים הבאים איזה מהם הוא עמיד ל‪:Ciprofloxacin-‬‬


‫א‪B.fragilis .‬‬
‫ב‪E.Coli .‬‬
‫ג‪Pseudomonas .‬‬
‫ד‪Staph. aureus .‬‬
‫ה‪Legionella pneumophila .‬‬

‫‪ .17‬יליד בן ‪ ,10‬בריא בד"כ‪ ,‬סימני של דלקת קרום המוח‪ .‬נעשה ‪ LP‬ונמצא ‪,protein – 147mg%‬‬
‫‪ .Glucose 15/97 (CSF/blood) ,WBC – 847 ,PMN – 92%‬בצביעת גראם לא נמצאו‬
‫חיידקים‪ .‬מה המחולל הסביר ביותר למחלתו ?‬
‫א‪Enterovirus .‬‬
‫ב‪Strep.pneumoniae .‬‬
‫ג‪Herpes Simplex .‬‬
‫ד‪H.influenza type b .‬‬
‫ה‪Listeria monocytogenes .‬‬

‫‪ .18‬מתי מומלץ לטפל ב‪ Primaquine-‬כדי למנוע הישנות של מלריה ?‬


‫א‪ .‬לאחר זיהום ב‪ P.falciparum-‬שנרכש עקב הזרקת סמים במזרק משותף‬
‫ב‪ .‬לאחר זיהום ב‪ P.vivax-‬שנרכש מעירוי דם‬
‫ג‪ .‬לאחר זיהום ב‪ P.vivax-‬שנרכש בטיול בהודו‬
‫ד‪ .‬לאחר זיהום ב‪ P.falciparum-‬שנרכש באפריקה בטיול‬
‫ה‪ .‬לאחר זיהום ב‪ P.malariae-‬שנרכש במרכז אמריקה בטיול‬
‫‪ .19‬לגבי ‪ ,Carbuncle‬מי מההיגדים הבאים נכון‪:‬‬
‫א‪ Carbuncle .‬הוא זיהום של בלוטות הזיעה‬
‫ב‪ .‬חיידק ה‪ Pseudomonas-‬הוא הגורם האופייני‬
‫ג‪ .‬סימנים סיסטמיים כמו חום וחולשה הם שכיחים‬
‫ד‪ Glomerulonephritis .‬הוא סיבוך אפשרי‬
‫ה‪ .‬הטיפול היעיל ביותר הוא ניקוז‬
‫‪ .20‬תרופת הבחירה לטיפול של ‪ Latent TB infection‬היא‪:‬‬
‫א‪Pyrazinamide .‬‬
‫ב‪Isoniazid .‬‬
‫ג‪Ethambutol .‬‬
‫ד‪Streptomycin .‬‬
‫ה‪Rifampin .‬‬

‫‪ .21‬מה נכון לגבי אזולים )טיפול לפטריות(?‬


‫א‪ .‬פלוקונזול יעיל כנגד קריפטוקוקוס‪ ,‬קנדידה ומוקור‬
‫ב‪ .‬פלוקנזול לא חודר טוב ל‪CSF‬‬
‫ג‪ voriconazole .‬היא טיפול הבחירה לאספרגילוס אינבזיבי )‪(invasive‬‬
‫ד‪ itraconazole .‬קיים רק למתן תוך ורידי‬
‫ה‪ .‬פלוקנזול הינו גורם שכיח לאי ספיקת כליות‬

‫‪ .22‬מנגנון הפעולה של פניצילינים משבש את‪:‬‬


‫א‪ .‬ייצור חומצות גרעין‬
‫ב‪ .‬ייצור של קיר )דופן( החיידק‬
‫ג‪ .‬ייצור של חלבונים‬
‫ד‪ .‬מטבוליזם של חומצה פולית‬
‫ה‪ .‬אף אחד מהנ"ל‬

‫אנדוקרינולוגיה‬
‫‪ .23‬כל המצבים הבאים עלולים לגרום להיפרפרולקטינמיה פרט ל‪:‬‬
‫א‪ .‬מצבי דחק ‪ -‬סטרס‬
‫ב‪ .‬תת פעילות ראשונית של בלוטת התריס‬
‫ג‪ .‬אי ספיקת כליות כרונית‬
‫ד‪ .‬הרפס זוסטר בדופן בית החזה‬
‫ה‪ .‬טיפול בקברגולין‬

‫‪ .24‬כל הבאים נכונים לגבי ‪ panhypopituitarism‬פרט ל‪:‬‬


‫א‪ .‬החולים סובלים מחולשה וחיוורון‬
‫ב‪ TSH .‬הינה בדיקת הבחירה במעקב אחר איזון תפקודי בלוטת התריס בחולה הסובל‬
‫מהיפופיטואיטריזם‬
‫ג‪ .‬בד"כ הורמון הגדילה הוא שנפגע ראשון התהליך הרס ההיפופיזה‬
‫ד‪ .‬חולים עלולים לפתח קומה במצבי דחק )סטרס( עם היפוגליקמיה‪ ,‬היפונתרמיה‬
‫והיפותרמיה‬
‫ה‪ .‬עלול להופיע איבוד שיער גוף‬

‫‪ .25‬מה נכון לגבי היפרתיירואידיזם?‬


‫א‪ .‬מחלת ‪ Graves‬היא סיבה נדירה להיפרתיירואידיזם‬
‫ב‪ .‬טיפול ‪ Mercaptizol‬הוא טיפול הבחירה ב ‪subacute thyroiditis -‬‬
‫ג‪ .‬חולשה ניכרת‪ ,‬אפתיה‪ ,‬הפרעות קצב ואוסטאופורוזיס הן ביטוי שכיח של המחלה‬
‫בקשישים‬
‫ד‪ .‬טיפול ב‪ I-131-‬אפשרי בטרימסטר האחרון להריון אצל נשים שנאלצו להפסיק טיפול‬
‫בגלל תופעות לוואי של ‪anti-thyroid drugs‬‬
‫ה‪ .‬טיפול ניתוחי הוא טיפול הבחירה בילדים חולי ‪ Graves‬שלא פיתחו רמיסיה תחת טיפול‬
‫ב ‪anti-thyroid drugs -‬‬
‫‪ .26‬חולה בן ‪ 77‬אובחן כסובל מסכרת מסוג ‪ ,2‬החל בטיפול בשילוב של ‪ Metformin‬ו‪-‬‬
‫‪ Glibenclamide‬במינונים מקסימאליים‪ .‬החולה מתלונן על חולשה‪ ,‬סחרחורות‪ .‬רמת הסוכר‬
‫שנבדקה בעת שהרגיש סחרחורת הייתה ‪ .55mg/dL‬מה נכון לעשות ?‬
‫א‪ .‬להפסיק את כל הטיפול‬
‫ב‪ .‬להפסיק ‪Metformin‬‬
‫ג‪ .‬לעבור לאינסולין‬
‫ד‪ .‬לא לשנות את הטיפול‪ ,‬להמליץ על ארוחות תכופות יותר‬
‫ה‪ .‬להפסיק ‪Glibenclamide‬‬

‫‪ .27‬בן ‪ 50‬אובחן לראשונה כחולה סכרת סוג ‪ .HbA1C - 8% .2‬מה צריך לכלול הבירור הראשוני?‬
‫א‪ .‬מדידת ל"ד וביצוע אק"ג‬
‫ב‪ .‬בדיקת קרקעיות העיניים‬
‫ג‪ .‬בדיקת מיקרואלבומין בשתן‬
‫ד‪ .‬בדיקת דפקים פריפריים ותחושה בגפיים‬
‫ה‪ .‬כל התשובות נכונות‬

‫‪ .28‬בת ‪ 65‬ללא סיפור קודם של סכרת‪ ,‬מופיעה בחדר מיון בהכרה מעורפלת‪ ,‬מיובשת‪ ,‬עם חום‬
‫‪ ,39°‬עם רמת סוכר בדם של ‪ .680mg/dL‬בצילום רנטגן של בית החזה אובחנה דלקת ריאות‪.‬‬
‫מה נכון לגבי חולה זאת‪:‬‬
‫א‪ .‬לחולה ‪ Hyperosmolar non-ketotic state‬כתוצאה מסכרת מסוג ‪ 2‬שלא אובחנה קודם‬
‫לכן‬
‫ב‪ .‬לחולה דלקת ריאות קשה‪ .‬העליה בסוכר נובעת מהתייבשות וחסרת משמעות קלינית‬
‫ג‪ .‬החולה זקוקה למתן נוזלים‪ ,‬אינסולין וטיפול אנטיביוטי‬
‫ד‪ .‬החולה זקוקה למתן נוזלים וטיפול אנטיביוטי‬
‫ה‪ .‬א‪+‬ג נכונות‬

‫‪ .29‬נער בן ‪ 12‬התמוטט באמצע אימון‪ .‬אימו מוסרת שלאחרונה התלונן על חולשה ברגליים‪ .‬לאחיו‬
‫בעיה נוירולוגית‪ .‬בבדיקה‪ :‬ל"ד ‪ ,70/50‬בעור היפרפיגמנטציה באזור המפרקים‪ ,‬אין ויטיליגו‪.‬‬
‫בבדיקת דם רמת טסטוסטרון נמוכה‪ .‬איזו מהבדיקות הבאות עלולה להיות לא תקינה‪:‬‬
‫א‪ .‬דם ל‪Adrenal antibodies-‬‬
‫ב‪ .‬דם ל‪very long chain fatty acids-‬‬
‫ג‪ CT .‬אדרנל‬
‫ד‪ MRI .‬היפופיזה‬
‫ה‪ CT .‬בית חזה‬

‫‪ .30‬באיזה מהמצבים הבאים קיים סיכון גבוה במיוחד להתייבשות והפרעה באלקטרוליטים ?‬
‫א‪Non classical 21-hydroxylase deficiency .‬‬
‫ב‪Secondary adrenal insufficiency .‬‬
‫ג‪MEN2b .‬‬
‫ד‪Gastroenteritis .‬‬
‫ה‪MEN1 .‬‬

‫‪ .31‬חמש שעות לאחר התחלת הטיפול בילד עם קטואצידוזיס סוכרתית הילד שוקע והכרתו‬
‫מעורפלת‪ .‬בדיקות המעבדה שבוצעו מראות על‪ :‬קטונים בשתן ‪Glucose- ,PH-7.25 ,+++‬‬
‫‪ ,Na-134 ,K-2.9 ,60‬ביקרבונט‪ .16-‬הסיבה הסבירה ביותר למצבו הקליני‪:‬‬
‫א‪ .‬היפוגליקמיה קלינית בשל הורדה מהירה של רמת הסוכר‬
‫ב‪ .‬מצב זיהומי ספטי‬
‫ג‪ .‬מצקת מח‬
‫ד‪ .‬הפרעה באלקטרוליטים‬
‫ה‪ .‬שימוש באינסולין מסוג ‪ NPH‬במקום אינסולין רגיל‬
‫‪ .32‬ילד בן ‪ 5‬שנים מופנה למיון בשל סכרת חדשה‪ .‬איזה מבין הסימפטומים אינו מתאים‬
‫לאבחנה‪:‬‬
‫א‪ .‬ירידה במשקל‬
‫ב‪ .‬חוסר תיאבון‬
‫ג‪ .‬נוקטוריה‬
‫ד‪ .‬צימאון‬
‫ה‪ .‬פוליאוריה‬

‫‪ .33‬הורים לילד עם סכרת מסוג ‪ 1‬מודאגים לגבי הסיכון לפתח סכרת אצל אשר הילדים במשפחה‪.‬‬
‫איזה מבין העובדות נכונה ?‬
‫א‪ .‬סכרת מסוג ‪ 1‬הינה מחלה תורשתית המועברת בצורה אוטוזומלית רצסיבית‬
‫ב‪ HLA-DR2 .‬מופיע בשכיחות יתר בחולים אלה‬
‫ג‪ .‬המחלה שכיחה יותר אצל בנות לעומת בנים‪ ,‬כמו ברב המחלות האוטואימוניות‪.‬‬
‫ד‪ .‬כאשר אחד מתאומים זהים מפתח את המחלה הסיכון שהתאום השני יפתח את המחלה‬
‫הוא רק ‪ ,40%‬מה שמעיד על חשיבות הגורם הסביבתי‪.‬‬
‫ה‪ .‬מתן חיסונים נגד זיהומים ויראליים מגן מפני התפתחות המחלה‪.‬‬

‫‪ .34‬חולה מגיע לחדר מיון עם רמת סידן בסרום של ‪ ,16mg/dL‬אלבומין ‪ ,4‬פוספור ‪,2.5‬‬
‫קריאטינין ‪ ,1‬מצא את הצרוף הטיפולי המתאים ביותר לחולה זה‪:‬‬
‫א‪ .‬מתן תוך ורידי של ביספוספונטים‬
‫ב‪ .‬מתן ‪ 4-6L‬סליין נורמאלי ליממה‪ Furosemide ,‬ואח"כ ביספוספונטים ‪IV‬‬
‫ג‪ .‬מתן ‪ Furosemide‬תוך ורידי‬
‫ד‪ .‬מתן ‪ Normal saline ,Disothiazide‬וביספוספונטים ‪IV‬‬
‫ה‪ .‬מתן ‪ ,Normal saline‬גלוקוקורטיקואידים ו‪Furosemide-‬‬

‫‪ .35‬אישה בת ‪ ,60‬בד"כ בריאה‪ ,‬מעולם לא סבלה משברים‪ ,‬במנופאוזה מזה ‪ 8‬שנים‪ ,‬עדיין סובלת‬
‫מגלי חום‪ ,‬מעשנת ‪ 10‬סיגריות ביום‪ ,‬אינה צורכת מוצרי חלב‪ ,‬עברה בדיקת צפיפות עצם‬
‫במכשיר ‪ .DEXA‬בבדיקה ‪ -‬צפיפות העצם בעמוד השדרה וצוואר הירך בערכי ‪ T score‬הינה‬
‫)‪ .(-2.8‬מה תעשה?‬
‫א‪ .‬אסביר לחולה שהיא סובלת מאוסטאופורוזיס ונמצאת בסיכון מוגבר לשברים‪ .‬עליה‬
‫להפסיק לעשן ולהקפיד על פעילות גופנית סדירה‪ .‬בשלב זה אינה זקוקה לטיפול תרופתי‬
‫ב‪ .‬אמליץ להפסיק לעשן ולהתחיל טיפול ב‪Alendronate -‬‬
‫ג‪ .‬אמליץ להפסיק לעשן ולהתחיל טיפול ב‪Raloxifen -‬‬
‫ד‪ .‬אמליץ על מתן תוספי סידן‪ ,‬ויטמין ‪,D‬ו‪Alendronate -‬‬
‫ה‪ .‬אמליץ על ביצוע בדיקת צפיפות עצם לביתה בת ה‪30-‬‬

‫‪ .36‬באיזה מהמצבים הבאים לא תעלה שכיחות ה ‪? Carpal tunnel syndrome‬‬


‫א‪ .‬חולי אקרומגליה‬
‫ב‪ .‬הריון‬
‫ג‪ .‬חולים הסובלים מתת‪-‬פעילות בלוטת התריס‬
‫ד‪ .‬חולי סכרת‬
‫ה‪ .‬חולים עם תסמונת קושינג‬

‫‪ .37‬הסיבה השכיחה ביותר להיפרתיירואידיזם ראשוני ?‬


‫א‪parathyroid adenoma .‬‬
‫ב‪parathyroid hyperplasia .‬‬
‫ג‪parathyroid carcinoma .‬‬
‫ד‪ parathyroid adenoma .‬ו‪ parathyroid hyperplasia-‬בשכיחות זהה‬
‫‪ .38‬האבחנה של ‪ Minimally invasive follicular carcinoma‬לעומת ‪Follicular adenoma‬‬
‫בבלוטת התריס תלויה ב‪:‬‬
‫א‪ .‬חדירה לקופסית ו\או לכלי דם‬
‫ב‪ .‬מיטוזות ופליאומורפיזם‬
‫גודל הפוליקולים‬ ‫ג‪.‬‬
‫ד‪ .‬נוכחות של ‪hurtle cells‬‬

‫‪ .39‬מה נכון לגבי ‪? subacute granulomatous thyroiditis‬‬


‫א‪ .‬מחלה אוטואימונית‬
‫ב‪ .‬מחלה מופיעה לאחר זיהום ויראלי‬
‫מחלה המופיעה לאחר לידה‬ ‫ג‪.‬‬
‫ד‪ .‬התאים הפוליקולריים נוטים לעבור שינוי והם בעלי ציטופלזמה אצידופילית‬

‫‪ .40‬לגבי המחלה בתמונה מספר ‪) 2‬ראה תמונה בסוף הבחינה( ‪ -‬תסנין דלקתי מונונוקלארי באיים‬
‫על שם לנגנהאנס‪ ,‬כל הבאים קשורים לפתוגנזה של המחלה פרט ל‪:‬‬
‫א‪autoimmunity .‬‬
‫‪genetic susceptibility‬‬ ‫ב‪.‬‬
‫‪insulin resistance‬‬ ‫ג‪.‬‬
‫‪environmental insult‬‬ ‫ד‪.‬‬
‫‪ .41‬מה נכון לגבי תמונה ‪:3‬‬
‫א‪ .‬הממצא מופיע בשלבים מוקדמים של המחלה ונעלם אחר‪-‬כך‬
‫ב‪ .‬תמונה אופיינית לסכרת סוג ‪2‬‬
‫ג‪ .‬לפתוגנזה של המחלה יש קשר הדוק עם ‪HLADR3-4‬‬
‫ד‪ .‬להשמנת יתר אין קשר לסוג זה של סכרת‬

‫‪ .42‬מה הסיבה השכיחה ל‪? Inappropriate ADH secretion-‬‬


‫א‪ .‬מנינגיטיס‬
‫ב‪ .‬אדנומה בהיפופיזה‬
‫ג‪ .‬קרניופרינגיומה‬
‫ד‪ .‬גליומה בהיפותלמוס‬
‫ה‪ .‬אף אחד מהנ"ל‬

‫‪ .43‬מה נכון לגבי הסיבות להיפופיטואיטריזם ?‬


‫א‪ .‬אדנומה לא מפרישה‬
‫ב‪ .‬גרורה ב‪Sella turcica-‬‬
‫ג‪Pituitary Apoplexy .‬‬
‫ד‪ .‬קרניופרינגיומה‬
‫ה‪ .‬כולם נכונה‬

‫‪ .44‬לגבי פאוכרומוציטומה‪ ,‬הכל נכון פרט ל‪:‬‬


‫א‪ .‬רובם מתחילים במדולה של יותרת הכליה‬
‫ב‪ .‬מפריש קטכולאמינים‬
‫ג‪ .‬רובם ממאירים‬
‫ד‪ .‬גידול עשיר בכלי דם‬
‫ה‪ 10% .‬קשורים לסינדרום ‪MEN‬‬
‫‪ .45‬לגבי ‪ ,Conn syndrome‬הכל נכון פרט ל‪:‬‬
‫א‪ .‬היפראלדוסטרוניזם‬
‫ב‪ .‬עליה בלחץ דם‬
‫ג‪ .‬הסיבה יכולה להיות אדנומה באדרנל‬
‫ד‪ .‬הסיבה יכולה להיות אדנומה של ההיפופיזה‬
‫ה‪ .‬רמה נמוכה של ‪Renin‬‬

‫‪ .46‬לפי התמונה ההיסטולוגית של סלייד מס ‪ ,1‬סמן את התשובה הנכונה ?‬


‫א‪ .‬מדובר ב‪Cortical adenoma-‬‬
‫ב‪ .‬מדובר ב‪Cortical carcinoma-‬‬
‫ג‪ .‬מדובר ב‪Invasive adenoma-‬‬
‫ד‪ .‬החולה סובל מ‪CONN syndrome-‬‬
‫ה‪ .‬אף תשובה אינה נכונה‬

‫המטואונקולוגיה‬
‫‪ .47‬מהם הבדלים העיקריים בבדיקה ראשונית בפיזור המחלה בין ‪(HD) Hodgkin's lymphoma‬‬
‫ל‪? (NHL) Non Hodgkin's lymphoma -‬‬
‫א‪ .‬ב‪ NHL-‬המחלה מתייצגת בבלוטות בבית החזה באחוזים ניכרים יותר ובבלוטות‬
‫מזנטריליות באחוזים פחותים יותר מאשר ‪HD‬‬
‫ב‪ .‬ב‪ HD-‬המחלה המתייצגת בפיזור ניכר במערכת השתן לעומת ‪NHL‬‬
‫ג‪ .‬ב‪ HD-‬פיזור משמעותי במערכת העיכול לעומת ‪NHL‬‬
‫ד‪ .‬ב‪ HD-‬המחלה מתייצגת בבלוטות בבית החזה באחוזים ניכרים יותר ובבלוטות‬
‫מזנטריליות באחוזים פחותים יותר מאשר ‪NHL‬‬

‫‪ .48‬אילו מהמשפטים הבאים נכון ?‬


‫א‪ .‬חייבות להתרחש מוטציות בשני האללים של פרוטו‪-‬אונקוגן‪ ,‬על מנת שיתרום לתהליך‬
‫הסרטני‬
‫ב‪ .‬חלבון ‪ P53‬מעורר אפופטוזיס בתאים בכל מקרה בו נגרם נזק ל‪DNA-‬‬
‫ג‪ .‬טרנסלוקציה )‪ (8;21‬ו‪ Inversion 16-‬הקיימת ב‪ AML-‬פוגעת בתפקוד של אותו מסלול‬
‫המטופויטי‬
‫ד‪ .‬בלימפומות קיימות טרנסלוקציות הגורמות להיווצרות חלבון כימרי אונקוגני‬
‫ה‪ .‬התרופה ‪ ATRA‬הניתנת לחולי ‪ APL‬מעוררת אפופטוזיס ע"י שחרור ‪Cyt-C‬‬
‫מהמיטוכונדריה‬

‫‪ .49‬גבר בן ‪ 65‬התייצג עם בלוטות לימפה שנמושו בצוואר תחתון מימין ובבית שחי ימין‪ .‬ביופסיה‬
‫מהבלוטה בצוואר הראתה שמדובר ב‪ .Diffuse large cell B cell lymphoma-‬מה נכון לגבי‬
‫המקרה שלפנינו ?‬
‫א‪ .‬מומלץ לבצע ‪ CT‬של הצוואר ובית החזה במסגרת העיבוד של החולה לקביעת דרגת‬
‫המחלה‪ .‬אין צורך לבצע ‪ CT‬בטן‪/‬אגן‪ ,‬כי הסיכוי למעורבות של בלוטות לימפה ביטניות‬
‫הינו קטן‪.‬‬
‫ב‪ .‬קיים סיכוי רב שבבדיקת ביופסית לשד עצם תמצא הסננה של הלימפומה במח העצם‪.‬‬
‫ג‪ .‬במידה והמחלה תשאר בסוף הבדיקות בשלב מוקדם והחולה אסימפטומטי ‪ -‬ניתן לטפל‬
‫בקרינה בלבד לאזור הצוואר ובית השחי‪.‬‬
‫ד‪ .‬גם לאחר השגת תגובה מלאה לאחר טיפול בכמותרפיה ‪ CHOP‬יחד עם מבטרה‪ ,‬קיים‬
‫סיכוי של מעל ‪ 50%‬לחזרת המחלה לאורך השנים‬
‫ה‪ .‬לו היה החולה צעיר יותר היה לו סיכוי טוב יותר להשיג ריפוי ע"י הטיפול בכמותרפיה‬
‫‪ + CHOP‬מבטרה‬
‫‪ .50‬איזו מהתרופות הבאות תתאים כקו טיפול ראשוני לחולה בת ‪ 3‬עם סרטן גרורתי לשלד ?‬
‫א‪(Arimidex) Anastrozole .‬‬
‫ב‪Tamoxifen .‬‬
‫ג‪(Femora) Letrozole .‬‬
‫ד‪(Aromasin) Exemestane .‬‬
‫‪ .51‬כל האמצעים הבאים מהווים חלק מטיפול במתוטרקסאט במנות גבוהות‪ ,‬פרט ל‪:‬‬
‫א‪ .‬מתן )‪Filgrastin (Neupogen‬‬
‫ב‪ .‬מתן )‪5-formyltetrahydrofolate (Leucovorin‬‬
‫ג‪ .‬מתן סודיום ביקרבונט דרך הוריד‬
‫ד‪ .‬מתן נוזליםלתוך הוריד בכמות גדולה‬

‫‪ .52‬גבר בן ‪ 58‬סיים קו שני של כמותרפיה‪ ,‬ומגיע לביקור אצל האונקולוג עם תוצאות בדיקת‬
‫צילומי ה‪ CT-‬במעטפה‪ .‬הוא לא פותח את המעטפה כי הוא מפחד מאוד‪ .‬המטופל מדגיששעד‬
‫כה התמודד עם המחלה‪ ,‬אבל לאחרונה קשה לו רגשית‪ ,‬מתקשה להרדם‪ ,‬פוחד מהמוות‪.‬‬
‫כרופא ניתן להמליץ לו על כל הבאים פרט ל‪:‬‬
‫א‪ .‬להציע לפתוח את המעטפה בנוכחות הרופא המטפל‬
‫ב‪ .‬להפנות לפסיכיאטר לטיפול תרופתי בחרדה ולאור הסיכון האובדני‬
‫ג‪ .‬המטופל סובל מתגובת חרדה ויש להפנותו לפסיכולוג‬
‫ד‪ .‬לנסות להרגיעו‪ ,‬מכיוון שיש קשר בין חרדה אקוטית לדיכוי חיסוני‬

‫‪ .53‬בן ‪ 65‬מתלונן על חולשה וכאבי גב‪ .‬בבדיקה אנמיה נורמוציטית נורמוכרומית ושקיעת דם‬
‫מוחשת‪ .‬כימיה תקינה‪ .‬בדיקות פרוטאין אלקטרופורזה ואימונואלקטרופורזה ושתן ל‪Bence -‬‬
‫‪ Jones‬תקינות‪ .‬מה נכון ?‬
‫א‪ .‬ניתן לשלול בוודאות שמדובר ב‪Multiple myeloma-‬‬
‫ב‪ .‬על מנת לשלול מיילומה יש לבצע בדיקת לשד עצם‬
‫ג‪ .‬צריך לעשות מיפוי עצמות לבירור נגעים ליטיים‬
‫ד‪ 20% .‬מהחולים מתייצגים עם ‪Non-secretory myeloma‬‬

‫‪ .54‬בן ‪ ,48‬חום ‪ 38.8‬בערב ובלילה‪ ,‬הזעות לילה המחייבות החלפת בגד שינה‪ ,‬בלוטות בצוואר ימין‪,‬‬
‫בית שחי שמאל וימין‪ .‬בביופסיה – ‪ .Classical Hodgkin nodular sclerosis‬ב‪– PET/CT-‬‬
‫קליטה בצוואר‪ ,‬בית שחי‪ ,‬בלוטות במזנטריום ובשער הכבד‪ .‬שקיעת דם ‪ ,70‬ספירת דם –‬
‫לויקוציטוזיס ‪ ,17,000‬המוגלובין ‪ ,8.5‬אלבומין ‪ 3.2‬היתר תקין‪ .‬החולה בשלב‪:‬‬
‫א‪IIb .‬‬
‫ב‪IIIb .‬‬
‫ג‪IVb .‬‬
‫ד‪IIIa .‬‬
‫‪ .55‬הטיפול המומלץ לחולה זה‪:‬‬
‫א‪ .‬כמותרפיה והקרנות‬
‫ב‪ .‬כמותרפיה בלבד‬
‫ג‪ .‬השתלת מח עצם עצמית כחלק מקו ראשון‬
‫ד‪ .‬קרינה בלבד‬

‫‪ .56‬מה סיכויי ההחלמה של גבר בן ‪ 68‬אשר אובחן כסובל מ‪Complex karyotype acute -‬‬
‫‪? myeloid leukemia‬‬
‫א‪10% .‬‬
‫ב‪30% .‬‬
‫ג‪50% .‬‬
‫ד‪85% .‬‬
‫‪ .57‬מה הטיפול המועדף בחולה בן ‪ 25‬אשר אובחן כסובל מ‪ AML-‬ובבדיקות ציטוגנטיות נמצא‬
‫חסר בכרומוזום ‪? 7‬‬
‫א‪ .‬כמותרפיה להשגת הפוגה ובהמשך קורסים כמותרפיים למיצוק ההפוגה‬
‫ב‪ .‬כמותרפיה להשגת הפוגה והשתלת מח עצם מתורם‬
‫ג‪ .‬ל‪-‬א' ו‪-‬ב' אותה תועלת בדיוק‪ ,‬כך שניתן לשקול עפ"י העדפת החולה‬
‫ד‪ .‬כמותרפיה להשגת הפוגה‪ ,‬ואם מושגת הפוגה אין צורך בטיפול נוסף‬

‫‪ .58‬שתי השאלות מתייחסות לנתונים הבאים‪:‬‬


‫גבר בן ‪ 80‬עם מחלת לב איסכמית וסוכרת ברקע‪ ,‬התקבל לברור חולשה‪ .‬בבדיקה –‬
‫לימפאדנופתיה בכל התחנות וטחול מוגדל‪ .‬מעבדה‪WBC- ,Hb-9.9gr% ,MCV-88 :‬‬
‫‪ ,PLT-100,000 ,62,000‬במשטח דם תמונה של ‪ .CLL‬האבחנה היא‪:‬‬
‫א‪ALL .‬‬
‫ב‪CLM .‬‬
‫ג‪CLL .‬‬
‫ד‪DLBCL .‬‬
‫‪ .59‬הטיפול המתאים לחולה הנ"ל הוא‪:‬‬
‫א‪ .‬כלורמבוציל ופרדניזון‬
‫ב‪CHOP .‬‬
‫ג‪ .‬תשלובת ‪7:3‬‬
‫ד‪Cyclophosphamide + Fludarabine .‬‬
‫ה‪ .‬קרינה לאתר המחלה‬
‫‪ .60‬סיבה לאנמיה בחלים אונקולוגיים יכולה להיות‪:‬‬
‫א‪ .‬הסננת מח עצם‬
‫ב‪ .‬דיכוי מח עצם על‪-‬ידי כמותרפיה‬
‫ג‪ .‬דימום‬
‫ד‪ .‬מטבוליזם לא תקין של ברזל‬
‫ה‪ .‬כל הנ"ל‬
‫‪ .61‬מטרת הטיפול ב‪ EPO-‬אצל חולים אונקולוגיים עם אנמיה סימפטומטית היא‪:‬‬
‫א‪ .‬להעלות את רמת ההמוגלובין מעל ‪7.5gr%‬‬
‫ב‪ .‬לאפשר עליה במינון כמותרפיה‬
‫ג‪ .‬לשפר את איכות החיים ולהפחית צורך במתן דם‬
‫ד‪ .‬למנוע זיהומים חוזרים‬
‫‪ .62‬חולה בן ‪ 64‬הופנה לחדר מיון בשל כאביי ראש‪ ,‬קושי בדיבור‪ ,‬שלשולים וכאבי בטן‪ .‬בבדיקות‬
‫נמוש קצה טחול‪ .‬בבדיקות מעבדה‪ 3%) WBC-14,000 ,Hb-19gr% ,Hct-57% :‬מונוציטים‪,‬‬
‫‪ 2%‬אאוזינופילים‪ ,‬ו‪ 15%-‬לימפוציטים(‪ .PLT-700,000 ,‬במשטח דם ‪ 30%‬סגמנטים‪ .‬איזו‬
‫מהמחלות הבאות תבוא בחשבון באבחנה המבדלת ?‬
‫א‪ .‬תסמונת מיאלודיספלסטית‬
‫ב‪ .‬מיאלופיברוזיס‬
‫ג‪ .‬פוליציטמיה ורה‬
‫ד‪Essential thrombocytopenia .‬‬
‫ה‪ .‬לויקמיה מיאלואידית כרונית‬

‫‪ .63‬כדי לבסס את האבחנה יש לבצע את הבדיקות הבאות‪ ,‬פרט ל‪:‬‬


‫א‪ .‬נפח דם‬
‫ב‪ .‬בדיקות לנוכחות המוטציה ‪JAK2‬‬
‫ג‪ .‬ריווי חמצן עורקי‬
‫ד‪ .‬בדיקת קריוטיפ ונוכחות ‪5q‬‬
‫ה‪ .‬רמת ויטמין ‪B12‬‬
‫‪ .64‬חולים עם ‪ Myelofibrosis‬סובלים בד"כ מ‪:‬‬
‫א‪ .‬אנמיה‬
‫ב‪ .‬ספלנומגליה‬
‫ג‪ .‬נוכחות בלסטים בודדים בדם היקפי‬
‫ד‪ .‬כל הנ"ל‬
‫ה‪ .‬אף אחד מהנ"ל‬

‫‪ .65‬הגורמים הקובעים את הפרוגנוזה בתסמונת מיאלודיספלסטית כוללים את‪:‬‬


‫א‪ .‬גיל החולה‬
‫ב‪ .‬מספר בלסטים בדם היקפי‬
‫ג‪Normal or complex chromosomal abnormality .‬‬
‫ד‪ .‬המצאות תורם מתאים להשתלת מח עצם‬
‫ה‪ .‬כל התשובות נכונות‬

‫‪ .66‬מנת הקרינה לגידול נקבעת בעיקר לפי‪:‬‬


‫א‪ .‬גיל החולה‬
‫ב‪ .‬מצבו הרפואי הכולל‬
‫ג‪ .‬סוג הגידול‬
‫ד‪ .‬מיקום הגידול‬
‫ה‪ .‬רמת המוגלובין בדם‬

‫‪ .67‬במסגרת של ברור אנמיה אצל אישה בת ‪ 59‬בוצעה ביופסית לשד עצם )‪ .(BMB‬בחתכים‬
‫שנבצעו ב‪ H&E-‬נמצא תסנין שאתי בנוי מתאי לימפה קטנים שנצבעו ‪ CD20‬חיובי‪ .‬מה‬
‫הצביעות ההיסטוכימיות ברשימה הבאה הדרושות לאבחנה ראשונית ?‬
‫א‪CD3, CD5 .‬‬
‫ב‪CD5, CD10 .‬‬
‫ג‪CD45, CD30 .‬‬
‫ד‪CD4, CD8 .‬‬

‫‪ .68‬איזו לימפומה מאופיינת ע"י ממצא היסטולוגי שנקרא ‪? Lymphoepithelial lesion‬‬


‫א‪Small lymphocytic lymphoma .‬‬
‫ב‪Mantle cell lymphoma .‬‬
‫ג‪MALT lymphoma .‬‬
‫ד‪Mycosis fungoides .‬‬
‫ה‪Follicular lymphoma .‬‬

‫‪ .69‬מה הסטייה הציטוגנטית שמאפיינת את ‪? Burkitt's lymphoma‬‬


‫א‪Bcl-Abl fusion protein .‬‬
‫ב‪C-myc oncogene .‬‬
‫ג‪ALK fusion protein .‬‬
‫ד‪Cyclin D1 protein over expression .‬‬

‫‪ .70‬סלייד מס' ‪ -2‬הנגע שבסלייד קרוב לוודאי יראה צביעה ל‪(DLBCL) :‬‬
‫א‪ CD30 .‬ו‪CD15-‬‬
‫ב‪ CD20 .‬ו‪Ki67 -‬‬
‫ג‪ CD3 .‬ו‪ Ki67-‬נמוך‬
‫ד‪ .‬מיילופרוקסידאז‬
‫‪ .71‬המבנה שנקרא ”‪:“starry sky‬‬
‫א‪ .‬אבחנתי ל‪acute lymphoblastic lymphoma-‬‬
‫ב‪ .‬אבחנתי ל‪diffuse large-B cell lymphoma-‬‬
‫ג‪ .‬אבחנתי ל‪burkitt’s lymphoma-‬‬
‫ד‪ .‬הממצא אינו אבחנתי )אינו פתוגנומוני(‬

‫‪ .72‬אישה בת ‪ 57‬שמה לב לפני כמה חודשים שחבלות קלות גורמות לה דימומים בולטים בעור‪.‬‬
‫בבדיקה גופנית‪ ,‬בנוסף לשטפי דם רבים בעור בכל חלקי גופה‪ ,‬נמצא טחול מוגדל מאוד‪ .‬אין‬
‫חום‪ .‬תפקודי קרישה בגדר התקין‪ .‬בבדיקת דם היקפי נמצאו‪ :‬טסיות‪ ,850,000-‬תאים לבנים‪-‬‬
‫‪ ,21,000‬המטוקריט‪ ,48.2%-‬המוגלובין‪ 16.8-‬ג'\דצ"ל )עלייה בכל שורות התאים(‪ .‬תמונה מס'‬
‫‪ 5‬מראה את הביופסיה של לשד העצם‪ .‬מה האבחנה?‬
‫א‪acute lymphoid leukemia .‬‬
‫ב‪chronic lymphocytic leukemia .‬‬
‫ג‪chronic myeloproliferative disease .‬‬
‫ד‪leukemoid reaction .‬‬
‫‪ .73‬ילדה בת ‪ 5‬מראה גוש בלסת התחתונה הימנית שגדל מהר תוך חודשיים‪ .‬הביופסיה מהגוש‬
‫)תמונה מס' ‪ (6‬מראה תסנין מפושט של תאים קטנים עד בינוניים אחידים יחסית‪ .‬התאים‬
‫נצבעו ל‪ CD20/CD10/Bcl6-‬חיובי‪ ,‬עם אינדקס פרוליפרציה של ‪ .100%‬איזה נגיף קרוב‬
‫לוודאי קשור להתפתחות הגידול?‬
‫א‪ .‬אדנווירוס‬
‫ב‪ .‬הפטיטיס ‪B‬‬
‫ג‪ .‬הרפס וירוס ‪8‬‬
‫ד‪HIV .‬‬
‫ה‪EBV .‬‬

‫‪ .74‬כל הבאים הם סוגים של ‪) CHL‬הודג'קינס קלאסי( פרט ל‪:‬‬


‫א‪Mixed cellularity .‬‬
‫ב‪Nodular sclerosis .‬‬
‫ג‪Nodular lymphocytic predominance .‬‬
‫ד‪Lymphocyte depleted .‬‬
‫ה‪Lymphocyte rich .‬‬

‫נשים ושד‬
‫‪ .80‬מה נכון לגבי הריון אינטרסטיציאלי ?‬
‫א‪ .‬אחראי ל‪ 1-3%-‬מההריונות החצוצרתיים‬
‫ב‪ .‬לעיתים קרובות פורץ מאוחר )‪ 8-16‬שבועות הריון(‬
‫ג‪ .‬מלווה בדימום רב כשפורץ‬
‫ד‪ .‬במצבים מסוימים כריתת רחם היא טיפול הבחירה‬
‫ה‪ .‬הכל נכון‬

‫‪ .81‬סיבה שכיחה להריון הטרוטרופי‪:‬‬


‫א‪ .‬השמנת יתר‬
‫ב‪ .‬הפריה חוץ גופית‬
‫ג‪ .‬התקן תוך רחמי‬
‫ד‪ .‬ניתוח קיסרי קודם‬
‫ה‪ .‬דלקת באגן‬
‫‪ .82‬כל השיטות מקובלות לאבחון הריון חוץ רחמי‪ ,‬חוץ מ‪:‬‬
‫א‪ .‬בדיקת ‪β-HCG‬‬
‫ב‪ .‬דיקור אבחנתי של דגלאס‬
‫ג‪ .‬בדיקה אולטראסונוגרפית‬
‫ד‪ .‬לפרוסקופיה‬
‫ה‪ .‬רמת רטיקולוציטים בספירת דם‬

‫‪ .83‬בת ‪ ,26‬נשואה‪ , 2+‬בעלת התקן תוך רחמי‪ ,‬סובלת מזה חודשיים מכאבים בבטן תחתונה‪ ,‬ללא‬
‫תלונות גסטרואינטסטינליות או אורינריות‪ ,‬חום ‪ ,39°‬רגישות בהנעת צוואר הרחם‪ .‬מה‬
‫הסיבה הפחות סבירה ?‬
‫א‪ .‬דלקת תוספתן‬
‫ב‪ .‬הפלה בלתי שלמה‬
‫ג‪PID .‬‬
‫ד‪ .‬הריון חוץ רחמי‬

‫‪ .84‬טיפול ראשוני ל‪:PID-‬‬


‫א‪ .‬אנטיביוטי משולב‬
‫ב‪ .‬לפרוסקופיה ‪ +‬ניקוז אבצס ‪ +‬כיסוי אנטיביוטי‬
‫ג‪ .‬לפרוסקופיה ‪ +‬ניקוז ‪ +‬כיסוי אנטיביוטי בהתאם לתרבית‬
‫ד‪ .‬טיפול אנטיביוטי לוריד לאחר שהתקבלה תשובת תרבית‬

‫‪ .85‬בכדי לדעת האם התרחש ביוץ‪,‬במסגרת בירור פוריות‪,‬יומלץ לאישה‪:‬‬


‫א‪ .‬לבדוק ‪ LH‬בשתן‬
‫לבדוק פרוגסטרון בשלב פוליקולרי‬ ‫ב‪.‬‬
‫לבדוק פרוגסטרון בשלב לוטאלי‬ ‫ג‪.‬‬
‫להדגים גופיף צהוב באולטרסאונד‬ ‫ד‪.‬‬

‫‪ .86‬הפרופיל ההורמונלי בבדיקת דם בגיל המעבר יראה‪:‬‬


‫ה‪ LH .‬ו‪ FSH-‬גבוהים‬
‫‪ FSH‬ו‪ LH -‬נמוכים מאוד‬ ‫ו‪.‬‬
‫‪ FSH‬ו‪ LH -‬בתחום הנורמה‬ ‫ז‪.‬‬
‫ח‪ FSH .‬גבוה ו‪ LH-‬בתחום הנורמה‬

‫‪ .87‬נשים עם אמנוראה שלא מדממת לאחר מתן כדורים המכילים פרוגסטרון‪ ,‬האבחנה המבדלת‬
‫תכלול קרוב לודאי את כל הבאים פרט ל‪:‬‬
‫ה‪ .‬תסמונת השחלות הפוליציסטית‬
‫אישה בפוסט מנופאוזה‬ ‫ו‪.‬‬
‫ספורטאית המתאמנת לקראת מרתון‬ ‫ז‪.‬‬
‫ח‪ .‬הריון‬
‫‪ .88‬אישה עם מחזור כל ‪ 35‬יום ‪ ,‬יום הביוץ המשוער הוא‪:‬‬
‫א‪ 11 .‬יום‬
‫ב‪ 14 .‬יום‬
‫ג‪ 18 .‬יום‬
‫ד‪ 21 .‬יום‬
‫ה‪ 24 .‬יום‬
‫‪.89‬שימושי הגלולה שאינם קשורים למניעת הריון כוללים את כל הבאים פרט ל‪:‬‬
‫א‪ .‬מניעת אנדומטריוזיס‬
‫טיפול באקנה ותשעורת יתר‬ ‫ב‪.‬‬
‫טיפול בדימומים נרתיקיים לא פונקציונאליים‬ ‫ג‪.‬‬
‫טיפול בהתבגרות מינית‬ ‫ד‪.‬‬
‫‪ .90‬לגבי הטיפול של ‪ clomiphen citrate‬בכל נכון פרט ל‪:‬‬
‫שיעור הריונות מרובי העוברים הוא ‪6%-7%‬‬ ‫א‪.‬‬
‫סיכון גבוה להתפתחות תסמונת גירוי שחלתי‬ ‫ב‪.‬‬
‫לעיתים מתרחש חוסר התפתחות רירית רחם‬ ‫ג‪.‬‬
‫הפיכת ריר הזרע לעוין‬ ‫ד‪.‬‬

‫‪ .91‬גונדוטרופינים ניתנים כטיפול פוריות בכל ההוראות הבאות פרט ל‪:‬‬


‫ה‪ .‬אי פוריות ממקור בלתי מוסבר‬
‫כשלון בכניסה להריון לאחר ‪ 3-4‬מחזורי טיפול ב‪PCO‬‬ ‫ו‪.‬‬
‫מטופלות הסובלות מ‪hypo gonadotrophic hypogondism‬‬ ‫ז‪.‬‬
‫ח‪ .‬מטופלות לאר כריתה דו צדדית של החצוצרות‬

‫‪ .92‬השימושים הנוספים ב‪ GnRH agonist -‬כוללים את הבאים פרט ל‪:‬‬


‫מניעת הרס זקיקים בזמן מתן כימותרפיה לנשים בגיל הפוריות‬ ‫א‪.‬‬
‫הורדה של נפח שרירן הרחם )מיומה( לפני ביצוע הניתוח‬ ‫ב‪.‬‬
‫טיפול באיחור התבגרות מינית‬ ‫ג‪.‬‬
‫מניעת אנדומטריוזיס‬ ‫ד‪.‬‬

‫‪ .93‬שתי סיבות עיקריות לאי פוריות באשה הן‪:‬‬


‫ליקויים מכאניים בחצוצרות‪ +‬הפרעות פוריות‬ ‫ו‪.‬‬
‫ליקויים מכאניים בחצוצרות‪ +‬אנדומריוזיס‬ ‫ז‪.‬‬
‫ח‪ .‬הפרעות ביוץ ‪+‬ליקויים בצוואר הרחם‬
‫ט‪ .‬הפרעות ביוץ‪ +‬אנדומטריוזיס‬
‫אנדומטריוזיס‪+‬אי פוריות בלתי מוסברת‬ ‫י‪.‬‬

‫‪ .94‬הבדיקה המדויקת ביותר לאבחון אי פוריות מכאנית היא‪:‬‬


‫היסטרוסקופיה‬ ‫א‪.‬‬
‫לפרוסקופיה‬ ‫ב‪.‬‬
‫צילום רחם‬ ‫ג‪.‬‬
‫‪ US‬וגינאלי‬ ‫ד‪.‬‬
‫אף תשובה אינה נכונה‬ ‫ה‪.‬‬

‫‪ .95‬התפתחות נגעים טרום סרטניים בצוואר הרחם קשורה בעיקר ב‪:‬‬


‫א‪ .‬ניקוטין‪,‬נגיף של הרפס‪ ,‬קלמידיה‬
‫טוקספלסמה‪ ,‬קלמידיה‪ ,‬הרפס‬ ‫ב‪.‬‬
‫ניקוטין בלבד‬ ‫ג‪.‬‬
‫כל סוגי הפפילומות למיניהן ‪,‬קלמידיה‬ ‫ד‪.‬‬

‫‪ .96‬בת ‪ ,36‬הריון ראשון‪ ,‬הגיעה עם דימום נרתיקי וכאבי בטן‪ .‬ב‪ US-‬נשללה שליית פתח ו‪vasa -‬‬
‫‪ .previa‬כל המדדים הבאים מחייבים יילוד מיידי פרט ל‪:‬‬
‫א‪ .‬לחץ דם אימהי ‪ 88/42‬ודופק ‪ 125‬לדקה‪.‬‬
‫ב‪ .‬בניטור הדופק העוברי קיימות האטות מאוחרות והעלמות של ההשתנות‬
‫ג‪ .‬האישה בשבוע ‪31‬‬
‫ד‪ .‬המוגלובין האישה ‪ 8.5‬ובתפקודי הקרישה נמצאו סימני ‪DIC‬‬
‫‪ .97‬מה מקור המרכיבים האפיתליאליים והמזנכימליים ב‪malignant mixed mullerian tumor-‬‬
‫)|‪? (carconocarcoma‬‬
‫א‪ .‬אנדודרם‬
‫ב‪ .‬מזודרם‬
‫ג‪ .‬אקטודרם‬
‫ד‪ .‬כל התשובות נכונות‬
‫‪ .98‬סלייד מספר ‪ 3‬שייך לקבוצת ה‪:‬‬
‫א‪Superficial epithelial-stromal tumor .‬‬
‫ב‪Germ cell tumor .‬‬
‫ג‪Trophoblastic tumor .‬‬
‫ד‪Sex cord stromal tumor .‬‬
‫ה‪Metastatic ovarian tumor .‬‬

‫‪ .99‬סלייד מספר ‪ .5‬הממצא‪:‬‬


‫א‪ .‬ממאיר‬
‫ב‪ .‬שפיר‬
‫ג‪borderline .‬‬
‫ד‪metastatic .‬‬

‫‪ .100‬סמן את המשפט השגוי לגבי ‪:mammary duct ectasia‬‬


‫א‪ .‬קשורה להפרשה סמיכה בדוקטי של השד‬
‫ב‪ .‬מחלה טרום סרטנית‬
‫ג‪ .‬יכולה לגרום לפטמה שקועה ועיבוי עור ולחקות גידול סרטני‬
‫ד‪ .‬אופייני תסנין לימפופלסמוציטרי וגרנולומות מסביב לדוקטי מודלקים‬
‫ה‪ .‬מופיעה בנשים מבוגרות )‪(60-70‬‬

‫‪ .101‬סמן את המשפט השגוי לגבי ‪:Lobar carcinoma in situ‬‬


‫א‪ .‬בד"כ ממצא מיקרי ולא גורם לגוש נמוש בשד‬
‫ב‪ .‬בד"כ לא רואים הסתיידויות‪ ,‬ולכן לעיתים לא מתגלה בממוגרפיות‬
‫ג‪ .‬בערך ב‪ 50%-‬מהמקרים תתפתח בסופו של דבר ‪invasive carcinoma‬‬
‫ד‪ .‬לא שולחת גרורות לבלוטות לימפה‬
‫ה‪ Cadherin E .‬לא צובע את תאי הגידול‪ ,‬אך חיובי ב‪.ductal carcinoma in situ-‬‬

‫‪ .102‬ב‪ Medullary carcinoma-‬של השד הכל נכון פרט ל‪:‬‬


‫א‪ .‬ברב המקרים הצביעה לרצפטורים לאסטרוגן ופרוגסטרון שלילית‬
‫ב‪ .‬ברב המקרים אין ‪ over expression‬של ‪ERB2‬‬
‫ג‪ .‬מיקרוסקופית הגידול מורכב מתאים אטיפיים רבים מעורבים בתסנין לימפוציטרי‬
‫ד‪ .‬בגידול אזורים רבים של פיברוזיס‬
‫ה‪ .‬שייך לקבוצת גידולים בעלי התנהגות קלינית יחסית טובה‬

‫‪ .103‬סלייד מספר ‪:4‬‬


‫א‪ .‬תהליך שפיר‬
‫ב‪ .‬תהליך ממאיר‬
‫ג‪ .‬תהליך טרום סרטני‬
‫ד‪ .‬תהליך טרום אינבזיבי‬

‫‪ .104‬הטיפול הכירורגי הנכון בגוש בגודל ‪ 2‬ס"מ המאובחן היסטולוגית כ‪ductal carcinoma in -‬‬
‫‪ situ‬הוא‪:‬‬
‫א‪Bilateral mastectomy .‬‬
‫ב‪Lumpectomy + axillary lymph node dissection .‬‬
‫ג‪Lumpectomy + irradiation .‬‬
‫ד‪Modified radical mastectomy .‬‬
‫‪ .105‬מה ההיסטולוגיה השכיחה יותר של סרטן צוואר הרחם הקשורה ל‪? HPV-‬‬
‫א‪Squamous cell carcinoma .‬‬
‫ב‪Adenocarcinoma .‬‬
‫ג‪Clear cell carcinoma .‬‬
‫ד‪Mucinous carcinoma .‬‬
‫ה‪Oat cell carcinoma .‬‬

‫‪ .106‬באיזה מבין הגידולים הבאים של האנדומטריום מוצאים מוטציות של הגן ‪? P53‬‬


‫א‪Endometrioid carcinoma .‬‬
‫ב‪Serous papillary carcinoma .‬‬
‫ג‪Malignant Mixed Mullerian tumor .‬‬
‫ד‪Endometrial stromal sarcoma high grade tumor .‬‬
‫ה‪Leiomyosarcoma .‬‬

‫אורולוגיה‬
‫‪ .107‬מה נכון לגבי התהליך שבתרשים מספר ‪? 6‬‬
‫א‪ .‬רוב הנגעים מופיעים באיזור המרכזי של הערמונית מסביב לשופכה‬
‫ב‪ .‬בבלוטות של הנגע נמצאו תאים בזליים‬
‫ג‪ .‬סיפור משפחתי של המחלה אינו קשור לסיכון מוגבר לפתח אותה‬
‫ד‪ .‬המחלה יכולה להופיע עם נגעים בעצמות‬

‫‪ .108‬לגבי ‪ ,Germ cell tumors‬הכל נכון פרט ל‪:‬‬


‫א‪ .‬רב הגידולים מראים מבנה אחיד‬
‫ב‪ .‬מבחינה קלינית חשוב להבדיל בין ‪ Seminoma‬ל‪non-seminomatous-‬‬
‫ג‪ 10% .‬מהגידולים מופיעים עם רקע אשך טמיר‬
‫ד‪ .‬השינוי הכרומוזומלי החשוב ביותר הוא ‪Isochromosome 12‬‬
‫ה‪ Seminoma .‬הוא הסוג השכיח של גידולים מסוג זה‬

‫‪ .109‬כל הבאים נכונים‪ ,‬פרט ל‪:‬‬


‫א‪ -Intratubular germ cell neoplasia .‬תהליך שהופך ל‪ Germ cell tumor -‬ב‪50%-‬‬
‫מהמקרים‬
‫ב‪ .‬תאי ‪ Seminoma‬נצבעים ל‪ α-FP-‬ו‪HCG-‬‬
‫ג‪ .‬החולים עם ‪ Spermatocytic seminoma‬הם בדרך‪-‬כלל מעל גיל ‪65‬‬
‫ד‪ Embryonic Carcinoma .‬הרבה יותר אגרסיבית מ‪Seminoma-‬‬

‫‪ .110‬מה נכון לגבי תהליכים פתולוגיים בערמונית ?‬


‫א‪ .‬ב‪ BHP-‬נראית היפרפלזיה של תאי אפיתל וגם סטרומה‬
‫ב‪ Gleason's score .‬קובע את ה‪ Stage-‬של סרטן הערמונית‬
‫ג‪ .‬בגיל מעל ‪ 70‬כ‪ 30%-‬מהגברים מראים ‪BHP‬‬
‫ד‪ .‬בחלק מהבלוטות ב‪ BHP-‬לא נמצאו תאים בזליים‬
‫ה‪ .‬ל‪ P53-‬תפקיד מרכזי בהתפתחות סרטן הערמונית‬

‫‪ .111‬גבר בן ‪ 60‬עם הגדלה שפירה של הערמונית‪ ,‬עובר החלפת ירך ומפתח עצירת שתן‬
‫רפרקטורית‪ .‬איזה מהתרופות תהיה היעילה היותר על מנת להיגמל מהקטטר ?‬
‫א‪Avodart .‬‬
‫ב‪Bethanechol .‬‬
‫ג‪Ephedrine .‬‬
‫ד‪Xataral .‬‬
‫ה‪Novitropan .‬‬
‫‪ .112‬גבר בן ‪ 65‬עובר ‪ 6 .TURP‬חודשים מאוחר יותר הוא מתלונן על אי שליטה לסרוגין בעיקר‬
‫בלילה‪ .‬ב‪ uroflow-‬מתקבלת עקומת פעמון תקינה עם זרם מקסימאלי של ‪ 25‬סמ"ק לשנייה‪.‬‬
‫החולה מסוגל לעצור שתן באופן רצוני‪ .‬הבדיקה האבחנתית המתאימה ביותר למצב זה היא‪:‬‬
‫א‪IVP .‬‬
‫ב‪US .‬‬
‫ג‪ .‬ציסטוסקופיה‬
‫ד‪ .‬ציסטומטוגרם‬
‫ה‪Voiding cystourethrogram .‬‬
‫‪ .113‬באבן יציקה של ‪ 4‬ס"מ בכליה‪ ,‬הטיפול המועדף כיום הינו‪:‬‬
‫א‪ .‬פילוליטוטומיה‬
‫ב‪ .‬נפרקטומיה‬
‫ג‪PCUL .‬‬
‫ד‪ESWL .‬‬
‫ה‪ .‬שתייה מרובה והחמצת שתן‬

‫‪ .114‬חולה לויקמיה עלול לייצר אבנים מסוג‪:‬‬


‫א‪ .‬קלציום אוקסלאט‬
‫ב‪ .‬קלציום פוספט‬
‫ג‪ .‬חומצה אורית‬
‫ד‪ .‬אבני יציקה‬
‫ה‪ .‬אבני ציסטין‬

‫‪ .115‬בדיקות הכרחיות לבירור דימום בשתן )המטוריה מאקרוסקופית( כולל את כל הדברים‬


‫הבאים‪ ,‬למעט‪:‬‬
‫א‪ .‬הדמיה של הכליות והשופכנים ע"י ‪IVP‬‬
‫ב‪ .‬מיפוי כליות‬
‫ג‪ .‬שתן לציטולוגיה‬
‫ד‪ .‬ציסטוסקופיה‬

‫‪ .116‬איזו מההצהרות הבאות נכונה לגבי דימום בשתן )המטוריה( כתוצאה מגידול של שלפוחית‬
‫השתן ?‬
‫א‪ .‬מלווה בד"כ בצריבה ו‪/‬או כאבים במתן שתן‬
‫ב‪ .‬הדמם עלול להופיע לסירוגין‬
‫ג‪ .‬בד"כ גורם לאנמיה‬
‫ד‪ .‬מופיע בד"כ בתחילת תהליך ההשתנה‬

‫‪ .117‬הניקוז הלימפתי מהאשכים מגיע אל‪:‬‬


‫א‪ .‬קשרי לימפה שטחיים במפשעות‬
‫ב‪ .‬קשרי לימפה עמוקים במפשעות‬
‫ג‪ .‬קשרי לימפה באגן‬
‫ד‪ .‬קשרי לימפה ברטרופריטונאום‬

‫‪ .118‬כל המשפטים הבאים לגבי ייצור הטסטוסטרון נכונים‪ ,‬פרט ל‪:‬‬


‫א‪ .‬מיוצר ע"י תאי סרטולי באשך‬
‫ב‪ .‬ייצורו מתווך ע"י ההורמון ‪LH‬‬
‫ג‪ .‬מקורו בכולסטרול‬
‫ד‪ .‬כ‪ 5%-‬מהטסטוסטרון שבסרום נוצר בבלוטת יותרת הכליה‬
‫‪ .119‬הפרמטרים הבאים מעלים את סגולותיו של ה‪ ,PSA-‬פרט ל‪:‬‬
‫א‪PSA velocity .‬‬
‫ב‪ .‬היחס בין ‪ PSA‬לפוספטזה חומצית‬
‫ג‪Ratio: Free PSA / Total PSA .‬‬
‫ד‪PSA density .‬‬
‫‪ .120‬הסיבוכים שייתכנו לאחר כריתה רדיקלית של הערמונית כוללים את כל הנ"ל‪ ,‬פרט ל‪:‬‬
‫א‪ .‬אין אונות‬
‫ב‪ .‬אי שליטה בשתן‬
‫ג‪ .‬היווצרות אנסטומוזות בין השלפוחית לשופכה‬
‫ד‪ .‬רפלוקס לאחד השופכנים‬
‫ה‪ .‬התנקבות של הרקטום‬

‫תמונה מספר ‪:2‬‬

‫תשובות‪:‬‬

‫ג‬ ‫‪101‬‬ ‫ב‬ ‫‪81‬‬ ‫ג‬ ‫‪61‬‬ ‫ב‬ ‫‪41‬‬ ‫ג‬ ‫‪21‬‬ ‫ה‬ ‫‪1‬‬
‫ד‬ ‫‪102‬‬ ‫ה‬ ‫‪82‬‬ ‫ג‬ ‫‪62‬‬ ‫ה‬ ‫‪42‬‬ ‫ב‬ ‫‪22‬‬ ‫ד‬ ‫‪2‬‬
‫ב‬ ‫‪103‬‬ ‫ב‬ ‫‪83‬‬ ‫ד‬ ‫‪63‬‬ ‫ה‬ ‫‪43‬‬ ‫ה‬ ‫‪23‬‬ ‫א‬ ‫‪3‬‬
‫ג‬ ‫‪104‬‬ ‫א‬ ‫‪84‬‬ ‫ד‬ ‫‪64‬‬ ‫ג‬ ‫‪44‬‬ ‫ב‬ ‫‪24‬‬ ‫ב‬ ‫‪4‬‬
‫א‬ ‫‪105‬‬ ‫ג‪+‬ד‬ ‫‪85‬‬ ‫ה‬ ‫‪65‬‬ ‫ד‬ ‫‪45‬‬ ‫ג‬ ‫‪25‬‬ ‫ג‬ ‫‪5‬‬
‫ב‬ ‫‪106‬‬ ‫א‬ ‫‪86‬‬ ‫ג‬ ‫‪66‬‬ ‫ה‬ ‫‪46‬‬ ‫ה‬ ‫‪26‬‬ ‫ב‬ ‫‪6‬‬
‫ד‬ ‫‪107‬‬ ‫א‬ ‫‪87‬‬ ‫ב‬ ‫‪67‬‬ ‫ד‬ ‫‪47‬‬ ‫ה‬ ‫‪27‬‬ ‫ה‬ ‫‪7‬‬
‫א‬ ‫‪108‬‬ ‫ד‬ ‫‪88‬‬ ‫ג‬ ‫‪68‬‬ ‫ג‬ ‫‪48‬‬ ‫ה‬ ‫‪28‬‬ ‫ג‬ ‫‪8‬‬
‫ב‬ ‫‪109‬‬ ‫ד‬ ‫‪89‬‬ ‫ב‬ ‫‪69‬‬ ‫ה‬ ‫‪49‬‬ ‫ב‬ ‫‪29‬‬ ‫ד‬ ‫‪9‬‬
‫א‬ ‫‪110‬‬ ‫ב‬ ‫‪90‬‬ ‫ב‬ ‫‪70‬‬ ‫ב‬ ‫‪50‬‬ ‫ד‬ ‫‪30‬‬ ‫ב‬ ‫‪10‬‬
‫ד‬ ‫‪111‬‬ ‫ד‬ ‫‪91‬‬ ‫ד‬ ‫‪71‬‬ ‫א‬ ‫‪51‬‬ ‫ג‬ ‫‪31‬‬ ‫ב‬ ‫‪11‬‬
‫ד‬ ‫‪112‬‬ ‫ג‬ ‫‪92‬‬ ‫ג‬ ‫‪72‬‬ ‫ב‬ ‫‪52‬‬ ‫ב‬ ‫‪32‬‬ ‫א‬ ‫‪12‬‬
‫ג‬ ‫‪113‬‬ ‫א‬ ‫‪93‬‬ ‫ה‬ ‫‪73‬‬ ‫ב‬ ‫‪53‬‬ ‫ד‬ ‫‪33‬‬ ‫ד‬ ‫‪13‬‬
‫ג‬ ‫‪114‬‬ ‫ב‬ ‫‪94‬‬ ‫ג‬ ‫‪74‬‬ ‫ב‬ ‫‪54‬‬ ‫ב‬ ‫‪34‬‬ ‫ג‬ ‫‪14‬‬
‫ב‬ ‫‪115‬‬ ‫ג‬ ‫‪95‬‬ ‫ב‬ ‫‪75‬‬ ‫א‬ ‫‪55‬‬ ‫ד‬ ‫‪35‬‬ ‫ג‬ ‫‪15‬‬
‫ב‬ ‫‪116‬‬ ‫ג‬ ‫‪96‬‬ ‫ד‬ ‫‪76‬‬ ‫א‬ ‫‪56‬‬ ‫ה‬ ‫‪36‬‬ ‫א‬ ‫‪16‬‬
‫ד‬ ‫‪117‬‬ ‫ב‬ ‫‪97‬‬ ‫ד‬ ‫‪77‬‬ ‫ב‬ ‫‪57‬‬ ‫א‬ ‫‪37‬‬ ‫ב‬ ‫‪17‬‬
‫א‬ ‫‪118‬‬ ‫ד‬ ‫‪98‬‬ ‫ב‬ ‫‪78‬‬ ‫ג‬ ‫‪58‬‬ ‫א‬ ‫‪38‬‬ ‫ג‬ ‫‪18‬‬
‫ב‬ ‫‪119‬‬ ‫א‬ ‫‪99‬‬ ‫ג‬ ‫‪79‬‬ ‫א‪+‬ד‬ ‫‪59‬‬ ‫ב‬ ‫‪39‬‬ ‫ה‬ ‫‪19‬‬
‫ד‬ ‫‪120‬‬ ‫ב‬ ‫‪100‬‬ ‫ה‬ ‫‪80‬‬ ‫ה‬ ‫‪60‬‬ ‫ג‬ ‫‪40‬‬ ‫ב‬ ‫‪20‬‬
‫)אנדוקרינולוגיה‪ ,‬המטואונקולוגיה‪ ,‬זיהומיות‪+‬אנטיוביוטיקה‪,‬‬
‫אשך ופרוסטטה‪ ,‬נשים ושד(‬
‫מקבץ ‪ – III‬מועד א' ‪2007‬‬

‫א‪ .‬פריחות‬ ‫זיהומיות‬


‫ב‪ .‬פוגעת בעצם ובשיניים בילדים‬
‫ג‪ .‬נפרוטוקסית‬ ‫‪ .1‬ילד בן חמש מתלונן על כאב ראש‪,‬‬
‫ד‪ .‬דיכוי מח עצם‬ ‫בחילה‪ ,photophobia ,‬חום ‪ .38.7‬אין‬
‫ה‪ .‬פגיעה עצבית בילדים מתחת לגיל ‪8‬‬ ‫מקור ברור לחום‪ ,‬יש קישיון עורף אבל‬
‫ו‪ .‬נשירת שיער‬ ‫סימן ‪ Kernig‬שלילי‪ .‬בדיקה נוירולוגית‬
‫תקינה כולל קרקעיות עיניים‪ .‬לקחת‬
‫בדיקת דם )שטרם חזרו( והרכבת ערוי‪.‬‬
‫‪ .5‬לגבי קבוצת ה‪ ,Triazoles -‬ציין איזה‬ ‫מה הצעד הבא?‬
‫מהמשפטים הבאים נכון‪:‬‬ ‫א‪ .‬מתן ‪ dexamethasone‬ומיד אח"כ‬
‫א‪ .‬פלוקונזול יעיל כנגד קנדידה‪,‬‬ ‫אנטיביוטיקה‪.‬‬
‫קריפטוקוקוס ואספרגילוס‬ ‫ב‪ CT .‬מח לאחר ‪dexamethazone‬‬
‫ב‪ .‬פלוקונזול נספג היטב במעי אבל לא‬ ‫ואנטיביוטיקה‬
‫חודר טוב למערכת העצבים המרכזית‬ ‫ג‪ .‬ניקור מותני‬
‫ג‪ .‬איטרקונזול נספג היטב במעי וחודר‬ ‫ד‪ .‬לחכות לתוצאות בדיקת הדם‬
‫היטב למערכת העצבים המרכזית‬
‫ד‪ .‬ווריקונזול הוא תרופת הבחירה‬ ‫ה‪ CT .‬מערות האף‬
‫לטיפול באספרגילוס‬
‫‪ .2‬הטיפול האנטיביוטי האמפירי בחולה‬
‫‪ .6‬מקובל לטפל בקולכיצין במצבים‬ ‫עם חום ניוטרופני בלבד יכול להיות כל‬
‫הבאים‪:‬‬ ‫אחת מהאפשרויות הבאות פרט ל‪:‬‬
‫א‪ .‬זיהום סטפילוקוקלי‬
‫א‪Imipenem .‬‬
‫ב‪ .‬ספסיס מ‪E.coli-‬‬
‫ג‪ .‬ספסיס מפסאודומונאס‬ ‫ב‪Ceftazidime + Vancomycin .‬‬
‫ד‪ .‬ספסיס מפנאומוקוק‬ ‫ג‪Ceftriaxone + Amikacin .‬‬
‫ה‪ .‬כל המצבים נכונים‪.‬‬ ‫ד‪Cefepime .‬‬
‫ה‪.‬‬
‫‪ .7‬השימוש ב‪ Azythromycin-‬טוב לכל‬ ‫‪Piperacilline/Tazobactam+Amikac‬‬
‫המצבים הבאים פרט ל‪:‬‬ ‫‪in‬‬
‫א‪ .‬שעלת‬
‫ב‪ .‬כלמידיה טרכומטיס‬ ‫‪ .3‬מהי אחת מתופעות הלוואי של‬
‫ג‪ .‬זיהום ב‪GAS-‬‬ ‫טטראציקלינים ?‬
‫ד‪ .‬דלקת ריאות אטיפית‬ ‫א‪ .‬כיבים בושט‬
‫ה‪ .‬זיהום בדרכי השתן‬ ‫ב‪Red man syndrome .‬‬
‫ג‪ .‬חירשות‬
‫‪ .8‬מבין המאפיינים הבאים מה לא נכון‬ ‫ד‪Grey baby syndrome .‬‬
‫לגבי ‪? clindamycin‬‬ ‫ה‪optic neuritis .‬‬
‫א‪ .‬מעכב יצירת חלבון‬
‫ב‪ .‬פעיל כנגד חיידקים גרם חיוביים פרט‬ ‫‪ .4‬מדוע הופסק השימוש הנרחב‬
‫לאנטרוקוקים‪.‬‬ ‫בכלורומפניקול?‬
‫א‪ .‬הפלה – העובר נדבק בוודאות‬ ‫ג‪ .‬פעיל כנגד חיידקים אנארוביים כולל‬
‫ב‪ .‬טפול ב‪ spiramycin -‬עד לבדיקת מי‬ ‫בקטרואידים‪.‬‬
‫שפיר בשבוע ‪18‬‬ ‫ד‪ .‬פעיל כנגד ‪E.coli‬‬
‫ג‪ .‬טפול ב‪+ pyrimethamine -‬‬ ‫ה‪ .‬יש לה מנגנוני עמידות ‪cross‬‬
‫‪ sulfadiazine‬עד ללידה‬ ‫‪ resistance‬עם המקרולידים‪.‬‬
‫ד‪ .‬מעקב אולטראסאונד )במיוחד ראש‪,‬‬
‫מח‪ ,‬כבד‪ ,‬טחול( עד לבדיקת מי שפיר‬ ‫‪ .9‬מתי מומלץ לטפל ב‪ primaquine-‬על‬
‫מנת שלא תהיה השנות של מלריה?‬
‫‪ .13‬כל ההיגדים הבאים לגבי‬ ‫א‪ .‬לאחר זיהום ב‪ P.malariae -‬שנרכש‬
‫מטרונידזול נכונים פרט ל‪:‬‬ ‫במרכז אמריקה בטיול‬
‫א‪ .‬יעיל לטיפול בחיידקים אארובים‬
‫ב‪ .‬יעיל לטיפול ב‪-‬‬ ‫ב‪ .‬לאחר זיהום ב‪P.falciparum -‬‬
‫שנרכש באפריקה בטיול‬
‫‪Pseudomembranous colitis‬‬
‫ג‪ .‬טיפול למורסה כבדית מ‪-‬‬ ‫ג‪ .‬לאחר זיהום ב‪ P.vivax -‬שנרכש‬
‫בטיול בהודו‬
‫‪Entamoeba histolytica‬‬
‫ד‪ .‬יעיל לטיפול כנגד טפילי מעי‬ ‫ד‪ .‬לאחר זיהום ב‪ P.vivax -‬שנרכש‬
‫ה‪ .‬קיים במתן פומי ותוך ורידי‬ ‫מעירוי דם‬
‫ה‪ .‬לאחר זיהום ב‪P.falciparum -‬‬
‫‪ .14‬איזה מההגדים הבאים לגבי‬ ‫שנרכש עקב הזרקת סמים במזרק‬
‫‪Trimethoprim-Sulfamethoxazole‬‬ ‫משותף‬
‫נכון?‬
‫א‪ .‬יעיל לטיפול בחיידקים אנארובים‬ ‫‪ .10‬באדם מבוגר הסובל מחום ניצפית‬
‫ב‪ .‬יעיל לטיפול בדלקת ראות הנגרמת‬ ‫פריחה אדומה עם גבולות חדים‬
‫ע"י פנאומוקוקוס‬ ‫המתפשטת במהירות על הפנים‪ .‬איזה‬
‫ג‪ .‬יעיל לטיפול באנטרוקוקוס מסוג‬ ‫מהמחוללים הבאים גרם קרוב לודאי‬
‫למחלתו?‬
‫‪VRE‬‬
‫ד‪ .‬יעיל לטיפול בסטפילוקוק זהוב עמיד‬ ‫א‪Staphylococcus aureus .‬‬
‫למתיצילין‬ ‫ב‪Group A streaptococcus .‬‬
‫ה‪ .‬יעיל לטיפול ב‪Clostridium -‬‬ ‫ג‪E.coli .‬‬
‫‪difficele associated colitis‬‬ ‫ד‪pseudomonas aeruginosa .‬‬
‫ה‪Sreptococcus Anaerobic .‬‬
‫‪ .15‬ילד בן ‪ 8‬ננשך ע"י כלב בידו הימנית‪.‬‬
‫‪ 3‬ימים מאוחר יותר החל לסבול מחום‬ ‫‪ .11‬לאיזה מהמזהמים הבאים קיים‬
‫גבוה‪ ,‬נפיחות של היד וסמני דלקת‬ ‫חיסון העשוי למנוע הופעת המחלה?‬
‫במפרק פרק יד‪ .‬מה מבין המחוללים‬ ‫א‪ .‬קולי‬
‫הבאים הוא הגרם הצפוי של זיהום זה?‬ ‫ב‪ .‬שיגלה‬
‫א‪Pseudomonas aeruginose .‬‬ ‫ג‪Norovirus .‬‬
‫ב‪Brucella mellitensis .‬‬ ‫ד‪ .‬קמפילובקטר‬
‫ג‪Pasteurella multocida .‬‬ ‫ה‪ .‬סלמונלה טיפי‬
‫ד‪Albicans candida .‬‬
‫‪ .12‬לאישה בהריון שבוע ‪ IgM 16‬ו‪-‬‬
‫ה‪Hroup B streptococcus .‬‬
‫‪ IgG‬חיוביים ל‪ .toxoplasma -‬לפני‬
‫ההריון היו בדיקות שליליות‪ .‬מה‬
‫תמליץ?‬
‫‪ .16‬מהתרופות הבאות‪ ,‬מהיא התרופה‬
‫‪ Hurthle cells .21‬אופייניים ל‪-‬‬ ‫היעילה כנגד ‪? Pseudomonas‬‬
‫א‪Pappilary Carcinoma .‬‬ ‫א‪Cefuroxime .‬‬
‫ב‪Graves disease .‬‬ ‫ב‪Ceftriaxone .‬‬
‫ג‪Hashimoto thyroiditis .‬‬ ‫ג‪Ceftazidime .‬‬
‫ד‪Thyroiditis De quervain .‬‬ ‫ד‪Cefazolin .‬‬
‫‪Thyroiditis‬‬ ‫ה‪Cefotaxime .‬‬
‫ה‪Hyperthyroidism .‬‬
‫‪ .18‬לאיזה מהתכשירים הבאים יש‬
‫‪ .22‬גידול שמקורו בתאי ‪:C‬‬ ‫הפעילות הטובה ביותר כנגד‬
‫א‪Papillary carcinoma .‬‬ ‫‪methicillin-susceptible S.aureus‬‬
‫ב‪Medullary carcinoma .‬‬ ‫מיצרי ‪?betalactamases‬‬
‫ג‪Follicular carcinoma .‬‬ ‫א‪Penicilin G .‬‬
‫ד‪Anaplastic carcinoma .‬‬ ‫ב‪Ampicillin .‬‬
‫ה‪Follicular adenoma .‬‬ ‫ג‪Cloxacillin .‬‬
‫ד‪Mezlocillin .‬‬
‫‪ .23‬לגבי ‪Medullary Carcinoma‬‬ ‫ה‪Piperacillin .‬‬
‫הקשורה ל‪ ,MEN -‬הכל נכון פרט ל‪:‬‬
‫א‪ .‬קורת בדר"כ בגיל צעיר יותר מאשר‬ ‫‪ .19‬על הטיפול האנטיביוטי האמפירי‬
‫במקרים הספורדים‬ ‫בחולה נויטרופני לכלול ‪Vancomycin‬‬
‫ב‪ .‬מלווה ב‪C cell hyperplasia -‬‬ ‫במידה ו‪:‬‬
‫ג‪ .‬קורת בשכיחות גבוהה יותר דו‪-‬‬ ‫א‪ .‬יש חשד לאבצס תוך בטני‬
‫צדדית‬ ‫ב‪ .‬יש חום מעל ‪ 40‬מעלות‬
‫ד‪ .‬לא מכילה עמילואיד‬ ‫ג‪ .‬יש חשד לזיהום הקטטר התוך ורידי‬
‫ד‪ .‬יש דלקת ריאות‬
‫‪ .24‬ניתן לפעמים להבדיל בין‬ ‫ה‪ .‬יש חשד ל‪granulocytopenic -‬‬
‫‪ parathyroid adenoma‬ל‪-‬‬ ‫‪enterocolitis‬‬
‫‪ parathyroid hyperplasia‬לפי –‪:‬‬
‫א‪ .‬אחוז השומן בבלוטה‬ ‫‪.20‬‬
‫ב‪ .‬נוכחות של שריד של בלוטת‬ ‫מבין המחוללים הבאים‪ ,‬איזה נחשב‬
‫פאראתירואיד תקינהליד‬ ‫שכיח יותר כגורם לאנדוקרדיטיס‬
‫הממצא‬ ‫במסתם טבעי?‬
‫ג‪ .‬משקל הבלוטה‬ ‫‪Enterococcus faecalis‬‬ ‫א‪.‬‬
‫ד‪ .‬מחיצות פיברוטיות‬ ‫‪Staphylococcus epidermitis‬‬ ‫ב‪.‬‬
‫ה‪ .‬מיטוזות‬ ‫‪Candida albicans‬‬ ‫ג‪.‬‬
‫‪Betahemolitic‬‬ ‫ד‪.‬‬
‫‪streptococcus’ Group A‬‬
‫‪ .25‬סוכרת מסוג ‪ – 1‬מה נכון?‬ ‫‪Pseudomonas aeruginosa‬‬ ‫ה‪.‬‬
‫א‪ .‬מחלה תורשתית המועברת בצורה‬
‫אוטוזומלית רצסיבית‬
‫אנדוקרינולוגיה‬
‫ה‪ .‬אנאלוגים של אינסולין שיפרו‬ ‫ב‪ HLA-DR 3-4 .‬מופיע בשכיחות יתר‬
‫בצורה משמעותית את תוכניות‬ ‫בחולים אלה‬
‫הטיפול בחולי סוכרת מסוג ‪1‬‬ ‫ג‪ .‬כאשר אחד מתאומים מפתח את‬
‫המחלה‪ ,‬הסיכון שהתאום השני‬
‫‪ .28‬בסוכרת מסוג ‪ ,1‬כל המשפטים‬ ‫יפתח את המחלה הוא ‪ ,90%‬מה‬
‫הבאים לגבי השלב הטרום סכרתי‬ ‫שמעיד על חשיבות הגורם‬
‫נכונים פרט ל – ‪:‬‬ ‫התורשתי‬
‫א‪ .‬ברוב המקרים יש נוגדנים כנגד‬ ‫ד‪ .‬צריכת חלב פרה מגינה מפני‬
‫מרכיבים שונים של תאי בטא‬ ‫התפתחות המחלה‬
‫ב‪ .‬דיאטה דלת סוכר ופעילות גופנית‬
‫לא יידחו את הופעת המחלה‬ ‫‪ .26‬כל המשפטים הבאים נכונים לגבי‬
‫ג‪ .‬מדובר בתהליך הנמשך לאורך‬ ‫אינסולין‪ ,‬פרט ל –‬
‫חודשים עד שנים‬ ‫א‪ .‬מופרש בשתי פאזות לאחר קבלת‬
‫ד‪ .‬יש היום טיפול אשר ימנע את‬ ‫הגירוי המתאים‬
‫הופעת המחלה‪ ,‬לכן חשוב לאבחן‬ ‫ב‪ .‬עם כל מולקולה של אינסולין‬
‫בשלב זה‬ ‫מופרשות ‪ 2‬מולקולות של ‪C-‬‬
‫ה‪ .‬קרובי משפחה דרגה ראשונה‬ ‫‪) peptide‬מולקולה השייכת‬
‫מהווים את קבוצת ה‪screening -‬‬ ‫לשרשרת אלפא והשנייה לשרשרת‬
‫הראשונית‬ ‫בטא(‬
‫ג‪ .‬גם חומצות אמיניות בדם מביאות‬
‫‪ .29‬בת ‪ 25‬מגיע לרופא המשפחה עם‬ ‫להפרשת אינסולין‬
‫רמת סוכר בצום של ‪ .130‬לפני ‪ 3‬שנים‬ ‫ד‪ .‬במצבים פתולוגים שונים יש ירידה‬
‫בזמן הריון טופלה בדיאטה עקב עליה‬ ‫בהפרשת הפאזה הראשונה של‬
‫ברמת הסוכר‪ .‬אין סיפור משפחתי של‬ ‫האנסולין‬
‫סוכרת‪ BMI .‬של החולה ‪ .27‬מה‬ ‫ה‪ .‬החלבון הראשוני שנוצר בתא הוא‬
‫האבחנה הנכונה לגבי חולה זו?‬
‫‪preproinsulin‬‬
‫א‪Gestational diabetes .‬‬
‫ב‪Diabetes mellitus type II .‬‬ ‫‪ .27‬כל המשפטים הבאים נכונים לגבי‬
‫ג‪Diabetes mellitus type I .‬‬ ‫סוכרת מסוג ‪ 1‬פרט ל – ‪:‬‬
‫ד‪Autoimmune diabetes of adult .‬‬ ‫א‪ .‬שימוש במשאבת אינסולין או מספר‬
‫ה‪MODY .‬‬ ‫רב של זריקות אינסולין ביום הינן‬
‫שתי השיטות המקובלות‬
‫‪ .30‬גבר בן ‪ 55‬הופיע במרפאה עם‬ ‫ב‪ .‬מטרת הטיפול העיקרית היא‬
‫תלונות על צימאון‪ ,‬פוליאוריה וירידה‬ ‫שמירה על ערכי סוכר קרובים‬
‫במשקל – ‪ 10‬ק"ג ב‪ 3 -‬חודשים‬ ‫לערכים התקינים – אך גישה זו‬
‫האחרונים‪ .‬רמת סוכר אקראית – ‪180‬‬ ‫מעלה את הסיכון להתקפי‬
‫‪.mg\dl‬‬ ‫היפוגליקמיה‬
‫מה נכון לעשות בכדי לשלול את מחלת‬ ‫ג‪ .‬מקובל היום להוסיף טיפול דרך‬
‫הסוכרת?‬ ‫הפה כמו סולפונילאוריאה בחולי‬
‫א‪ .‬לבצע בדיקת סוכר עוד שעתיים‬ ‫סוכרת מסוג ‪ 1‬עם עודף משקל‬
‫ב‪ .‬לבצע העמסת סוכר עם ‪ 75‬מ"ג‬ ‫ד‪ .‬מעקב יומי הדוק אחר רמות הסוכר‬
‫גלוקוז ביום אחר‬ ‫בדם )‪ 3-6‬פעמים ביום(‬
‫ג‪ .‬לבצע העמסת סוכר עם ‪ 50‬מ"ג‬
‫גלוקוז ביום אחר‬
‫ד‪ .‬לבצע העמסת סוכר עם ‪ 100‬מ"ג‬
‫גלוקוז ביום אחר‬
‫ה‪ .‬אין צורך בביצוע בדיקות נוספות‬
‫מהו התיאור המתאים לתמונה?‬
‫‪ .31‬בן ‪ ,48‬חולה סוכרת מסוג ‪ ,2‬מטופל‬
‫א‪Insulits in DM-2 .‬‬ ‫בטיפול פומי‬
‫ב‪Hyalinosis in DM-2 .‬‬ ‫)‪.(Metformin+Glibenclamide‬‬
‫ג‪ .‬סקלרוזיס בגלומרולי‬ ‫מה ישקף באופן המהימן ביותר את‬
‫ד‪ .‬אף אחת מהתשובות‬ ‫מצב האיזון שלו לאורך זמן?‬
‫א‪ .‬רמת סוכר בצום‬
‫ב‪ .‬רמות סוכר פוסטפרנדיאליות )לאחר‬
‫‪ .35‬כל הבאים נחשבים מנגנונים‬ ‫ארוחה(‬
‫בפתוגנזה של סיבוכים בסכרת פרט ל‪:-‬‬
‫ג‪ .‬רמת הסוכר בשעה ‪ 3‬בלילה‬
‫א‪Non-enzymatic Glycosylation .‬‬
‫ד‪ .‬רמת ‪HbAlc‬‬
‫ב‪Activation of protein Kinase C .‬‬
‫ה‪ .‬כל התשובות נכונות‬
‫ג‪Influx of intarcellular calcium .‬‬
‫‪and loss of calcium homeostasis‬‬ ‫‪ .32‬צירוף של ‪Galactorrhea-‬‬
‫ד‪Osmotic Injury to insulin .‬‬ ‫‪ Amenorrhea‬והפרעות בפוריות יכולות‬
‫‪independent cell, Due to‬‬ ‫להופיע כתוצאה מכל אחת מההפרעות‬
‫‪hyperglycemia‬‬ ‫הבאות פרט ל –‪:‬‬
‫א‪ .‬תת פעילות של בלוטת התריס‬
‫‪ .36‬מה לא נכון לגבי היפופיזה קידמית‬ ‫ב‪Acromegaly .‬‬
‫?‬ ‫ג‪ .‬גידול בהיפופיזה‬
‫א‪ .‬היפרפרולקטינומה נגרמת על ידי‬
‫אדנומה של ההיפופיזה הקדמית‬ ‫ד‪Herpes Zoster .‬‬
‫ה‪ .‬טיפול בתכשירים דופמינרגים‬
‫ב‪ .‬אדנומה מפרישה ‪ GH‬יכולה לגרום‬
‫להיפופיטוטריטיזם‬ ‫‪ .33‬אחד הסיבוכים אינו נחשב כסיבוך‬
‫ג‪ .‬נמק של ההיפוזה הקדמית יכול‬ ‫של סכרת‪:‬‬
‫לגרום ל ‪diabetes insipidus‬‬ ‫א‪necrotizing papillitis .‬‬
‫ד‪ .‬אדנומה של ההיפופיזה הקדמית‬
‫ב‪hyaline atherosclorsis .‬‬
‫יכולה להיות קשורה לתסמונת ‪.MEN‬‬
‫ג‪microangiopathy .‬‬
‫‪Non secretory pituitary .37‬‬ ‫ד‪Nodular glomeruloscelorsis .‬‬
‫‪ adenoma‬יכולה לגרום ל‬ ‫ה‪thyroditis .‬‬
‫‪: hypopituatirsm‬‬ ‫‪ .34‬להלן תמונה מספר‪: 1-‬‬
‫א‪ .‬נכון‪ ,‬בגלל לחץ ואטרופיה של יתר‬
‫הבלוטה הקדמית‬
‫ב‪ .‬נכון‪ ,‬בגלל לחץ ואטרופיה של‬
‫ההיפופיזה האחורית‬
‫ה‪ .‬שלב ראשון‪ ,‬לבדוק רמת קורטיזול‬ ‫ג‪ .‬לא נכון‪ ,‬בגלל פעילות יתר של‬
‫בדם לאחר סטימולציה עם‬ ‫ההיפופיזה הקדמית‬
‫‪ ,ACTH‬ובהתאם לתוצאה להחליט‬ ‫ד‪ .‬לא נכון‪ ,‬בגלל אטרופיה של היפופיזה‬
‫איך לנהוג‪.‬‬ ‫קדמית‬

‫‪ .42‬כל הבעיות הבאות יכולות להיות‬ ‫‪ .38‬אבחון ממאירות‬


‫בפאוכרומוציטומה נקבע על ידי ‪:‬‬
‫מוסברות ע"י ‪Cushing syndrome‬‬ ‫א‪ .‬מיטוזות‬
‫פרט ל ‪:‬‬ ‫ב‪ .‬פלאומורפיזם של התאים ואזורי נמק‬
‫א‪ .‬אירוע מוחי‬ ‫ג‪ .‬דימום נרחב‬
‫ב‪ .‬מיופתיה‬ ‫ד‪ .‬גרורות‬
‫ג‪ .‬הירסוטיזם‬ ‫ה‪ .‬כולם נכונים‬
‫ד‪ .‬פרפור פרוזדורים‬
‫ה‪ .‬פוליאוריה ופולידיפסיה‬
‫‪ .39‬סלייד מספר ‪Papillary ) 3‬‬
‫‪ .43‬לכל אחד מהמצבים הבאים אפשר‬ ‫‪:(Carcinoma of thyroid‬‬
‫ליחס היפרפיגמנטציה פרט ל ‪:‬‬ ‫א‪ .‬גורמת בדרך כלל לתת פעילות של‬
‫האיבר‬
‫א‪Conn's disease .‬‬ ‫ב‪ .‬גורמת בדרך כלל ליתר פעילות של‬
‫ב‪ .‬מחלת אדיסון‬ ‫האיבר‬
‫ג‪21-hydoxylase deficiency .‬‬ ‫ג‪ .‬גורמת בדרך כלל למוות של רוב‬
‫ד‪Nelson's syndrome .‬‬ ‫החולים‬
‫ה‪ .‬גידול בהיפופיזה‬ ‫ד‪ .‬יכולה להופיע בבלוטות לימפה‬
‫אזוריות‬
‫‪ .44‬בביקורת טיפת חלב של תינוקות‬ ‫ה‪ .‬נוטה להתפשט לריאות ולעצמות‬
‫בת שנתיים‪ ,‬שבדיקתה הייתה תקינה‬
‫קודם לכן‪ ,‬התגלה שיעור באזור הערווה‬ ‫‪ .40‬סלייד מספר ‪Adenoma of – 4‬‬
‫א‪classical 21 hydroxylase .‬‬ ‫‪Adrenal‬‬
‫‪deficiency- salt wasting type‬‬
‫ב‪classical 21 hydroxylase .‬‬ ‫‪ .41‬חולה בדלקת פרקים‪ ,‬מטופל‬
‫‪deficiency - simple virilizing‬‬ ‫בפרדינזון ‪ 20‬מ"ג ליום במשך ‪6‬‬
‫ג‪non classical 21 hydroxlase .‬‬ ‫שבועות‪.‬‬
‫‪deficiency‬‬ ‫איזו מהגישות הבאות נכונה כשרוצים‬
‫ד‪17 – hydroxylase deficiency .‬‬ ‫להספיק טיפול ‪:‬‬
‫ה‪11 – hydroxylase deficiency .‬‬ ‫א‪ .‬בשלב ראשון‪ ,‬לבדוק קורטיזול בדם‬
‫ב‪ 8-‬בבוקר ובהתאם לתוצאה‬
‫להחליט איך לנהוג‬
‫‪ .45‬מה נכון לגבי אנלוגים לסומסטטין ?‬ ‫ב‪ .‬בשל ראשון‪ ,‬לבדוק ‪ ACTH‬בדם ב‪-‬‬
‫א‪ .‬זה טיפול הבחירה באקרומגלי‬ ‫‪ 8‬בבוקר‪ ,‬ובהתאם לתוצאה‬
‫שנגרמה על ידי מיקרואדנומה של‬ ‫להחליט כיצד לנהוג‬
‫ההיפופיזה‬ ‫ג‪ .‬היות והחולה טופל במשך פחות‬
‫ב‪ .‬במרבית חולי אקרומגלי גורמים‬ ‫מחודשיים‪ ,‬ניתן להפסיק טיפול‬
‫להקטנה של מקרואדנומה‬ ‫מייד‪.‬‬
‫ג‪ .‬יכולים להחמיר היפוגליקמיה בחלק‬ ‫ד‪ .‬בשלב ראשון‪ ,‬להוריד מינון‬
‫מחולי אינסולינומה‬ ‫פרדינזון ל ‪ 17‬מ"ג למשך ‪ 4‬ימים‪.‬‬
‫ד‪ .‬חלבון ‪ p53‬מעורר אפופטוזיס‬ ‫ד‪ .‬נקשרים באותה מידה לחמשת סוגי‬
‫בתאים בכל מקרה בו נגרם נזק ל‪-‬‬ ‫קולטנים של סומטוסטטין‬
‫‪DNA‬‬ ‫ה‪ .‬אינם בשימוש למטרות אבחנתיות‬
‫ה‪ .‬התרופה ‪ ATRA‬הניתנת לחולי‬
‫‪ APL‬מעודדת אפופטוזיס ע"י‬ ‫המטואונקולוגיה‬
‫שחרור ‪Cytochrome C‬‬
‫מהמיטוכונדריות‬ ‫‪ – Hodgkin’s lymphoma .46‬מה‬
‫ההבדלים העיקריים בבדיקה ראשונית‬
‫‪ .49‬כל המשפטים הבאים נכונים פרט ל‪-‬‬ ‫בפיזור המחלה ב‪ HD -‬לבין ‪Non-‬‬
‫א‪ .‬ב‪ Burkitt’s lymphoma -‬משופעל‬ ‫‪?(NHL) Hodgkin’s lymphoma‬‬
‫האונקוגן ‪ myc‬דרך‬ ‫א‪ .‬ב‪ HD -‬מתייצגת המחלה בפיזור‬
‫טרנסלוקציהלאחד הגנים‬ ‫ניכר במערכת אורינרית לעומת‬
‫לאימונוגלובולינים‬ ‫‪NHL‬‬
‫ב‪ .‬לחלבון ‪ Bcr/Abl‬יש פעילות‬ ‫ב‪ .‬ב‪ HD -‬מתייצגת המחלה בבלוטות‬
‫טירוזין‪-‬קינאז מוגברת ביחס לזו‬ ‫טורקליות באחוזים ניכרים יותר‬
‫של החלבון ‪Abl‬‬ ‫ובבלוטות מזטריאליות באחוזים‬
‫ג‪ .‬על מנת שתא נורמלי יהפוך‬ ‫פחותים יותר מאשר ‪NHL‬‬
‫לממאיר‪ ,‬צריכים להיפגע בו מספר‬ ‫ג‪ .‬ב‪ HD -‬פיזור משמעותי במערכת‬
‫מסלולים ביוכימיים‪ ,‬כך שיהפוך‬ ‫העיכול לעומת ‪NHL‬‬
‫בלתי תלוי בסיגנלים חיצוניים‬ ‫ד‪ .‬ב‪ NHL -‬מתייצגת המחלה בבלוטות‬
‫לפרוליפרציה‪ ,‬בלתי רגיש לסיגנלים‬ ‫טורקליות באחוזים ניכרים יותר‬
‫מעכבי גדילה‪ ,‬בעל פוטנציאל שכפול‬ ‫ובבלוטות מזנטריליות באחוזים‬
‫בלתי מוגבל ומסוגל להתחמק‬
‫מאפופטוזיס‪.‬‬ ‫פחותים יותר מאשר ‪HD‬‬
‫ד‪ .‬מתילציה מהווה מנגנון אפיגנטי‬ ‫‪ .47‬הסתיידויות בבלוטות לימפה‪:‬‬
‫חשוב בסרטן‪ ,‬בגורמה‬
‫א‪ .‬נדיר לפני טיפול‬
‫לאינאקטיבציה של ‪tumor‬‬
‫ב‪ .‬קורה במחלה שאינה אגרסיבית‬
‫‪ suppressor genes‬רבים‪.‬‬
‫ג‪ .‬לאחר טיפול ייתכנו הסתיידויות‬
‫ה‪ .‬בלימפומות קיימות טרנסלוקציות‬
‫הגורמות להיווצרות חלבון כימרי‪-‬‬ ‫ד‪ .‬נפוץ לפני טיפול‬
‫אונקוגני‪.‬‬ ‫ה‪ .‬א‪+‬ג נכונים‬
‫ו‪ .‬ב‪+‬ד נכונים‬
‫‪ .50‬מה נכון לגבי לימפומה מסוג‬
‫‪? diffuse large B cell‬‬ ‫‪ .48‬אילו מהמשפטים הבאים נכון?‬
‫א‪ .‬מהווה חלק קטן מכלל הלימפומות‬ ‫א‪ .‬חלבון ‪ BCL2‬פועל בגרעין כפקטור‬
‫ב‪ .‬מאופיינת ע"י הישנויות חוזרות‬ ‫שעתוק המגביר קצב חלוקת תאים‬
‫לאורך שנים רבות‬ ‫ב‪ .‬טעויות בתהליך ה‪gene -‬‬
‫ג‪ .‬לעיתים נדירות מערבת מערכות‬ ‫‪ rearrangement‬בגנים‬
‫שונות‪ ,‬כמו דרכי עיכול ועוד‬ ‫לאימונוגלובולינים‪ ,‬תורמות‬
‫ד‪ .‬יכלה להתפתח באזור חוט השדרה‬ ‫להתפתחות לימפומות‬
‫ולגרום ל‪spinal cord -‬‬ ‫ג‪ .‬חייבות להתרחש מוטציות בשני‬
‫האללים של פרוטו‪-‬אונקוגן‪ ,‬על‬
‫‪compression‬‬ ‫מנת שיתרום לתהליך הסרטני‬
‫ה‪ .‬הגברת הפירוק המטבולי של‬ ‫ה‪ .‬הפרוגנוזה גרועה יותר בחולים‬
‫אסטרוגן ע"י הכבד‬ ‫מבוגרים מעל גיל ‪60‬‬

‫‪ .54‬הטיפול היעיל ביותר בהקאות על‬ ‫‪ .51‬מה נכון לגבי אופיואידים אצל‬
‫רקע כמותרפיה הוא‪:‬‬ ‫חולים אונקולוגים?‬
‫א‪ .‬טיפול פסיכולוגי ופסיכיאטרי‬ ‫א‪ .‬רוב חולי הסרטן מפתחים סיבולת‬
‫ב‪ .‬טיפול מניעתי‪ ,‬אשר ניתן לפני‬ ‫)טולרנס( לאפקט האנלגטי של‬
‫כמותרפיה ונמשך עד ‪ 5‬ימים לאחר‬ ‫אופיואידים‬
‫מכן‬ ‫ב‪ .‬אופיואידים ניתנים רק לחולי סרטן‬
‫ג‪ .‬טיפול שניתן ‪ 24‬שעות לאחר סיום‬ ‫סופניים‬
‫כמותרפיה‬ ‫ג‪ .‬עצירות הינה תופעת לוואי שכיחה‬
‫ד‪ .‬טיפול שניתן ‪ 5‬ימים לאחר סיום‬ ‫אצל חולי סרטן המטופלים‬
‫כמותרפיה‬ ‫באופיואידים‬
‫ד‪ .‬אופיואידים הינם "אבן היסוד"‬
‫‪ .55‬להלן תמונה מספר ‪:2‬‬ ‫לטיפול בחולים אונקולוגים‬
‫הסובלים מכאב‬
‫ה‪ .‬תשובות ג‪+‬ד נכונות‬

‫‪ .52‬לגבי )‪Bevacizumam (Avastin‬‬


‫הכל נכון פרט ל‪:-‬‬
‫א‪ .‬התרופה הינה נוגדן מונוקלונלי‬
‫ב‪ .‬פועלת במנגנון של התקשרות ל‪-‬‬
‫‪VEGF‬‬
‫ג‪ .‬מנגנון הפעולה קשור לעיכוב יצירת‬
‫כלי דם בגידול הממאיר‬
‫ד‪ .‬לא מקובל לתת תרופה זאת בשילוב‬
‫עם תרופות ציטוטוקסיות‬
‫מה אבחנתך ?‬ ‫ה‪ .‬יעילותה של תרופה זאת הוכחה‬
‫א‪Follicular lymphoma .‬‬ ‫בסרטן גרורתי של המעי הגס‬
‫ב‪Mantle Cell lymphoma .‬‬
‫ג‪Nodular lymphocytic .‬‬ ‫‪ .53‬מהו מנגנון הפעולה של ‪Tamoxifen‬‬
‫‪predominant Hodgkin‬‬ ‫בחולות עם סרטן גרורתי של השד?‬
‫‪lymphoma‬‬ ‫א‪ .‬עיכוב יצירת אסטרוגנים ע"י‬
‫ד‪Nodal Marginal Zone .‬‬ ‫השחלות‬
‫‪Lymphoma‬‬ ‫ב‪ .‬עיכוב הפרשת הורמונים‬
‫ה‪Burkitt’s Lymphoma .‬‬ ‫גונדוטרופים ע"י יותרת המוח‬
‫)היפופיזה(‬
‫‪ .56‬מה נכון לגבי ‪Burkitt’s‬‬ ‫ג‪ .‬התקשרות לקולטנים )רצפטורים(‬
‫‪? Lymphoma‬‬ ‫של אסטרוגן הנמצאים בגרעין של‬
‫התאים הממאירים‬
‫א‪ “stary sky pattern” .‬אבחנתי‬
‫לגידול זה‬ ‫ד‪ .‬עיכוב הפיכת האנדרוגנים‪,‬‬
‫המיוצרים ביותרת הכליה )אדרנל(‬
‫לאסטרוגנים‬
CD20 .‫ג‬ ‫הגידול מתאפיין באחוז צמיחה‬ .‫ב‬
CD138 .‫ד‬ 100% ‫( של‬growth fraction)
LAP .‫ה‬ ‫התאים הממאירים הם תאי לימפה‬ .‫ג‬
T
‫ איזו טרנסלוקציה אופיניית‬.59 ‫הפגם הגנטי שאופייני לגידול זה‬ .‫ד‬
‫ללימפומה פוליקולרית‬
t(11,18) ‫הוא‬
14/18 .‫א‬
‫ מהמקרים קיים‬80% -‫ביותר מ‬ .‫ה‬
8/14 .‫ב‬
HIV ‫קשר בין המחלה והנגיף‬
11/14 .‫ג‬
22/9 .‫ד‬
:3- ‫ להלן תמונה מספר‬.57
‫נשים ושד‬

Carcinoma of ‫ לגבי‬.60
:-‫ הכל נכון חוץ פרט ל‬endometrium
‫ מןפיע ברוב המקרים בגיל מבוגר‬.‫א‬
(postmenopausal) ‫אחרי הבלות‬
‫ של‬Serous papillary carcinoma .‫ב‬
-‫האמדומטריום גידול אגרסיבי יותר מ‬
Endometroid type of
adenocarcinoma
‫ רוב הגידולים של רירת הרחם הם‬.‫ג‬
adenocarcinoma ‫מסוג של‬
‫ ב‬squamous differentiation .‫ד‬
‫ הוא פקטור‬adecocarcinoma
.‫פרוגניסטי גרוע‬
Serous type of endometrial .‫ה‬
P-35 ‫קשור למוטציה של‬carcinoma
?‫מהי אבחנתך‬
‫ סמן‬Trophpblastic disease ‫ לגבי‬.61 Follicular Lymphoma .‫א‬
:‫את הנכון‬ Malt lymphoma .‫ב‬
placental site trophoblastic .‫א‬ Mixed cellular HL .‫ג‬
intermediate -‫ בנוי מ‬tumor Mantle lymphoma .‫ד‬
trophoblast Plasma cell myeloma .‫ה‬
syn- ‫ בנויה מ‬choriocarcinoma .‫ב‬
cytotrophoblast + cytotrophoblast ‫ מהו הסמן המתאים לתאים‬.58
placental site trophoblastic .‫ג‬ ? 3 ‫בתמונה‬
placental ‫ מפריש בעיקר‬tumor CD5 .‫א‬
lactogen CD10 .‫ב‬
‫ד‪ .‬הפרוגנוזה של סרטן שד האישה‬ ‫ד‪ choriocarcinoma .‬מפריש ‪Human‬‬
‫טובה יותר מאשר בגבר ב‪ stage -‬זהה‪.‬‬ ‫‪chorionic gonadotropin‬‬
‫ה‪ .‬רוב ‪ Ductual carcinoma‬נצבעות ל‬ ‫ה‪ .‬כולם נכונים‪.‬‬
‫‪E-cadherin‬‬
‫‪ call exner bodies .62‬רואים ב‪:‬‬
‫‪ .67‬האבחנה הסבירה ביותר בממוגרפיה‬ ‫א‪dysgerminoma .‬‬
‫של גוש עגול נייד גמיש עם הסתיידות‬ ‫ב‪Brenner tumor .‬‬
‫בצורת פופקןרן היא ‪:‬‬ ‫ג‪granulosa cell tumor .‬‬
‫א‪ .‬פפילומה‬
‫ב‪ .‬פיברואדנומה‬ ‫ד‪fibroma.‬‬
‫ג‪ .‬נמק שומני‪.‬‬
‫ד‪ .‬המגניומה‬ ‫‪ Lubolar carcinoma in situ .63‬הכל‬
‫ה‪DCIS .‬‬ ‫נכון פרט ל‪:-‬‬
‫א‪ .‬בדרך כלל ממצא מקרי‬
‫‪ .68‬קולטן ‪ HER2‬חיובי בגידול ממאיר‬ ‫ב‪ .‬בדרך כלל לא רואים הסתיידות ולכן‬
‫בשד קשור עם כל הבאים פרט ל‪:-‬‬ ‫לעתים לא מתגלים בממוגרפיה‪.‬‬
‫א‪ .‬פרוגנוה טובה יותר יחסית לגידולים‬ ‫ג‪ .‬מתגלה כגוש נמוש קשה‬
‫עם קולטן שלילי‪.‬‬ ‫ד‪ .‬לא שולחת גרורות לבלוטות לימפה‪.‬‬
‫ב‪ .‬המלצה למתן טיפול כימוטרפי‬ ‫ה‪ .‬מהווה מרקר של סיכון גבוה‬
‫משלים עם אינטרציקלינים‪.‬‬ ‫להתפתחות סרטן אינבזיבי‪.‬‬
‫ג‪ .‬עמידות לטיפול הורמונלי ע"י‬
‫‪tamoxifen‬‬ ‫‪ .64‬סלייד ‪Paget’s Disaese‬‬
‫ד‪ .‬המלצה למתן טיפול משלים‬
‫‪Herceptin‬‬ ‫‪ Tubular carcinoma .65‬של השד‪ .‬הכל‬
‫נכון פרט ל‪:-‬‬
‫‪ .69‬כל המשפטים הבאים בקשר‬ ‫א‪ .‬גידול בעל דיפרנציאציה גבוהה‬
‫לנגזרות אסטרוגניות נכונות פרט ל‪:-‬‬ ‫ב‪ .‬בעל פרוגנוזה טובה‬
‫א‪ .‬אתניל אסטרדיול היא הנגזרת‬ ‫ג‪ .‬כמעט כל הגידולים מסוג זה שליללים‬
‫האסטרוגנית הנפוצה ביותר בגלולה‬ ‫לרצפטורים לאסטרוגן‬
‫למניעת היריון‪.‬‬ ‫ד‪ .‬גרורות לבלוטות לימפה בפחות מ‬
‫ב‪ . .‬אתניל אסטרדיול פועל לזמן קצר‬ ‫‪ 10%‬של המקרים‬
‫בגלל פירוק מהיר בכבד‪.‬‬ ‫ה‪ .‬גידול קטן יחסית ) כ ‪1‬סמ(‬
‫ג‪ .‬מתן אסטרוגן דרך מדבקה על העור‬
‫עוקף את השפעותיו הראשוניות בכבד‪.‬‬ ‫‪ .66‬מה נכון לגבי גידולי השד?‬
‫ד‪ .‬עדיף לתת אסטרוגנים יחד עם נגזרת‬ ‫א‪ .‬ב‪ Medullary carcinoma -‬של השד‬
‫פרוגסטינית כדי לצמצם את השפעות‬ ‫נצפית ‪overexpression of Her2/Neu‬‬
‫פרולפרטיביות ברחם‬ ‫ב‪ .‬בין הנשים נשאיות של ‪BRCA1‬‬
‫ה‪ .‬אסטרוגנים יעילים במיוחד במניעת‬ ‫‪ Gene‬רוב הגידולים הממאירים הם‬
‫גלי חום אצל נשים בגיל המעבר‬
‫מסוג של ‪Tubular carcinoma‬‬
‫‪ .70‬סמנו את ההצהרה הנכונה ביותר‬ ‫ג‪ Inflammatory carcinoma .‬מראה‬
‫בקשר למטבוליטים של טיבולין‬ ‫אזורים נרחבים של דלקת חריפה‬
‫הנוצרים ברקמות השונות‪:‬‬ ‫בגידול‪.‬‬
‫‪endometroid carcinoma‬‬ ‫א‪.‬‬ ‫א‪ .‬ברקמת השד‪ ,‬הם מונעים יצירה של‬
‫‪serous papillary carcinoma‬‬ ‫ב‪.‬‬ ‫אסטרוגנים פעילים‬
‫‪malignant mixed mullerian‬‬ ‫ג‪.‬‬ ‫ב‪ .‬ברקמת השד‪ ,‬הם מעודדים יצירה של‬
‫נגזרות אסטרוגניות בלתי פעילות‬
‫‪tumor‬‬ ‫ג‪ .‬טיבולין אינו עובר מטבוליזם כלל‬
‫‪endometrial stromal sarcoma,‬‬ ‫ד‪.‬‬ ‫ברקמת השד‪.‬‬
‫‪high grade‬‬ ‫ד‪ .‬בעצם‪ ,‬הם אנטגוניסטים לרצפטור‬
‫‪Leimyosarcoma‬‬ ‫ה‪.‬‬ ‫לאסטרוגן‪.‬‬
‫ה‪ .‬ברירית הרחם‪ ,‬הם גורמים לשגשוג‬
‫האנדומטריום‪.‬‬
‫‪ .76‬טיפול לסרטן צוואר הרחם בשלב‬
‫מתקדם לא נתיח‪:‬‬ ‫‪ .71‬סמן את המשפט הנכון ביותר‬
‫א‪ .‬כימותרפיה בלבד‬ ‫בהקשר להפלה מאיימת‪:‬‬
‫ב‪ .‬הקרנתי חיצוני‪ ,‬פנימי בשילוב עם‬ ‫א‪ .‬מוגדר כדימום נרתיקי לפני שבוע ‪20‬‬
‫כימותרפיה‬ ‫ב‪ .‬מוגדר כדימום נרתיקי בטרימסטר‬
‫ג‪ .‬הקרנתי פנימי בשילוב עם הורמונים‬ ‫הראשון‪.‬‬
‫ד‪ .‬טפול הקרנתי אינו יעיל כ"כ בסרטן‬ ‫ג‪ .‬קורה ב ‪ 10%‬מהלידות‬
‫צוואר הרחם‬ ‫ד‪ .‬שכיח יותר בנשים מולטיפורות עם‬
‫‪ Rh‬שלילי‪.‬‬
‫‪ .77‬בת ‪ ,30‬הריון ראשון שבוע ‪ ,34‬פנתה‬
‫למיון יולדות בגלל דימום נרתיקי וכאבי‬ ‫‪ .72‬בת ‪ 65‬פנתה לברור מיימת‪ -‬איזה‬
‫בטן‪.‬‬ ‫מהבדיקות המצוינות אינה מתבקשת‬
‫לפניך הניטור העוברי‪:‬‬ ‫בשלב זה?‬
‫תמונה של ‪Late Decelerations‬‬ ‫א‪ .‬צילום חזה‬
‫מהו הטיפול המומלץ?‬ ‫ב‪ .‬בדיקת על קול בטן‪ +‬אגן‬
‫א‪ .‬ילוד מיידי לאחר ייצוב האישה‬ ‫ג‪ .‬תפקודי כבד כבד‬
‫ב‪ .‬ייצוב האישה ולאחר מכן אשפוז‬ ‫ד‪ .‬תפקושי נשימה‬
‫למעקב והשגחה עד שבוע ‪ ,37‬ואז‬ ‫ה‪ .‬בדיקת שד‬
‫יילוד‪.‬‬
‫ג‪ .‬ייצוב האישה‪,‬ביצוע ניטור עוברי‬ ‫‪ .73‬סלייד – ‪Granulosa theca cell‬‬
‫ממושך יותר ויילוד רק אם לא‬ ‫‪tumor‬‬
‫יהיה שיפור במוניטור‪.‬‬
‫ד‪ .‬הניהול משתנה ותלוי בסיבה שגרמה‬ ‫‪ .74‬אישה בשנות ה‪,30-‬משתמשת מזה‬
‫לדימום‪.‬‬ ‫חודשיים בהתקן תוך רחמי‪ .‬מגיעה‬
‫לחדר מיון כסובלת מחום ‪ .38.3‬בבדיקה‬
‫‪ .78‬באיזה מבין הבאים יש לחשוד‬ ‫גופנית רגישות בהנעת טפולים‪ .‬מה‬
‫בחסימה או חסר ב‪VAS -‬‬ ‫האבחנה הסבירה ביותר‪:‬‬
‫‪ DEFERENCE‬בגברים אזוספרמיים?‬ ‫‪.‬א‪ .‬הריון תקין‬
‫ב‪ .‬הפלה מלאה‬
‫א‪ .‬קריוטיפ ‪XXY‬‬ ‫ג‪ .‬הפלה חלקית‬
‫ב‪ .‬חסרים קטנים בזרוע הארוכה של ‪Y‬‬ ‫ד‪PID.‬‬
‫ג‪ .‬נשאי המוטציה לציסטיק פיברוזיס‬ ‫ה‪.‬‬
‫ד‪ .‬אזוספרמיה ואוליגוספרמיה קשה‬
‫ה‪ .‬במצבים של אזוספרמיה‬ ‫‪ .75‬באיזה גידול מהבאים מוצאים‬
‫ואוליגוספרמיה קשה וגם במקרה‬ ‫מוטציות של הגן ‪ ?p53‬סמן את הנכון‬
‫ג‪ .‬אצל זוגות בריאים ממשפחות‬ ‫של נשאות המוטציה לציסטיק‬
‫בריאות ללא קרבת משפחה הסיכון‬ ‫פיברוזיס‬
‫אפסי‬
‫ד‪ .‬שיעור הבעיות המולדות תלוי בעיקר‬ ‫‪ .79‬המשפטים הבאים בנוגע לנזק‬
‫בגיל האם‬ ‫נוירולוגי מאוחר נכונים‪ ,‬פרט ל‪:-‬‬
‫א‪ .‬היפוקסיה שלעצמה אינה גוררת נזק‬
‫נוירולוגי‪.‬‬
‫‪ .83‬נקראת בשלוש לפנות בוקר לבדוק‬ ‫ב‪ .‬מצב של תשניק מאופיין ע"י חמצת‬
‫אישה במיון יולדות‪ .‬נמסר לך כי מדובר‬ ‫מטבולית‬
‫בהיפרדות שליה‪ .‬מה הסימן השכיח‬ ‫ג‪ .‬חמצת מטבולית עשויה לגרום‬
‫ביותר שאותו תצפה למצוא לאור‬ ‫לפגיעה רב מערכתית בעובר ובילוד‬
‫האבחנה הנ"ל?‬ ‫ד‪ .‬במרבית המקרים של שיתוק מוחין‬
‫א‪ .‬דימום נרתיקי‬ ‫ניתן לזהות סימנים של מצוקת‬
‫ב‪ .‬כאבי בטן‬ ‫העובר סביב הלידה‬
‫ג‪ .‬ירידה בתנועות העובר‬
‫ד‪ .‬הפרעה בתפקודי קרישה‬ ‫‪ .80‬מה המשפט הנכון לגבי הסמן ‪CA-‬‬
‫‪?125‬‬
‫‪ .84‬הטיפול ב‪ MILD PID -‬הינו‪:‬‬ ‫א‪ .‬הסמן עולה רק בממאירויות ממקור‬
‫א‪ .‬לפרוסקופיה‬ ‫גניטלי‬
‫ב‪ .‬היסטרוסקופיה‬ ‫ב‪ .‬המסן אינו מתאים לבדיקת סקר‬
‫ג‪ .‬אנטיביוטיקה פראנטרלית ‪+‬‬ ‫לגילוי מוקדם של סרטן השחלות‬
‫אישפוז‬ ‫ג‪ .‬הסמן לרוב נמוך יותר בנשים‬
‫ד‪ .‬אנטיביוטיקה פומית אמבולטורית‬ ‫צעירות לפני גיל המעבר‬
‫‪ +‬מעקב‬ ‫ד‪ .‬הסמן הינו תגובה אימונית לתאי‬
‫ה‪ .‬כריתת האיבר המודל‬ ‫הגידול‬

‫‪ .85‬המדידה הסונוגרפית המדויקת‬


‫ביותר לקביעת גיל ההריון הינה‪:‬‬ ‫‪ .81‬מי מהבדיקות הבאות קשורה‬
‫לסיכון הגבוה ביותר להפלה?‬
‫א‪ .‬היקף ראש העובר בשבוע ‪30‬‬
‫א‪ .‬ניקור מי שפיר בשבוע ‪ 15‬להריון‬
‫ב‪ .‬קוטר ביפריאטלי של ראש‬
‫העובר בשבוע ‪20‬‬ ‫ב‪ .‬ניקור מי שפיר בשבוע ‪ 32‬להריון‬
‫ג‪ .‬מרחק ‪Crown-Rump Length‬‬ ‫ג‪ .‬בדיקת שקיפות עורפית‬
‫בשבוע ‪8‬‬ ‫ד‪ .‬ביופסית סיסי שליה‬
‫ד‪ .‬קוטר שק ההריון בשבוע ‪8‬‬
‫‪ .82‬מה הקביעה הנכונה הנוגעת לשיעור‬
‫ה‪ .‬אורך הירך בשבוע ‪20‬‬ ‫הבעיות המולדות באוכלוסיה?‬
‫א‪ .‬אצל קרובי משפחה הסיכון גבוה‬
‫‪ .86‬מה נכון לגבי ניטור הפעילות‬ ‫אם ידוע על בעיה רפואית מולדת‬
‫הרחמית?‬ ‫במשפחה‬
‫א‪ .‬ניטור חיצוני של הפעילות הרחמית‬ ‫ב‪ .‬שיעור הבעיות באוכלוסיה הכללית‬
‫איננו מספיק לניהול תקין של לידה‬ ‫נאמד בכ‪3%-‬‬
‫ב‪ .‬ניטור הפעילות הרחמית חשוב לשם‬
‫הגדרת סוג ההאטות בדופק העוברי‬
‫ג‪ .‬הטיפול נועד לנשים שקיימת אצלן‬ ‫ג‪ .‬ניטור פנימי של הפעילות הרחמית‬
‫הוראת נגד לניתוח‬ ‫איננו מוסיף על המידע המתקבל מניטור‬
‫ד‪ .‬הטיפול נועד לטווח קצר לנשים‬ ‫חיצוני‬
‫שמעוניינות בטיפולי פריו‪.‬‬ ‫ד‪ .‬ניטור חיצוני של הפעילות הרחמית‬
‫מתבצע באמצעות גלי אולטרסאונד‬
‫‪ .91‬בהפלה ספונטנית הכל נכון פרט ל‪:‬‬
‫א‪ 15%-20% .‬מההריונות הקליניים‬ ‫‪ .87‬תפקיד המרכיב האסטרוגני בגלולה‬
‫מסתיימים בהפלה ספונטנית‬ ‫המשולבת כולל את כל הבאים פרט ל‪:-‬‬
‫ב‪ 30% .‬מההרינות הפרה קליניים‬ ‫א‪ .‬הפרשת ‪LH surge‬‬
‫מסתיימים בהפלה‬ ‫ב‪ .‬מניעת דימומים לא סדירים‬
‫ג‪ .‬כ‪ 80% -‬מההפלות הספונטניות‬ ‫ג‪ .‬מניעת הפרשת ‪FSH‬‬
‫מתרחשות בטרימסטר הראשון של‬ ‫ד‪ .‬פוטנציאציה של פעילות הפרוגסטרון‬
‫ההריון‬
‫ד‪ .‬לנשים שהייתה להן הפלה ספונטנית‬ ‫‪ .88‬כל המשפטים הבאים בקשר‬
‫אחת יש סיכוי של ‪ 50%‬להפלה חוזרת‬ ‫להשפעות שימוש בגלולות על גידולים‬
‫ממאירים נכונים פרט ל‪:-‬‬
‫‪ .92‬לגבי סרטן צואר הרחם‪ ,‬סמן את‬ ‫א‪ .‬עליה בשכיחות ‪Premenopausal‬‬
‫הנכון‪:‬‬ ‫‪breast carcinoma‬‬
‫א‪ .‬ירידה בכמות המקרים של סרטן‬ ‫ב‪ .‬עליה בשכיחות ‪Epithelial ovarian‬‬
‫פולשני של צואר הרחם קשורה‬
‫‪carcinoma‬‬
‫לירידה בהדבקה ב‪HPV -‬‬ ‫ג‪ .‬ירידה בשכיחות ‪In situ ca of cervix‬‬
‫ב‪ .‬סקירת האוכלוסיה ב‪pap smear -‬‬ ‫ד‪ .‬ירידה בשכיחות ‪Endometrial‬‬
‫היא הגורם העיקרי בירידת כמות‬
‫המקרים של סרטן צואר הרחם‬ ‫‪carcinoma‬‬
‫בעולם המערבי‬
‫ג‪ Clear Cell Carcinoma .‬של צואר‬ ‫‪ .89‬בת ‪ ,25‬רווקה‪ ,‬ללא ילדים‪ .‬בבדיקה‬
‫שגרתית נצפה שרירן בקוטר ‪ 6‬ס"מ‬
‫הרחם קשורה ל‪ HPV -‬מסוג ‪16,15‬‬
‫מסוג ‪ Sub serous‬בפונדוס‪ .‬סמן את‬
‫ד‪ .‬קוילוציטוזיס מופיעה רק במקרים‬ ‫המשפט הנכון בהקשר למצבה‪:‬‬
‫של הדבקה ב‪ HPV -‬מסוג ‪16,18,31‬‬ ‫א‪ .‬יש לשקול ביצוע היסטרוסקופיה‬
‫ה‪ Condyloma acuminate .‬בדרך כלל‬ ‫לכריתת הממצא‬
‫קשורה ל‪ HPV -‬מסוג ‪16,18‬‬ ‫ב‪ .‬במידה ויש עדות שהשרירן הכפיל את‬
‫נפחו‪ ,‬יש להציע כריתת רחם‬
‫אשך ופרוסטטה‬ ‫ג‪ .‬בד"כ החולה לא תסבול מדימום‬
‫וסתי ובין וסתי מוגבר‬
‫‪ .93‬בן ‪ 35‬פנה לרופא בגלל גוש באשך‬ ‫ד‪ .‬כריתה לפרוסקופית אינה אפשרית‬
‫השמאלי‪ .‬רמת ה‪ β-HCG‬וה‪α -‬‬ ‫במצב זה‬
‫‪ fetoprotein‬בדם הייתה תקינה‪ .‬הגידול‬
‫בנוי ממשטחים של תאים בהירים‬ ‫‪ .90‬סמן את המשפט הנכון בקשר‬
‫אטיפיים עם הסננה לימפוציטרית‬ ‫לטיפול ב‪:GnRH agonist -‬‬
‫בסטרומה‪ .‬צביעה אימונוהיסטוכימית‬ ‫א‪ .‬הטיפול המומלץ לנשים אחרי גיל‬
‫ל‪placental alkaline phosphatase‬‬ ‫הפריון‬
‫הייתה חיובית‪.‬‬ ‫ב‪ .‬הטיפול יעיל לאורך זמן גם לאחר‬
‫מה האבחנה שלך?‬ ‫סיומו‬
‫כל המצבים הבאים עלולים לגרום‬ ‫א‪Embrional carcinoma.‬‬
‫להפרעת השתנה זו‪ ,‬פרט ל‪:‬‬ ‫ב‪Lymphoma .‬‬
‫א‪ .‬חסימת מוצא השלפוחית עקב סרטן‬ ‫ג‪Choriocracinoma .‬‬
‫הערמונית‬
‫ד‪Seminoma .‬‬
‫ב‪ .‬זיהום בדרכי השתן‬
‫ה‪Teratoma .‬‬
‫ג‪ .‬הגדלה שפירה של הערמונית‬
‫ד‪ .‬פגיעה בהתכווצות הדטרוזור‬ ‫‪ .94‬מה נכון לגבי ‪? Yolk Sac Tumor‬‬
‫)‪(Detrusor‬‬ ‫א‪ .‬הוא שכיח אצל גברים מבוגרים‬
‫ה‪ .‬היצרות בשופכה‬ ‫ב‪ .‬צביעה ל‪ β-HCG -‬תמיד חיובית‬
‫ג‪ .‬בדרך כלל הינו גידול אופייני בילדים‬
‫‪ .98‬כל המצבים הבאים מהווים‬ ‫ד‪ .‬הוא סוג של טרטומה‬
‫התוויית נגד לטיפול בברכיותרפיה‬ ‫ה‪ .‬התא העיקרי של הגידול הוא תא‬
‫בסרטן הערמונית‪ ,‬פרט ל‪:‬‬ ‫טרופובלסט‬
‫א‪ .‬חולה שעבר כריתה של הערמונית‬
‫דרך השופכה לפני ‪ 3‬שנים‬ ‫‪ .95‬סמן מה נכון לגבי ‪Gleason’s‬‬
‫ב‪ .‬התמיינות פתולוגית בדירוג )‪8(4+4‬‬
‫‪?score‬‬
‫‪Adenocarcinoma Gleason‬‬ ‫א‪ .‬זה דירוג קליני של תפקוד אורודינמי‬
‫ג‪ .‬זרם שתן חסימתי בעל שיא נמוך מ‪8-‬‬
‫מ"ל לשנייה‬ ‫ב‪ .‬זו מערכת של ‪ staging‬בגידולי האשך‬
‫ד‪ .‬נוכחות של גרורות גרמיות במיפוי‬ ‫ג‪ .‬זו מערכת של ‪ staging‬של סרטן‬
‫עצם‬ ‫הערמונית‪ ,‬מבוססת על אטיפיה ומספר‬
‫ה‪ .‬רמת ‪ PSA‬בסרום ‪ 9.97‬ננוגרם למ"ל‬ ‫מיטוזות‬
‫ד‪ .‬זו מערכת ‪ staging‬של סרטן‬
‫‪ .99‬במה שונה השיטה להערכת‬ ‫הערמונית‪ ,‬מבוססת על מבנה וצורת‬
‫ההיסטולוגיה של גידולי הערמונית‬ ‫הבלוטות‬
‫)דירוג גליסון( מיתר הגידולים‬ ‫ה‪ .‬זו מערכת לקביעת דרגת המעורבות‬
‫הסרטניים בגוף האדם?‬ ‫של בלוטות הלימפה בסרטן הערמונית‬
‫א‪ .‬אין שוני בהערכה ההיסטולוגית בין‬
‫דגימה מהערמונית לדגימה מאיבר‬ ‫‪ .96‬גבר בן ‪ 70‬עבר ביופסיית מחט של‬
‫אחר בגוף‬ ‫הערמונית‪ .‬בבדיקה מיקרוסקופית‬
‫ב‪ .‬בגידולי הערמונית יש חשיבות רבה‬ ‫התגלו בלוטות עגולות קטנות עם‬
‫יותר לגודל וצורת גרעינון התא‬ ‫גרעינונים בולטים‪ .‬איזו צביעה‬
‫אימונוהיסטוכימית תתמוך באבחנה של‬
‫ג‪ .‬בגידולי הערמונית בודקים את‬ ‫קרצינומה?‬
‫ארכיטקטורת הרקמה ולא‬
‫מסתמכים על מבנה התא הבודד‬ ‫א‪PSA .‬‬
‫ד‪ .‬בהכנת פרפרט הערמונית יש צורך‬ ‫ב‪Basal cell cytokeratin .‬‬
‫בצביעות חומציות עקב הבסיסיות‬ ‫ג‪Alfafetoprotein .‬‬
‫הגבוהה של נוזל הערמונית‬ ‫ד‪Placental alkaline phosphtase .‬‬
‫ה‪ .‬רמת צביעת הרקמה ל‪ PAS-‬שאינו‬ ‫ה‪Leukocyte antigen common .‬‬
‫קיים ברקמות אחרות היא הגורם‬
‫החשוב ביותר לקביעת נוכחות‬ ‫‪ .97‬גבר בן ‪ .68‬בדיקת זרם השתן‬
‫ממאירית‬ ‫מופיעה בגרף‪.‬‬
‫ג‪ .‬מרכז ההשתנה במח אחראי על‬ ‫‪ .100‬מה נכון לגבי ה‪?PSA-‬‬
‫פעילות תקינה של עצבוב דרכי השתן‬ ‫א‪ .‬כל עלייה ברמתו מעל ‪ 4‬נ"ג למ"ל‬
‫התחתונות‬ ‫הינה סגולית לאבחנת סרטן‬
‫ד‪ .‬העצב ההיפוגסטרי אחראי חהגברת‬ ‫הערמונית‬
‫הטונוס של הספינקטר השופכתי‬
‫ה‪ .‬העצב הסומטי )פודנדל( אחראי‬ ‫ב‪ .‬כל עליה מעל ‪ 10‬מחייבת הפניית‬
‫לכיווץ הספינטקר הרצוני‬ ‫החולה מידיית לכריתה רדיקלית‬
‫של הערמונית‬
‫‪ .103‬בהתייחס לתרופות מסוג מעכבי‬ ‫ג‪ .‬זהום ויראלי בדרכי השתן יכול‬
‫אלפא כל הבאים נכונים פרט ל‪:-‬‬ ‫לגרום לירידה בערכי ‪PSA‬‬
‫א‪ .‬השפעתם הקלינית נכרת רק לאחר‬ ‫ד‪ .‬באמצעות ערך ‪ PSA‬ניתן להעריך‬
‫‪ 4-6‬חודשי טיפול‬ ‫באופן סטטיסטי את הסיכוי לגלות‬
‫ב‪ .‬גורמות להופעות אג'קולציה‬ ‫סרטן ערמונית בנבדק‬
‫רטרוגרדית ב‪30% -‬‬ ‫ה‪ .‬בנבדק עם ערכים נמוכים מ‪ 4-‬אין‬
‫ג‪ .‬מקטינות סיכון להתפתחות עצירת‬ ‫סיכוי בכלל לנוכחות סרטן ערמונית‬
‫שתן ב‪50%-‬‬ ‫בבלוטה‬
‫ד‪ .‬גורמות לירידה בנפח הערמונית ב‪-‬‬
‫‪30%‬‬ ‫‪ .101‬סיבוכי כריתה רדיקלית של‬
‫ה‪ .‬גורמות לירידה של ‪ 50%‬בערכי ‪PSA‬‬ ‫הערמונית כוללים את הכל פרט ל‪:‬‬
‫א‪.‬דמם מהפלקסוס של הוריד הפנילי‬
‫‪ .104‬מחסום ה‪ blood testis -‬נוצר‬ ‫ב‪.‬פגיעה בספינקטר החיצוני‬
‫ע"י‪:‬‬ ‫ג‪ .‬פגיעה בסעיפי העצב שמקורם‬
‫א‪ .‬אנדותל קפילרי‬ ‫בפלקסוס הפלבי‬
‫ב‪ .‬תאים מיקסואידים‬ ‫ד‪.‬אטוניה של כיס השתן‬
‫ג‪ BM .‬של ה‪Seminifurous tubuli -‬‬ ‫ה‪ .‬הנקבות של הרקטום‬
‫ד‪ .‬תאי סרטולי‬
‫‪ .102‬לגבי עצבוב שלפוחית השתן‪:‬‬
‫א‪ .‬העצב הסימפטטי הנו אחראי‬
‫להתכווצות השלפוחית‬
‫ב‪ .‬העצב הפלבי הינו כולינרגי והפעלתו‬
‫גורמת להתכווצות שלפוחית השתן‬
‫תשובות‬

‫א‬ ‫‪.71‬‬ ‫ג‬ ‫‪.36‬‬ ‫ג‬ ‫‪.1‬‬


‫ד‬ ‫‪.72‬‬ ‫א‬ ‫‪.37‬‬ ‫ג בוטל‬ ‫‪.2‬‬
‫א‬ ‫‪.73‬‬ ‫ד‬ ‫‪.38‬‬ ‫א‬ ‫‪.3‬‬
‫ד‬ ‫‪.74‬‬ ‫ד‬ ‫‪.39‬‬ ‫ד‬ ‫‪.4‬‬
‫א‬ ‫‪.75‬‬ ‫א‬ ‫‪.40‬‬ ‫ד‬ ‫‪.5‬‬
‫ב‬ ‫‪.76‬‬ ‫ד‬ ‫‪.41‬‬ ‫ד‬ ‫‪.6‬‬
‫א‬ ‫‪.77‬‬ ‫ד‬ ‫‪.42‬‬ ‫ה‬ ‫‪.7‬‬
‫ד‬ ‫‪.78‬‬ ‫א‬ ‫‪.43‬‬ ‫ד‬ ‫‪.8‬‬
‫ד‬ ‫‪.79‬‬ ‫ג‬ ‫‪.44‬‬ ‫ג‬ ‫‪.9‬‬
‫ב‬ ‫‪.80‬‬ ‫ג‬ ‫‪.45‬‬ ‫ב‬ ‫‪.10‬‬
‫ד‬ ‫‪.81‬‬ ‫ב‬ ‫‪.46‬‬ ‫ה‬ ‫‪.11‬‬
‫ב‬ ‫‪.82‬‬ ‫ה‬ ‫‪.47‬‬ ‫‪.12‬‬
‫א‬ ‫‪.83‬‬ ‫ב‬ ‫‪.48‬‬ ‫א‬ ‫‪.13‬‬
‫ד‬ ‫‪.84‬‬ ‫ה‬ ‫‪.49‬‬ ‫ד בוטל‬ ‫‪.14‬‬
‫ג‬ ‫‪.85‬‬ ‫ה‬ ‫‪.50‬‬ ‫ג‬ ‫‪.15‬‬
‫ב‬ ‫‪.86‬‬ ‫ה‬ ‫‪.51‬‬ ‫ג‬ ‫‪.16‬‬
‫א‬ ‫‪.87‬‬ ‫ד‬ ‫‪.52‬‬ ‫ג‬ ‫‪.17‬‬
‫ג‬ ‫‪.88‬‬ ‫ג‬ ‫‪.53‬‬ ‫ג‬ ‫‪.18‬‬
‫ג‬ ‫‪.89‬‬ ‫ב‬ ‫‪.54‬‬ ‫ג‬ ‫‪.19‬‬
‫ד‬ ‫‪.90‬‬ ‫א‬ ‫‪.55‬‬ ‫א בוטל‬ ‫‪.20‬‬
‫ד‬ ‫‪.91‬‬ ‫ב‬ ‫‪.56‬‬ ‫ג‬ ‫‪.21‬‬
‫ב‬ ‫‪.92‬‬ ‫ד‬ ‫‪.57‬‬ ‫ב‬ ‫‪.22‬‬
‫ד‬ ‫‪.93‬‬ ‫ד‬ ‫‪.58‬‬ ‫ד‬ ‫‪.23‬‬
‫א‪,‬ג‬ ‫‪.94‬‬ ‫א‬ ‫‪.59‬‬ ‫ב‬ ‫‪.24‬‬
‫ד‬ ‫‪.95‬‬ ‫ד‬ ‫‪.60‬‬ ‫ב‬ ‫‪.25‬‬
‫ב‬ ‫‪.96‬‬ ‫ה‬ ‫‪.61‬‬ ‫ב‬ ‫‪.26‬‬
‫ב‬ ‫‪.97‬‬ ‫ג‬ ‫‪.62‬‬ ‫ג‬ ‫‪.27‬‬
‫ה‬ ‫‪.98‬‬ ‫ג‬ ‫‪.63‬‬ ‫ד‬ ‫‪.28‬‬
‫ג‬ ‫‪.99‬‬ ‫א‬ ‫‪.64‬‬ ‫ב‬ ‫‪.29‬‬
‫ד‬ ‫‪.100‬‬ ‫ג‬ ‫‪.65‬‬ ‫ב‬ ‫‪.30‬‬
‫ד‬ ‫‪.101‬‬ ‫ה‬ ‫‪.66‬‬ ‫ד‬ ‫‪.31‬‬
‫א‬ ‫‪.102‬‬ ‫ב‬ ‫‪.67‬‬ ‫ה‬ ‫‪.32‬‬
‫ב‬ ‫‪.103‬‬ ‫א‬ ‫‪.68‬‬ ‫ה‬ ‫‪.33‬‬
‫ב‬ ‫‪.104‬‬ ‫ב‬ ‫‪.69‬‬ ‫ה‬ ‫‪.34‬‬
‫א‬ ‫‪.70‬‬ ‫ג‬ ‫‪.35‬‬
‫)אנדוקרינולוגיה‪ ,‬המטואונקולוגיה‪ ,‬נשים‪ ,‬שד‪ ,‬אשך‪ ,‬פרוסטטה ומחלות זיהומיות(‬

‫מקבץ ‪ - 3‬אוסף שאלות‬

‫מכילה תמיד פפילות‬ ‫ד‪.‬‬ ‫‪ .1‬סמן את הסעיף הנכון בקשר לסיבוכים‬


‫בסוכרת‪:‬‬
‫‪Minimally invasive follicular .6‬‬ ‫א‪ .‬גנגרנה ‪ gangrene‬בגפיים‬
‫‪:carcinoma‬‬ ‫התחתונות‪ ,‬אינה שכיחה יותר אצל‬
‫א‪ .‬בדר"כ קשרית בודדת‬ ‫חולים סוכרתיים כאשר משווים‬
‫אוכלוסיה באותו גיל‬
‫ב‪ .‬נראית חדירה של הגידול לקופסית‬
‫או לכלי הדם‬ ‫ב‪ .‬מבחינה היסטולוגית אין הבדל‬
‫באתרוסקלרוזיס אצל חולים‬
‫ג‪ 10 .‬שנות הישרדות מעל ‪90%‬‬ ‫סוכרתיים ולא סוכרתיים‬
‫ד‪ .‬א ‪ +‬ב ‪ +‬ג נכונים‬ ‫ג‪Hyaline arteriolosclerosis .‬‬
‫ה‪ .‬כל התשובות אינן נכונות‬ ‫פתוגנומונית לסוכרת‬
‫ד‪Membranous .‬‬
‫‪:Reidel Thyroiditis .7‬‬ ‫‪ glomerulonephritis‬עם משקעים‬
‫א‪ .‬בלוטת תריס מוגדלת ונוקשה‪,‬‬ ‫אלקטרון‪-‬דנס סובאנדותליאלים‬
‫מלווה לעיתים קרובות ב‪Dyspnea -‬‬ ‫אופיינית ל‪diabetic nephropathy -‬‬
‫ב‪ .‬שכיחה יותר בנשים צעירות‬
‫ג‪ .‬בהיסטולוגיה נראים תאי ענק‬ ‫‪ .2‬סמן מה נכון בקשר לפתוגנזה של‬
‫ד‪ .‬קשור ל‪HLA B8 -‬‬ ‫‪:Diabetic neuropathy‬‬
‫א‪ .‬שמירה על רמת סוכר תקין בדם‬
‫‪:Parathyroid Adenoma .8‬‬ ‫אינה משפיעה על התפתחות‬
‫א‪ .‬מערבת בדר"כ רק בלוטה אחת‬ ‫הסיבוכים‬
‫ב‪ .‬באדנומה בניגוד לרקמת‬ ‫ב‪ Vasculitis .‬גורם לניוון העצבים‬
‫פאראתירואיד תקינה‪ ,‬תאי השומן‬ ‫הפריפרים‬
‫אינם בולטים‬ ‫ג‪Microangiopathy and .‬‬
‫ג‪ .‬מחיצות פיברוטיות מעובות‬ ‫‪ Demyelinization‬הם המנגנונים‬
‫אופייניות‬ ‫העיקריים בפתוגנזה‬
‫ד‪ .‬א‪+‬ב נכונים‬ ‫ד‪ .‬המחסור באינסולין מביא לירידת‬
‫גלוקוז משמעותית בתאי שוואן‬
‫‪ .9‬מה לא נכון לגבי ‪? Pheochromocytoma‬‬ ‫‪Schwan cells‬‬
‫א‪ .‬מפריש ‪Catecholamines‬‬
‫‪ .3‬אחד הבאים אינו נחשב כסיבוך עיניים של‬
‫ב‪ .‬רובם ממאירים‬
‫סוכרת )‪:(Diabetic Ocular Complication‬‬
‫ג‪ .‬הסטרומה עשירה בכלי דם‬
‫א‪Proliferative retinopathy .‬‬
‫ד‪ .‬בגידולים שפירים יכולה להיות‬
‫חדירה לקופסית ולכלי דם‬ ‫ב‪Non-Proliferative retinopathy .‬‬
‫ה‪ .‬רובם מתחילים במדולה של בלוטת‬ ‫ג‪Pteryoium .‬‬
‫יותרת הכליה‬ ‫ד‪Cataract .‬‬

‫‪ .10‬מה נכון לגבי ‪? Coon Syndrome‬‬ ‫‪ .4‬סלייד – ‪ - hashimoto thyroiditis‬בוטל‬


‫א‪ .‬הסיבה יכולה להיות אדנומה של‬
‫ההיפופוזה‬ ‫‪ .5‬בחר‪/‬י את התשובה הנכונה‪Papillary ,‬‬
‫ב‪ .‬הסיבה יכולה להיות אדנומה של‬ ‫‪:Carcinoma‬‬
‫האדררנל‬ ‫א‪ .‬שכיחה ביותר מעל גיל ‪70‬‬
‫ג‪ .‬רמה נמוכה של ‪Renin‬‬ ‫ב‪ .‬בדר"כ מוקפת ע"י קופסית‬
‫ד‪Hyperaldesteronemia .‬‬ ‫ג‪ .‬גרורות לקשרי לימפה שכיחות‬
‫ב‪ .‬חשיפה מוקדמת לחלב פרה‬ ‫ה‪ .‬עליה לחץ דם‬
‫ג‪ .‬מחסור בויטמין ‪D‬‬
‫ד‪ .‬השמנת יתר בגיל ההתבגרות‬ ‫‪ .11‬סלייד‬

‫‪ .17‬בסוכרת מסוג ‪:I‬‬ ‫‪ .12‬מה נכון לגבי הסיבות ל‪-‬‬


‫א‪ .‬תרופתית ניתן לטפל רק באינסולין‬ ‫‪? Hypopituitarism‬‬
‫ב‪ .‬בדר"כ הטיפול הראשוני הינו תרופה‬ ‫א‪ .‬אדנומה של ההיפופיזה‬
‫מקבוצת הסולפוניל אוראה‬ ‫ב‪Empty Sella Syndrome .‬‬
‫ג‪ .‬שילב תרופות דרך הפה ‪ +‬אינסולין‬ ‫ג‪ .‬גרורה של סרטן‬
‫בזריקות‬ ‫ד‪Craniopharyngioma .‬‬
‫ד‪ .‬אם מדובר בחולה מתחת לגיל ‪ ,20‬מומלץ‬ ‫ה‪ .‬כל התשובות נכונות‬
‫להתחיל אינסולין‪ .‬מעל גיל זה תרופות דרך‬
‫הפה‪.‬‬ ‫‪ .13‬לגבי ‪ , Adenohypophysa‬סמן את‬
‫התשובה הלא נכונה‪:‬‬
‫‪ .18‬סכרת נעורים‪ -‬מה נכון?‬
‫א‪ .‬הינה מחלה תורשתית המועברת בצורה‬ ‫א‪ Sheehan Syndrome .‬הינה הסיבה‬
‫אוטוזומאלית רצסיבית‪.‬‬ ‫השכיחה ל‪Daibetes Insipidus -‬‬
‫ב‪ HLADR1 .‬מופיע בשכיחות יתר אצל‬ ‫ב‪ .‬אנדומה של ההיפופיזה יכולה להיות‬
‫חולים אלה‬ ‫חלק מה‪Men Syndrome -‬‬
‫ג‪ .‬המחלה שכיחה יותר אצל בנות מאשר‬ ‫ג‪ Prolactinoma .‬הינה האדנומה‬
‫אצל בנים‪ ,‬כמו ברוב המחלות‬ ‫הפונקציונאלית השכיחה‬
‫האוטואימוניות‪.‬‬ ‫ד‪ .‬באדנומות גדולות אפשר לראות‬
‫ד‪ .‬כאשר אחד מתאומים זהים מפתח את‬ ‫דימומים ונמק‬
‫המחלה‪ ,‬הסיכוי שהשני יפתח מחלה הוא‬ ‫ה‪ .‬דימום בתוך אדנומה קשור ל‪-‬‬
‫‪ , 40%‬מה שמעיד על חשיבות הגורם‬ ‫‪Pituitary Apoplexy‬‬
‫הסביבתי‬
‫‪ .14‬אישה בת ‪ 30‬מתלוננת על עייפות ועליה‬
‫‪ .19‬ב‪ CT -‬בטן שבוצע מחלק מבירור כאבי‬ ‫במשקל‪ .‬בבדיקה גופנית נמצאה קשרית‬
‫בטן‪ ,‬נמתא באדרנל תהליך תופס מקום‬ ‫באונה הימנית‪ .‬בוצע ניקור מהקשרית‬
‫בקוטר ‪ 2‬ס"מ‪ .‬איזו מהבדיקות הבאות אין‬ ‫שהראה תאי וקטעי אפיתל פוליקולרי‪,‬‬
‫צורך לבצע?‬ ‫אונקוציטים‪ ,‬לימפוציטים רבים‪ ,‬תאי‬
‫א‪ .‬מבחן סטימולציה עם ‪ACTH‬‬ ‫פלסמה‪ ,‬מקרופאגים‪ ,‬תאים רב‪ -‬גרעיניים‬
‫ב‪ .‬איסוף שתן ‪ 24‬שעות לקטכולאמינים‬ ‫וקולואיד‪ .‬הממצא עשוי להתאים ל‪:-‬‬
‫ג‪ .‬בדיקת דם לרנין‬ ‫א‪Hashimoto’s Thyroiditis .‬‬
‫ד‪ .‬איסוף שתן ‪ 24‬שעות לקורטיזול‬ ‫ב‪Goiter .‬‬
‫ג‪Lymphoma .‬‬
‫‪ .20‬בבדיקה תקופתית של גבר בן ‪ 30‬נמדד‬ ‫ד‪Acute Thyroiditis .‬‬
‫לחץ דם של ‪ . 170/110‬על איזה מהבדיקות‬
‫הבאו תמליץ בשלב ראשון?‬ ‫ה‪Follicular Adenoma .‬‬
‫א‪ .‬בדיקת דם לרנין ואלדוסטרון לאחר‬
‫העמסת מלח‬ ‫‪ .15‬אישה בת ‪ 40‬מתלוננת על נפיחות‬
‫ב‪ .‬בדיקת דם לרנין ואלדוסטרון‬ ‫בקדמת הצוואר בזמן האחרון‪ .‬תפקודי‬
‫ג‪ .‬בדיקת דם לרנין ואלדוסטרון בשכיבה‪,‬‬ ‫בלוטת התריס תקינים‪ .‬הבלוטה נראת‬
‫ולאחר עמידה‪/‬הליכה של ‪ 4‬שעות‬ ‫מוגדלת‪ .‬בוצע ניקור שהראה תאי וקטעי‬
‫ד‪ .‬בדיקות דם לאשלגן‬ ‫אפיתל פוליקולרי‪ ,‬חלקם במבנים‪,‬‬
‫אונקוציטים‪ ,‬מקרופאגים וקולואיד‪.‬‬
‫‪.21‬איזה מהאבחנות הבאות אינה מסבירה‬ ‫הממצא עשוי להתאים ל‪:-‬‬
‫התגברות היפרפיגמנטציה בקפלי העור‬ ‫א‪Hashimoto’s Thyroiditis .‬‬
‫ובריריות?‬ ‫ב‪Papillary Carcinoma .‬‬
‫א‪ .‬מחלת אדיסון‬ ‫ג‪Hartle Cell Adenoma .‬‬
‫ב‪ .‬מחלת ‪ 21-hydroxylase deficiency‬לא‬ ‫ד‪Adenocarcinoma Goiter .‬‬
‫מאוזנת‬ ‫ה‪Hartle Cell Carcinoma .‬‬
‫ג‪ .‬מחלת נלסון‬
‫ד‪MEN type 2b .‬‬ ‫‪ .16‬כל הבאים יכולים להיות טריגר‬
‫להתפתחות סכרת מסוג ‪ ,I‬פרט ל‪:-‬‬
‫א‪ .‬מחלות ויראליות‬
‫ג‪Hyperaldosteronism .‬‬ ‫‪ .22‬איזה מהמצבים מהבאים אינו יכול‬
‫ד‪Primary Hypoparathyroidism .‬‬ ‫להסביר ‪ False Positive‬במבחן‬
‫‪Overnight Dexamethasone‬‬
‫‪ .28‬איזה מהמשפטים הבאים לגבי תהליך‬ ‫‪? Suppression Test‬‬
‫בירור של קשרית בבלוטת התריס אינו נכון ?‬ ‫א‪ .‬מחלה גסטרואינטסטינאלית בו זמנית‬
‫א‪ .‬בדיקת ה‪ Fine needle aspiration -‬הינה‬ ‫ב‪ .‬נטילה ‪ ,‬בטעות‪ ,‬מינון כפול של דקסמתזון‬
‫מבדיקה בטוחה‪ ,‬מהימנה וספציפית לאבחון‬ ‫ג‪ .‬נטילה בו זמנית של תרופה המגבירה את‬
‫ממאירויות בבלוטת התריס‪.‬‬ ‫קצב פירוק תרופות בכבד‬
‫ב‪ .‬באמצעות הבדיקה לא ניתן להבדיל בין‬ ‫ד‪ .‬נטילה בו זמנית של קוטיזול‬
‫גידול ממאיר לשפיר‬
‫ג‪ .‬מיפוי בלוטת התריס יכול להוות תחליף ל‪-‬‬ ‫‪ .23‬אזיה מהבאים אינו סיבה לתמותה‬
‫‪ ,Fine needle aspiration‬בחולים עם רמת‬ ‫בתסמונת קושינג?‬
‫א‪ .‬מחלות לב‬
‫‪ TSH‬מעל הנורמה‬ ‫ב‪ .‬תרומבואמבוליזם‬
‫ד‪ .‬ל‪ 11-20% -‬חולים נמצאה קשרית‬ ‫ג‪ .‬זיהום‬
‫החשודה לממאירות ב‪Fine cytology-‬‬ ‫ד‪ .‬היפרקלצמיה‬
‫‪needle aspiration‬‬
‫מתוכן כ‪ 20% -‬מתגלות כממאירות‬ ‫‪ .24‬חולה עם ‪non-classical 21‬‬
‫‪ hydroxylase deficiency‬שנמצאה‬
‫‪ .29‬גבר בן ‪ 55‬מתלונן על עייפות‪ ,‬חולשה‪,‬‬
‫הזעת יתר‪ ,‬כאבי ראש וכאבים בברכיים‪,‬‬ ‫הומוזיגוטית למוטציה קלה בגן ל ‪21‬‬
‫שהחמירו בהדרגה במהלך השנה‪ .‬כמו כן‬ ‫‪ . hydroxylase‬פונה אלייך כי בהריון‬
‫הבחין בנפיחות באצבעותיו ונאלץ להרחיב‬ ‫וחוששת שאם העובר היא נקבה‪ ,‬היא תיוולד‬
‫את טבעת הנישואין‪ .‬אזיו בדירה הכרחית‬ ‫עם גניטליה אמביוולנטית‪ .‬גישתכם היא‪:‬‬
‫לאבון המחלה ?‬ ‫א‪ .‬להמשיך טיפול בפרדניזון במהלך ההריון‪,‬‬
‫א‪ .‬בדיקת רמת הורמון גדילה בסרום בצום‬ ‫ולעקוב אחר רמות טסטוסטרון ו‪17 -‬‬
‫ב‪ .‬בדיקת יחס הורמון גדילה וגלוקוז בסרום‬ ‫הידרוקסי‪-‬פרוגסטרון‬
‫ג‪ .‬בדיקת ‪ IGF1‬והורמון גדילה בסרום‬ ‫ב‪ .‬להמליץ לבדוק את אבי העובר‬
‫ג‪ .‬להמליץ לבצע בדיקת סיסי שילייה לפני‬
‫ד‪ MRI .‬של ההיפופיזה‬ ‫שבוע ‪ 8‬להריון‬
‫ה‪ .‬אף תשובה אינה נכונה‬ ‫ד‪ .‬להמליץ על טיפול בדקסמתזון שיופסק‬
‫רק אם‪ .1 :‬יתברר שהעובר זכר או ‪ .2‬העובר‬
‫פרק מספר ‪ :2‬נשים‪ ,‬שד‬ ‫נקבה ויתברר שרמת ה‪ 17 -‬הידרוקסי‪-‬‬
‫פרוגסטרון בנוזל מי שפיר תקינה‪.‬‬
‫‪ Dysgerminoma .30‬של השחלה מקורה‬
‫ב‪ ) : -‬סמן את התשובה הנכונה(‬ ‫‪ .25‬איזה מהמשפטים הבאים נכון לגבי ‪21‬‬
‫א‪ .‬אפיתל צלומי )‪( Celomic epithelium‬‬ ‫‪? hydroxylase deficiency‬‬
‫ב‪Sex cords .‬‬ ‫א‪ .‬יש ‪HLA linkage‬‬
‫ג‪Germ cells .‬‬ ‫ב‪ .‬בצורה של ‪ salt wasting‬יש פגם ב‪zona -‬‬
‫ד‪ .‬אפיתל אורוטליאלי‬
‫ה‪ .‬טרופובלסט‬ ‫‪ glomerulosa‬אבל לא ב‪zona fasiculata -‬‬
‫ג‪ .‬ניתן לאבחן נשאים הטרוזיגוטיים ע"י‬
‫‪ .31‬הסיבות ל‪Endometrial Hyperplasia -‬‬ ‫מבחן סטימולציה עם ‪ACTH‬‬
‫‪) :‬סמן את הלא נכון(‬ ‫ד‪ .‬מצב זה שכיח יותר ביהודים יוצאי צפון‬
‫אפריקה‬
‫א‪Stein- Leventhal syndrome .‬‬
‫ב‪Granulosa cell tumor of ovary .‬‬ ‫‪ .26‬בגבר עם מחלת אדיסון יתכנו כל‬
‫ג‪ .‬שימוש ממושך בפרוגסטרון‬ ‫הסימפטומים והסיבוכים הבאים‪ ,‬פרט ל‪:-‬‬
‫ד‪ .‬שימוש ממושך באסטרוגן‬ ‫א‪ .‬אנמיה‬
‫ב‪ .‬אנורקסיה‬
‫‪ Botryoid Sarcoma .32‬הבחנה ל‪:-‬‬ ‫ג‪ .‬אובדן שיעור אקסילרי‬
‫א‪ Leiomyosarcom .‬של הרחם‬ ‫ד‪ .‬ויטליגו‬
‫ב‪ Fibrosarcoma .‬של השחלה‬
‫ג‪ Rhabdomyosarcoma .‬של הנרתיק‬ ‫‪ .27‬באיזה מהמצבים הבאים עשויה להופיע‬
‫פוליאוריה?‬
‫ד‪ Sex cord tumor .‬של השחלה‬
‫א‪Myxedema coma .‬‬
‫ה‪ Mixed mesodermal tumor .‬של הרחם‬
‫ב‪Primary Hyperparathyroidism .‬‬
‫ג‪ .‬סוג של ‪in situ carcinoma‬‬ ‫‪ PTEN mutation .33‬נחשב כפקטור גורם‬
‫ד‪ .‬כל הנ"ל אינם נכונים‬ ‫ל‪:-‬‬
‫א‪Trophoblastic disease .‬‬
‫‪ .42‬כיצד נעשת הערכת גובה הראש בתעלת‬ ‫ב‪Serous cystadenoma .‬‬
‫הלידה?‬
‫א‪ .‬בבדיקה גופנית לפי מיקום קצה הגלגלת‬ ‫ג‪Cervical dysplasia .‬‬
‫ביחס ל‪ischiatic spines -‬‬ ‫ד‪Endometrial hyperplasia .‬‬
‫ב‪ .‬בבדיקת ‪ US‬לפי מיקום קצה הגלגלת‬ ‫ה‪Immature teratoma .‬‬
‫ביחס ל‪ischiatic spines -‬‬
‫ג‪ .‬בבדיקה ע"י מוניטור עוברי לפי מיקום‬ ‫‪ .34‬איזה מהבאים שייך לקבומת ה‪Sex -‬‬
‫קצה הגלגלת ביחס ל‪ischiatic spines -‬‬ ‫‪? cord stromal tumors‬‬
‫ד‪ .‬בבידקת ‪ US‬לפי מדידת גובה קרקעית‬ ‫א‪Immature teratoma .‬‬
‫הרחם מהסימפיזיס פוביס‬ ‫ב‪Dermatoid cyst .‬‬
‫ג‪Granulosa cell tumor .‬‬
‫‪ .43‬בוטל‬ ‫ד‪Dysgerminoma .‬‬
‫ה‪Brenner Tumor .‬‬
‫‪ .44‬הטיפול ב‪ - Lichen Sclerosus -‬הוא‪:‬‬
‫א‪ .‬אנטיביוטיקה מהסוג פלג'יל )מטרונידזול(‬ ‫‪ Ectopic Pregnancy .35‬ניתן לראות בתוך‬
‫בכדורים דרך הפה‬ ‫הרחם ‪:‬‬
‫ב‪ .‬כריתת הנגע בערייה‬ ‫א‪ .‬סינציטיוטרופובלסט‬
‫ג‪ .‬משחה או קרם של דרמובט‪ -‬קלובטאזול‬ ‫ב‪ .‬היפרפלזיה של האנדומטריום‬
‫ד‪ .‬כדורי אנטיביוטיקה מהסוג אריתרומיצין‬
‫ג‪ .‬שינויים ע"ש ‪ Arias Stella‬בבלוטות‬
‫ה‪ .‬מריחת ‪KOH‬‬ ‫ד‪.‬‬
‫ה‪ .‬סיסי שילייה בתוך הרחם‬
‫‪ .45‬סמן את התשובה הנכונה ביותר לגבי‬
‫‪: Uterine sarcoma‬‬ ‫‪ .36‬סלייד‬
‫א‪ .‬סרקומות של הרחם נדירות ומהוות ‪3%‬‬
‫מהסרטנים ברחם‬
‫ב‪ .‬לסרקומה של הרחם פרוגנוזה טובה ביחס‬ ‫‪ Phyllodes tumor .37‬של השד‪ -‬סמן את‬
‫הנכון‪:‬‬
‫ל‪Endometrial carcinoma -‬‬ ‫א‪ .‬מראה אטיפיה קשה באפיתל הדוקטולרי‬
‫ג‪ .‬רוב הסרקומות של הרחם מתאפיינות‬ ‫ב‪ .‬מראה אטיפיה קשה באפיתל הלובולרי‬
‫בתגובה מצויינת לכמותרפיה‬ ‫ג‪ .‬כמעט תמיד שולח גרורות לבלוטות לימפה‬
‫ד‪ .‬סרקומה של הרחם היא הגידול השני‬ ‫ולמקומות מרוחקים‬
‫בשכיחותו אצל אישה בגיל המעבר‬ ‫ד‪ .‬רואים בגידול ריבוי של תאים סטרומלים‬
‫ה‪ .‬גידול קטן‬
‫‪ .46‬בניתוח ‪Radical Hysterectomy‬‬
‫מבצעים את כל הפעולות הבאות‪ ,‬פרט ל‪:-‬‬ ‫‪ .38‬בוטל‬
‫א‪ .‬כריתת נרתיק בחלק העליון‬
‫ב‪ .‬כריתת שתי השחלות‬ ‫‪ ,Lobular carcinoma .39‬סמן את הנכון‪:‬‬
‫ג‪ .‬כריתת רחם מלאה וחלק מה‪-‬‬ ‫א‪ .‬מראה הסננה של תאים בודדים לתוך‬
‫‪parametrium‬‬ ‫סטרומה‬
‫ד‪ .‬כריתת החלק הדיסטלי של ה‪-‬‬ ‫ב‪ .‬מאופיינת עי" איבוד של ‪Cell Adhesion‬‬
‫‪sacrouterine ligament‬‬ ‫‪Molecule E-Cadherin‬‬
‫ה‪ .‬כריתת הנרתיק בחלק העליון וכריתת‬ ‫ג‪ .‬ברוב המקרים חיובית לרצפטורים‬
‫שתי השחלות‬ ‫לאסטרוגן ופרוגסטרון‬
‫ד‪ .‬בהשוואה עם סוגים אחרים של סרטן‬
‫‪ .47‬הסמן ‪: CA-125‬‬ ‫השד שכיחות הבילטרליות גבוהה יותר‬
‫א‪ .‬הוא סמן של סרטן השחלות בלבד‬ ‫ה‪ .‬כולם נכונים‬
‫ב‪ .‬עולה בסרטן השחלות וסרטן פריטוניאלי‬
‫בלבד‬ ‫‪ .40‬סלייד‬
‫ג‪ .‬בדיקה טובה לצורך ‪ screening‬לסרטן‬
‫השחלות‬ ‫‪ .41‬לגבי המחלה שבסלייד ‪:‬‬
‫ד‪ .‬יכול להיות גבוה גם ב‪Endometrial -‬‬ ‫א‪ .‬לעיתים נדירות שולחת גרורות לבלוטות‬
‫‪carcinoma‬‬ ‫לימפה‬
‫ב‪ .‬לעיתים נדירות שולחת גרורות לעצמות‬
‫‪: Dysgerminoma of the ovary .55‬‬ ‫ה‪ .‬בדר"כ הרמות של ‪ CA-125‬נמוכות‬
‫א‪ .‬גידול אפיתליאלי שכיח שקורה בנשים‬ ‫כאשר מדובר בסרטן שחלה ‪stage I‬‬
‫צעירות‬
‫ב‪ .‬אופייני בעיקר אצל נשים בגיל ‪10-30‬‬ ‫‪ .48‬מה קשור ל‪? PCOS -‬‬
‫ג‪ .‬לא מגיב לכמותרפיה‬ ‫א‪ .‬סרטן המעי הגס‬
‫ד‪ .‬הגידול מלווה בהופעת שיעור יתר‬
‫ה‪ .‬לרוב בילטרלי‬ ‫ב‪ .‬סרטן צואר הרחם‬
‫ג‪ .‬דיספלזיה או אטיפיה של צואר‬
‫‪ .56‬מה אינו גורם סיכון להיפרדות שיליה?‬ ‫הרחם ו‪HPV -‬‬
‫א‪ .‬עישון‬ ‫ד‪ .‬סרטן האנדומטריום‬
‫ב‪ .‬יתר לחדץ דם‬ ‫ה‪ .‬גלאוקומה‬
‫ג‪ .‬שרירן ברחם‬
‫ד‪ .‬הריון מרובה עוברים‬ ‫‪ .49‬בנוגע לתסמונת ‪ ,Down‬מהו המשפט‬
‫ה‪ .‬סוכרת הריונית‬ ‫הנכון ?‬
‫א‪ .‬הסיכון הכללי להישנות טריזומיה ‪21‬‬
‫‪ .57‬בת ‪ ,25‬הריון ראשון שבוע ‪ ,35‬הגיעה‬ ‫אצל אישה שכבר ילדה תינוק פגוע‬
‫באמבולנס לביה"ח‪ ,‬בגלל דימום נרתיקי‬ ‫הוא נמוך‪ ,‬בגבולות ‪.1%‬‬
‫מוגבר‪ .‬בקבלתה לחץ דם ‪ ,75/45‬דופק ‪,135‬‬ ‫ב‪ .‬לצורך מתן ייעוץ גנטי להורים‪ ,‬ניתן‬
‫האישה בהכרה‪ .‬הטיפול באישה כולל את כל‬ ‫להסתפק בקביעת האבחנה על בסיס‬
‫הבאים פרט ל‪:-‬‬ ‫קליני כשהתמונה היא מובהקת‪.‬‬
‫א‪ .‬קריסטלואידים‬
‫ב‪ .‬פקטורי קרישה‬ ‫ג‪ .‬הגורם לטריזומיה ‪ 21‬הוא טעות‬
‫ג‪ .‬כדוריות אדומות‬ ‫במיוזיס האימהי תמיד‪ ,‬גם אם‬
‫ד‪ .‬משתנים‬ ‫קיימת טרנסלוקציה אצל אחד‬
‫ה‪ .‬השכבת האישה על צד שמאל‬ ‫ההורים כגון ‪Robertsonian 14/21‬‬
‫‪.translocation‬‬
‫‪ .58‬מומלץ לבצע לפורוסקופיה במסגרת‬ ‫ד‪ .‬הסיכוי להישנות תסמונת ‪ Down‬בכל‬
‫בירור אי פוריות בכל המצבים הבאים פרט‬ ‫הריון הוא בגבולות ‪50%‬‬
‫ל‪:-‬‬
‫א‪ .‬ממצאים לא תקינים בצילום רחם‬ ‫‪ .51‬בוטל‬
‫ב‪ .‬חשד לדלקת כרונית של צוואר הרחם‬
‫ג‪ .‬צילום רחם תקין‪ ,‬אך ללא הריון במשך‬ ‫‪ .52‬בכל המצבים הבאים יש יתרונות ברורים‬
‫שנה‪.‬‬ ‫בשימוש גלולה פרט ל‪:‬‬
‫ד‪ .‬רגישות ליוד‬ ‫א‪ .‬ירידה בשיעור ‪Endometrial carcinoma‬‬
‫ה‪ .‬סיפור של ‪ PID‬בעבר‬ ‫ב‪ .‬ירידה בשיעור ‪Cervical carcinoma‬‬
‫‪ .59‬השכיחות של קריוטיפ לא תקין בגברים‬ ‫ג‪ .‬ירידה בשיעור ‪Epithelial ovarian‬‬
‫לא פוריים עם אזוספרמיה היא‪:‬‬ ‫‪carcinoma‬‬
‫א‪2-5% .‬‬ ‫ד‪ .‬ירידה בשיעור ‪Ectopic pregnancy‬‬
‫ב‪10-15% .‬‬
‫ג‪20-25% .‬‬ ‫‪ .53‬כל המצבים הבאים הם גורמי סיכון‬
‫ד‪ .‬תלויה בנוכחות המוטציה לציסטיק‬ ‫לסרטן הרחם פרט ל‪:-‬‬
‫פיברוסיס‬ ‫א‪ .‬עודף משקל‬
‫ה‪ .‬פחות מ‪ 1%-‬אם אין סיפור משפחתי של‬ ‫ב‪ .‬טיפול באסטרוגנים‬
‫אי פוריות‬ ‫ג‪ .‬טמוקסיפן‬
‫ד‪ .‬יתר לחץ דם‬
‫‪ .60‬תחום התדירויות השכיח במתמר‬ ‫ה‪ .‬סיפור משפחתי‬
‫וגינאלי‪:‬‬
‫א‪ 2-3 .‬מגה‪-‬הרץ‬ ‫‪ .54‬מי מהפקטורים הבאים מהווה גורם‬
‫ב‪ 3-5 .‬מגה‪-‬הרץ‬ ‫סיכון לסרטן השחלה?‬
‫ג‪ 20-30 .‬מגה‪-‬הרץ‬ ‫א‪ .‬שימוש בגלולות מעל ‪ 5‬שנים‬
‫ד‪ 30-50 .‬מגה‪-‬הרץ‬ ‫ב‪ .‬סיפור קודם של סרטן השד‬
‫ג‪ .‬שימוש בטיפול הורמונלי תחליפי מעל ‪5‬‬
‫‪ .61‬הערכה סונוגרפית של גיל העובר בשליש‬ ‫שנים‬
‫הראשון להריון מדויוקת ביותר במדידה‬ ‫ד‪ .‬יתר לחד דם‬
‫של‪:‬‬ ‫ה‪ .‬סוכרת‬
‫א‪ .‬מרחק ראש עכוז‬
‫ב‪ .‬היקף הראש‬
‫‪ .67‬בבדיקה היסטולוגית בתמונה מספר ‪-1‬‬ ‫ג‪ .‬היקף הבטן‬
‫הממצא האופייני‪:‬‬ ‫ד‪ .‬קוטר דו קודקודי‬
‫א‪ .‬מבנה זקיקי )פוליקולרי(‬
‫ב‪ .‬נוכחות של קבוצות של תאים יותר‬ ‫‪ .62‬מה לא נכון לגבי נזק נוירולוגי מאוחר?‬
‫גדולים שנקראים ”‪“proliferation center‬‬ ‫א‪ .‬היפוקסיה בפני עצמה אינה גוררת נזק‬
‫ג‪ .‬פיברוזיס עדין והתעבות הקופסית‬ ‫נוירולוגי‬
‫ד‪ .‬נוכחות של היסטיוציטים רבים עם יצירת‬ ‫ב‪ .‬מצב של תשניק מאופיין ע"י חמצת‬
‫מבנה של ”‪“Starry Sky‬‬ ‫מטבולית‬
‫ג‪ .‬חמצת מטבולית עשויה לגרום לפגיעה רב‬
‫‪ .68‬ילד בן ‪ 8‬של עובר זר מאוגנדה מגיע‬ ‫מערכתית בעובר ובילוד‬
‫למיון עקב נפיחות בלסת ימנית תחתונה ‪,‬‬ ‫ד‪ .‬במרבית המקרים של שיתוק מוחין ניתן‬
‫שגדלה במהירות תוך כדי ימים אחדים‪.‬‬ ‫לזהות סימנים של מצוקת העובר סביב‬
‫בבדיקה אין חום‪ .‬תמונת דם היקפי תקינה‪.‬‬ ‫הלידה‬
‫בוצעה ביופסיה מהגוש בלסת התחתונה‪.‬‬
‫בבדיקת ‪ Flow cytometry‬תאי הגידול‬ ‫‪: Herceptin .63‬‬
‫א‪ .‬טיפול הורמונלי הניתן בסרטן שד‬
‫נצבעו בין היתר ‪ CD10/Bcl-6/CD20‬חיובי‪.‬‬ ‫מתקדם‬
‫בבדיקה היסטולוגית תמונה מספר ‪– 2‬‬
‫ב‪ .‬כאשר ‪ HER2‬מבוטא בעודף אין לתת‬
‫צביעת ‪: H&E‬‬
‫מה נכון לגבי המחלה?‬ ‫‪ Herceptin‬כיון שאין סיכוי לחסימת כל‬
‫א‪ .‬תאי הגידול נצבעים אימונוהיסטווכימית‬ ‫הרצפטורים‪ ,‬בממילא לא יעיל‬
‫ל‪TdT -‬‬ ‫ג‪ .‬אסור לתת ‪ Herceptin‬בשילוב עם‬
‫ב‪ .‬תאי הגידול נצבעים אימונוהיסטוכימית‬ ‫כמותתרפיה בגלל רעילות בלתי נסבלת‬
‫ל‪Myeloperoxidase -‬‬ ‫ד‪ .‬זהו ‪ Humanized Ab‬ספציפי כנגד ‪HER2‬‬
‫ג‪ .‬בתאי הגידול נמצא ‪ EBV‬ברוב המקרים‬
‫ד‪ .‬המחלה לא מגיבה לטיפול כמותרפי‬ ‫‪ .65‬האבחנה הסבירה ביותר בממוגרפיה של‬
‫גוש עגול עם הסתיידויות בצורת פופקורן‪:‬‬
‫‪ .69‬מה האבחנה?‬ ‫א‪ .‬פפילומה‬
‫ב‪ .‬פיברואדנומה‬
‫א‪T cell acute Lymphoblastic .‬‬ ‫ג‪ .‬גוש שומני‬
‫‪Leukemia‬‬ ‫ד‪ .‬המנגיומה‬
‫ב‪Acute Myeloid Leukemia .‬‬ ‫ה‪DCIS .‬‬
‫ג‪Plasma cell Myeloma .‬‬
‫ד‪Berkitt’s Lymphoma .‬‬ ‫פרק מספר ‪ :3‬המטואונקולוגיה‬

‫‪ .70‬אישה בת ‪ 59‬מגיעה לרופא מטפל עקב‬ ‫‪ .66‬גבר בן ‪ 65‬פונה למרפאה עקב חולשה‬
‫כאבי ראש‪ ,‬עייפות‪ ,‬סחרחורת וכאבי בטן‪.‬‬ ‫כללית‪ ,‬ירידה במשקל ‪ 6‬ק"ג מזה ‪5‬חודשים‪,‬‬
‫בבדיקה מתגלה יתר לחץ דם‪ .‬בבדיקת דם‬ ‫והגדלה של בלוטות לימפה בבתי שחי‬
‫היקפי‪ :‬עליה ניכרת בהמוגלובין ל‪20.4 g% -‬‬ ‫ובצוואר‪.‬‬
‫‪ ,‬עליה בטרומבוציטים‪ ,‬עליה בכדוריות‬ ‫בבדיקה גופנית נמצאה הגדלת טחול וכבד‬
‫לבנות )עם ספירה מבדלת תקינה‪ ,‬ללא תאים‬ ‫במידה קלה‪ .‬אין חום‪.‬‬
‫צעירים(‪.‬‬ ‫בבדיקת דם היקפי‪ :‬אנמיה קלה‪,‬‬
‫החולה נשלחה למרפאה המטולוגית‪ ,‬בוצעה‬ ‫טרומבוציטופניה קלה‪ ,‬לויקוציטים ‪25,000‬‬
‫ביופסיית לשד עצם‪.‬‬ ‫מהם ‪ 70%‬לימפוציטים‪.‬‬
‫בבדיקה היסטולוגית‪ ,‬תמונה מספר ‪-3‬‬ ‫בוצעה ביופסיה של קשר לימפה מוגדל‬
‫צביעת ‪. H&E‬‬ ‫בצוואר‪.‬‬
‫סמן את האבחנה הסבירה ביותר )תשובה‬ ‫בבדיקה היסטולוגית )צביעה ‪ -( H&E‬תמונה‬
‫אחת נכונה בלבד(‪:‬‬ ‫מספר ‪1‬‬
‫א‪Acute Myeloid Leukemia .‬‬ ‫בצביעה אימונוהיסטוכימית התאים נצבעו‬
‫ב‪Follicular Lymphoma .‬‬ ‫‪ CD20/CD5/CD23‬חיובי‪.‬‬
‫מהי האבחנה הסבירה ביותר?‬
‫ג‪Polycythemia Vera .‬‬
‫א‪Marginal Zone MALT Lymphoma .‬‬
‫ד‪Multiple Myeloma .‬‬
‫ב‪Chronic Lymphocytic Leukemia .‬‬
‫‪ .71‬סמן את התשובה הנכונה לגבי המחלה‬ ‫ג‪Hairy Cell Leukemie .‬‬
‫בתמונה מספר ‪:3‬‬ ‫ד‪Acute Myeloid Leukemia .‬‬
‫א‪ .‬ללא טיפול החולים בסכנת מוות‬
‫‪ .77‬הקריטריונים לאבחנה של‬ ‫מאוטמים עקב קרישיות יתר ו‪/‬או משטפי‬
‫‪ Polycythemia Vera‬כוללים את כל הבאים‬ ‫דם‬
‫פרט ל‪:-‬‬ ‫ב‪ .‬כדי לקבוע אבחנה של מחלה נדרשת‬
‫נוכחות של יותר מ‪ 20% -‬בלסטים בלשד‬
‫א‪ .‬נפח כדוריות הדם גבוה מ‪36 ml/kg -‬‬ ‫העצם‬
‫ב‪ .‬לויקוציטוזיס‬
‫ג‪ .‬תרומבוציטוזיס‬ ‫ג‪ .‬התא האופייני למחלה הוא ‪Reed‬‬
‫ד‪ .‬רמת אריתרופואטין נמוכה‬ ‫‪Sternberg‬‬
‫ה‪ .‬גרד‬ ‫ד‪ .‬לרוב החולים יש נוגדן מונוקלונלי בדם‬

‫‪ .78‬בתסמונת מיאלודיספלסטית הפקטורים‬ ‫‪ .72‬בוטל‬


‫הפרוגנוסטים הגרועים כוללים את כל‬
‫הבאים פרט ל‪:‬‬ ‫‪ .73‬נה נכון לגבי המחלה ע"ש הודג'קין‪:‬‬
‫א‪ .‬גיל גבוה‬ ‫א‪ .‬התא האבחנתי תמיד נצבע ב‪(CD45 -‬‬
‫ב‪ .‬מספר טסיות מתחת ל‪30,000 -‬‬ ‫‪Leukocyte Common Antigen) LCA‬‬
‫ג‪ 10% .‬בלסטים בלשד העצם‬ ‫ב‪ .‬הטחול מעורב ברוב המקרים‬
‫ד‪Complex Karyotype .‬‬ ‫ג‪ .‬בתת קבוצה הנקראת ‪Nodular sclerosis‬‬
‫ה‪ .‬ממצא של )‪(5q-‬‬ ‫המצאים האופיינים לתת סוג זה הם‬
‫פיברוזיס היאליני ונוכחות תאים גידוליים‬
‫‪ .79‬בוטל‬ ‫הנקראים ‪Lacunar cells‬‬
‫ד‪ .‬התאים ע"ש ‪ Reed Sternberg‬הם ברוב‬
‫‪ .80‬איזה מהמשפטים הבאים אינו נכון?‬
‫א‪ .‬על מנת שתא נורמלי יהפוך לממאיר‪,‬‬ ‫המקרים תאי לימפה ‪.T‬‬
‫צריכים להפגע בו מספר מסלולים‬
‫ביוכימיים‪ ,‬כדי שיהפוך בלתי תלוי בסיגנלים‬ ‫‪ .74‬סלייד‬
‫חיצוניים לפרוליפרציה‪ ,‬בלתי רגיש‬
‫לסיגנלים מעכבי גדילה‪ ,‬בעל פוטנציאל‬ ‫‪ .75‬גבר בן ‪ 70‬אובחן כסובל מ‪Follicular -‬‬
‫שכפול בלתי מוגבל ומסוגל להתחמק‬ ‫‪ Lymphoma‬בביופסיה של בלוטת לימפה‬
‫מאפופטוזיס‬ ‫במפשעה בצד שמאל‪ .‬מה נכון בהקשר‬
‫ב‪ .‬מתילציה מהווה מנגנון אפיגנטי חשוב‬ ‫למחלה?‬
‫בסרטן‪ ,‬בגורמה לאינאקטיבציה של ‪Tumor‬‬ ‫א‪ .‬יש סיכוי רב שבבדיקות לקביעת דרגת‬
‫‪ suppressor genes‬רבים‪.‬‬ ‫המחלה לא ימצאו אתרים נוספים‬
‫המעורבים במחלה‪.‬‬
‫ג‪ .‬חלבון ‪ Bcl2‬פועל בגרעין כפקטור שיעתוק‬ ‫ב‪ .‬לצורך קביעת דרגת המחלה מומלץ לבצע‬
‫המגביר קצב חלוקת תאים‬
‫בדיקת ‪ CT‬של הבטן‪ ,‬וניתן לוותר על בדיקת‬
‫ד‪ .‬לחלבון ‪ BCR/ABL‬יש פעילות של‬
‫טירוזין קינאז מוגברת ביחס לזו של חלבון‬ ‫‪ CT‬חזה‪.‬‬
‫ג‪ .‬מומלץ לבצע בדיקת לשד עצם כי יש סיכוי‬
‫‪ABL‬‬ ‫גבוה למעורבתו במחלה‬
‫‪ .81‬אילו מהמשפטים הבאים אינו נכון ?‬ ‫ד‪ .‬באם תמצא המחלה בשלב מתקדם )‪III-‬‬
‫א‪ .‬חלבון ‪ BCL2‬שונה בהרכב‬ ‫‪ (IV‬מומלץ לטפל במישלב כמותרפי כלשהו‪,‬‬
‫החומצות האמיניות שלו‬ ‫מתוך מספר המישלבים המקובלים‬
‫בלימפומה פוליקולרית ביחס‬ ‫ה‪ .‬שיעור התגובה לטיפול נמוך יותר בחולים‬
‫לחלבון זה בתאים בריאים‪.‬‬ ‫ם לימפומה במפשעה מאשר עם לימפומה‬
‫בצוואר‪.‬‬
‫ב‪ .‬בתאי סרטן רבים קיימת‬
‫פעילות של האנזים טלומראז‪,‬‬
‫המאפשרת להם יכולת שכפול‬ ‫‪ .76‬מה אינו נכון לגבי ‪Diffuse Large Cell‬‬
‫‪ DNA‬בלתי מוגבלת‪.‬‬ ‫‪? Lymphoma‬‬
‫א‪ .‬מהוים כ‪ 30% -‬מכלל הלימפומות‬
‫ג‪ .‬מרבית סוגי ‪ AML‬מתפתחים‬ ‫ב‪ .‬יכולה לערב מערכות גוף שונות כמו דרכי‬
‫ממוטציות המצטברות בתאי‬ ‫העיכול‬
‫גזע המטופוייטים‪.‬‬ ‫ג‪ .‬הפרוגנוזה גרועה בחולים צעירים‪ ,‬מעל גיל‬
‫ד‪ .‬ב‪Burkitt’s Lymphoma -‬‬ ‫‪ ,60‬עם ‪ LDH‬בדם הנמוך מהנורמה‬
‫משופעל האונקוגן ‪ myc‬דרך‬ ‫ד‪ .‬טיפול כמותרפי במישלב ‪ CHOP‬מרפא‬
‫טרנסלוקציה לאחד הגנים‬ ‫כ‪ 40% -‬מהחולים עם מחלה בדרגה‬
‫לאימונוגלובולינים‪.‬‬
‫מתקדמת ‪III-IV‬‬
‫‪ .82‬בת ‪ 68‬הובאה למיון ע"י בנה עקב חולשה‬
‫‪ .87‬בוטל‬ ‫הולכת וגוברת בחודש האחרון‪.‬‬
‫ברקע‪ :‬כאב גב תחתון מזה ‪ 3‬חודשים‪.‬‬
‫‪ .88‬בחר את התשובה הנכונה‪:‬‬ ‫בבדיקה‪ :‬פרט לחוורון בולט – אין ממצא‬
‫א‪ CML .‬היא מחלה המאופיינת במהלך‬ ‫מיוחד‪.‬‬
‫פאזי‪,‬לכן בפאזה הכרונית אין צורך בטיפול‬ ‫בבדיקות העזר‪WBC ,PLT 250,000 :‬‬
‫ב‪ .‬הפאזה המחייבת טיפול היא ‪Blast‬‬ ‫‪.Hb 9.0 ,4,500‬‬
‫‪Crisis‬‬ ‫כימיה‪albumin total protein 6.2g% :‬‬
‫ג‪ .‬כל חולה שאובחן כסובל מ‪ CML -‬חייב‬ ‫)‪Creatinin 7.8mg% (norm , (norm 6-7‬‬
‫לעבור השתלת מח עצם מתורם מייד עם‬ ‫)‪ , 0.9-1.3‬בצילום עמ"ש נמצא נגע ליטי‬
‫האבחנה‬ ‫בחוליה ‪ .L4‬בחשד ל‪Multiple Myeloma -‬‬
‫ד‪ .‬הטיפול בחולה עם ‪ CML‬חייב להתחיל‬ ‫בוצעה בדיקת פרוטאין אלקטרופורזיס‪,‬‬
‫מייד עם האבחנה בפאזה הכרונית‬ ‫אימונואלקטרופורזה ונמצאה היפוגמה‪-‬‬
‫גלובולינמיה‪.‬‬
‫‪ .89‬בחר את התשובה הנכונה‪:‬‬ ‫מה מבין המשפטים הבאים נכון‪:‬‬
‫א‪ .‬הטיפול המקובל היום בפאזה הכרונית‬ ‫א‪ .‬התוצאה של בדיקת אלקטרופורזה‬
‫הוא כימותרפיה פומית‬ ‫שוללת אבחנה של מילומה‬
‫ב‪ .‬אינטרפרון נותן ‪ 90%‬תגובה ציטוגנטית‬ ‫ב‪ .‬אי ספיקת כליות קשורה לפרוגנוזה של‬
‫בחולי ‪CML‬‬ ‫מחלה‬
‫ג‪ Glivac .‬הינו מעכב ספציפי של טירוזין‬ ‫ג‪ .‬בדיקת ‪ BJ‬בשתן תהיה קרוב לוודאי‬
‫קינאז‬ ‫חיובית‬
‫ד‪ Glivac .‬נותן מעל ‪ 80%‬תגובה ציטוגנטית‬ ‫ד‪ .‬אין צורך לבצע בדיקת לשד עצם כיוון‬
‫בחולי ‪ CML‬בפאזה כרונית ראשונה‬ ‫שיש מספיק פקטורים התומכים באבחנה‬
‫ה‪ .‬ג‪+‬ד‬
‫‪ .83‬הפרוגנוזה של ‪Multiple Myeloma‬‬
‫‪ .90‬מהו סוג הטיפול הקרינתי העדיף בחולי‬ ‫תלויה במספר גורמים פרט ל‪:-‬‬
‫סרטן הלבלב?‬ ‫א‪ .‬הפרעות ציטוגנטיות המערבות‬
‫א‪ .‬ברכיתרפיה של הלבלב בלבד‬ ‫כרומוזומים ‪14 ,13‬‬
‫ב‪ .‬טיפול קרינתי חיצוני עם אלקטרונים עם‬ ‫ב‪ .‬המצאות ‪Plasmacytoma‬‬
‫רמת אנרגיה ‪18 Mev‬‬ ‫ג‪ .‬רמת ‪ Microglobulin 2‬באבחנה‬
‫ג‪ .‬טיפול קרינתי חיצוני עם פוטונים עם רמת‬ ‫ד‪ CRP .‬גבוה באבחנה‬
‫אנרגיה ‪18 Mev‬‬
‫ד‪ .‬טיפול קרינתי משולב עם טיפול הורמונלי‬ ‫‪ .84‬הסימן לנזק בתפקוד השחלתי הוא‬
‫הגבוה ביותר בטיפול באיזו מהתרופות‬
‫פרק מספר ‪ :4‬אשך ופרוסטטה‬ ‫הבאות?‬
‫א‪ .‬ציקלופוספמיד‬
‫‪ .91‬מהו המיקום השכיח ביותר של סרטן‬ ‫ב‪ .‬מטוטרקסאט‬
‫הערמונית?‬ ‫ג‪ .‬וינקרסטין‬
‫א‪Anterior zone .‬‬ ‫ד‪ .‬גמציטבין‬
‫ב‪Central Zone .‬‬
‫ג‪Peripheral zone .‬‬ ‫‪ .85‬איזו מהתרופות הבאות היא ‪?pro-drug‬‬
‫ד‪Periurethral zone .‬‬ ‫א‪doxorubicin (adriamycin) .‬‬
‫ה‪Transition zone .‬‬ ‫ב‪ifosfamide .‬‬
‫ג‪vinorelbine .‬‬
‫‪ .92‬בבדיקה רקטלית של גבר בן ‪ ,70‬התגלה‬ ‫ד‪.‬‬
‫גוש נוקשה בערמונית‪ .‬בבדיקה‬
‫מיקרוסקופית של ביופסיית מחט נראו‬ ‫‪ .86‬מהו מנגנון הפעולה של‬
‫בלוטות קטנות עם אטיפיה גרעינית‪,‬‬ ‫)‪ metoclopramide (pramin‬במניעת‬
‫וגרעינים גדולים‪ .‬בצביעה לתאים בזאלים‪,‬‬ ‫בחילה והקאה כתוצאה מכימותרפיה?‬
‫לא נצפתה צביעה‪ .‬מהי האבחנה?‬ ‫א‪ .‬חסימת רצפטורים של דופמין‬
‫א‪seminoma .‬‬ ‫ב‪ .‬חסימת רצפטורים של סרטונין‬
‫ב‪benign prostatic hyperplasia .‬‬ ‫ג‪ .‬חסימת רצפטורים של ‪neurokinin-1‬‬
‫ג‪metastatic carcinoma .‬‬ ‫ד‪ .‬חסימת רצפטורים של‬
‫ד‪adenocarcinoma of the prostate .‬‬ ‫קורטיקוסטרואידים‬
‫א‪ .‬יותר יעיל מטיפול ע"י אחת התרופות‬ ‫ה‪chronic prostatitis .‬‬
‫בלבד‬
‫ב‪ .‬מקטין את סכנת ההתקדמות של ה‪-‬‬ ‫‪ .93‬הסמן בדם ובתאים של ‪Yolk sac‬‬
‫‪BHP‬‬ ‫‪ tumor‬הוא ‪) :‬להשלים(‬
‫ג‪ .‬מקטין את סכנת התפתחות חסימה‬ ‫_______________________________‬
‫חריפה )‪( acute retention‬‬ ‫__________________________‬
‫ד‪ .‬אינו משפיע על הצורך בניתוח‬
‫‪ .94‬הסמן בדם ובתאים של‬
‫‪ .101‬בהגדלה שפירה של הערמונית )‪( BHP‬‬ ‫‪) : Choriocarcinoma‬להשלים(‬
‫מה אינו נכון ?‬ ‫_______________________________‬
‫א‪ .‬גיל מתקדם מהווה גורם סיכון‬ ‫__________________________‬
‫להתפתחות ‪BHP‬‬
‫ב‪ .‬בחולים עם ‪ PHP‬בדיקה פיזיקלית‬ ‫‪ .95‬גידול בערמונית ב‪clinical stage T1 -‬‬
‫מספקת לרוב יותר מידע מהאנמנזה‬ ‫מתייחס ל‪:-‬‬
‫ג‪ .‬קיים קשר בין תפקוד תקין של האשכים‬ ‫א‪ .‬גידול נמוש בבדיקה רקטלית פחות מ‪1-‬‬
‫והתפתחות ‪BHP‬‬ ‫ס"מ‬
‫ד‪ .‬שכיחות מוקדים היסטולוגים של ‪BHP‬‬ ‫ב‪ .‬גידול שנתגלה באקראי בחולה שנותח‬
‫מגיעה לכ‪ 80% -‬בגברים בגיל ‪80‬‬ ‫בשל ‪BHP‬‬
‫ג‪ .‬גידול שאינו פורץ את הקופסית‬
‫‪ .102‬גבר בן ‪ 68‬מתלונן על זרם שתן חלש‬ ‫ד‪ .‬פריצה מקומית קטנה של הקופסית ע"י‬
‫והרגשת חוסר התרוקנות‪ .‬בבדיקה ערמונית‬ ‫הגידול‬
‫מוגדלת לא חשודה לגידול‪ .‬בדיקת זרם שתן‬
‫מראה עקומה שטוחה‪.‬‬ ‫‪ .96‬בחולה בן ‪ PSA 50‬בסרום ‪ . 9ng/ml‬גודל‬
‫האבחנה המבדלת בחולה זה כוללת את‬ ‫הערמונית בנפח של ‪ 40‬סמ"ק‪ .‬בבדיקה‬
‫הבאים פרט ל‪:-‬‬ ‫רקטלית של הערמונית ‪ 2+‬מוצקות רגילה‪.‬‬
‫א‪ .‬חסימת מוצא שלפוחית השתן ע"י סרטן‬ ‫הפרעות קלות בהטלת שתן‪ ,‬ללא שארית‬
‫הערמונית בשלב מתקדם‬ ‫שתן או הידרונפרוסיס‪.‬‬
‫ב‪ .‬זיהום דרכי השתן‬ ‫הצעד הדיאגנוסטי החשוב הבא‪:‬‬
‫ג‪ .‬הגדלה שפירה של הערמונית‬ ‫א‪ .‬ביופסיה מונחת סונאר של הערמונית‬
‫ד‪ .‬גידול של שלפוחית השתן הממוקם‬ ‫ב‪ US .‬כליות ודרכי שתן כולל שארית לאחר‬
‫בצוואר השלפוחית‬ ‫התרוקנות‬
‫ה‪ .‬היצרות בשופכה‬ ‫ג‪ .‬מיפוי עצם‬
‫ד‪ .‬מדידת יחס בין הפרקציה החופשית של‬
‫‪ .103‬איזה מהתרופות הרשומות ניתנות‬
‫לחולים המתלוננים על הפרעות חסימתיות‬ ‫‪ PSA‬לרמה הכללית של ‪PSA‬‬
‫בדרכי השתן?‬
‫‪ .97‬ניקוז הדם דרך ורידי האשך נעשה‬
‫א‪alpha adrenergic stimulators .‬‬ ‫בעיקר דרך‪:‬‬
‫ב‪alpha adrenergic blockers .‬‬ ‫א‪ .‬וריד הכסל המשותף‬
‫ג‪beta adrenergic blockers .‬‬ ‫ב‪ .‬וריד הכסל החיצוני‬
‫ד‪phosphodiasterase inhibitors .‬‬ ‫ג‪ .‬פלקסוס פמפינופורמי‬
‫ה‪calcium channel blockers .‬‬ ‫ד‪ .‬הוריד הפמורלי‬

‫‪ .104‬המרכיבים האנטומיים הבאים‬ ‫‪ .98‬בוטל‬


‫אחראים ליצירת התנגודת )‪( resistance‬‬ ‫‪ .99‬מה מהמשפטים הבאים אינו נכון לגבי‬
‫במוצא שלפוחית השתן ולכן משתתפים‬ ‫יצור טסטוסטרון?‬
‫במנגנון השליטה‪:‬‬ ‫א‪ .‬מיוצר ע"י תאי סרטולי באשך‬
‫א‪ .‬השופכה הבולברית בלבד‬
‫ב‪ .‬השופכה הפרוסטטית בלבד‬ ‫ב‪ .‬יצורו מתווך ע"י הורמון ה‪LH -‬‬
‫ג‪ .‬כל השופכה‬ ‫ג‪ .‬מקורו מכולסטרול‬
‫ד‪ .‬השופכה הפרוסטטית והממברנוזית‬ ‫ד‪ .‬כ‪ 5% -‬מהטטוסטרון בסרום נוצר‬
‫מבלוטת יותרת הכליה‬
‫פרק ‪ :5‬זיהומיות‬
‫‪ .100‬לגבי טיפול משולב בתרופות מקבוצת‬
‫‪ .105‬איזה מהחיידקים הבאים אינו משתייך‬ ‫מעכבי ‪ 5 alpha reductase‬וחוסמי אלפא‬
‫לקבוצת ה‪? HACEK -‬‬ ‫מה אינו נכון?‬
‫‪ .112‬מבין השיטות למניעתת הפצת חיידקים‬ ‫א‪Hemophilus .‬‬
‫עמידים מה חשוב ביותר ?‬ ‫ב‪Actinobacillus .‬‬
‫א‪ .‬לתת טיפול אנטיביוטי רחב‬
‫ב‪ .‬בידוד החולים בחדרים נפרדים‬ ‫ג‪Cardiobacterium .‬‬
‫ג‪ .‬רחצת ידיים לפני ואחרי כל מגע עם חולה‬ ‫ד‪E.coli .‬‬
‫ד‪ .‬רחצת גוף החולה עם כלורהקסדין‬ ‫ה‪kingella .‬‬
‫ה‪ .‬שימוש של תכשיר אנטיביוטי באף כדי‬
‫למוע קולוניזציה‬ ‫‪ .106‬בוטל‬

‫‪ .113‬הגדרה של בקטריוריה אסימפטומטית‬ ‫‪ .107‬משך הטיפול האנטיביוטי המומלץ‬


‫מתייחסת לחולה עם‪:‬‬ ‫לטיפול ב‪Prosthetic valve endocarditis -‬‬
‫א‪ .‬תרבית שתן חיובית המכילה ‪ 104‬או יותר‬ ‫הוא‪:‬‬
‫מושבות‪ /‬סמ"ק שתן‬ ‫א‪ .‬שבועיים‬
‫‪2‬‬
‫ב‪ .‬תרבית שתן חיובית המכילה ‪ 10‬או יותר‬ ‫ב‪ 3 .‬שבועות‬
‫מושבות‪ /‬סמ"ק שתן‬ ‫ג‪ 4 .‬שבועות‬
‫ג‪ .‬אם שתי תרביות שתן רצופות‪ ,‬הראשונה‬ ‫ד‪ 6 .‬שבועות‬
‫מכילה ‪ 105‬מושבות‪ /‬סמ"ק שתן והשניה‬ ‫ה‪ .‬כל התשובות נכונות‬
‫שלילית‬
‫ד‪ .‬שתי תרביות שתן עם אותו חיידק‪,‬‬ ‫‪ .108‬מי הם המחוללים השכיחים ביותר ל‪-‬‬
‫ובכמות השווה או יותר מ‪ 105 -‬מושבות‪/‬‬ ‫‪ Native valve endocarditis‬שנרכשו‬
‫סמ"ק שתן‬ ‫בקהילה|?‬
‫א‪Candida, Aspargillus .‬‬
‫‪ Staphylococcus saprophyticus .114‬הוא‬ ‫ב‪Pseudomonas, Pneumococcus .‬‬
‫חיידק‪:‬‬ ‫ג‪Streptococci, staphylococcus Aureus .‬‬
‫א‪ .‬שכיח ב‪ UTI -‬בנשים מבוגרות‬ ‫ד‪kingella, Eikenella .‬‬
‫ב‪ .‬שכיח ב‪ UTI -‬בנשים צעירות בארצות‬ ‫ה‪Bartonella, Brucella .‬‬
‫הברית‬
‫ג‪ .‬שכיח כגורם ל‪ UTI -‬על רקע של אבנים‬ ‫‪ .109‬כל המחוללים הבאים גורמים למחלות‬
‫בדרכי השתן‬ ‫המועברות במגע מיני פרט ל‪:-‬‬
‫ד‪ .‬שכיח ב‪ UTI -‬בנשים צעירות בישראל‬ ‫א‪ .‬הפטטיס ‪B‬‬
‫ה‪ .‬חיידק "קונטמינור" ללא משמעות‬ ‫ב‪ .‬הפטטיס ‪C‬‬
‫קלינית‬ ‫ג‪ .‬נגיף הפפילומה‬
‫ד‪ .‬סטרפטוקוק אגלקטיה‬
‫ה‪HIV .‬‬

‫‪ .110‬טיפול אמפירי התחלתי בחולים‬


‫ניוטרופנים עם חום אמור לכסות‪:‬‬
‫א‪ .‬קוקים גרם חיוביים כולל ‪MRSA‬‬
‫ב‪ .‬מתגים גרם שליליים כולל אנאירובים‬
‫ג‪ .‬מתגים גרם שליליים ו‪-‬וירוסים‬
‫ד‪ .‬מתגים גרם שליליים ופטריות‬
‫ה‪ .‬קוקים גרם חיוביים‪ ,‬מתגים גרם‬
‫שליליים כולל פסאודומונס‬

‫‪ .111‬בילד בן שלוש נצפו נגעים שטחיים‬


‫מכוסים בגלד בצבע דבש המתפשטים על‬
‫הפנים‪ .‬טיפול אנטיביוטי מתאים יכול למנוע‬
‫כל אחד מהסיבוכים הפוטנציאלים הבאים‬
‫של זיהום זה‪ ,‬פרט ל‪:-‬‬
‫א‪ .‬גלומרולונפריטיס‬
‫ב‪Necrotizing Fascitis .‬‬
‫ג‪Rheumatic Fever .‬‬
‫ד‪ .‬אוסטאופורוסיס‬
‫ה‪Toxic shock syndrome .‬‬
‫תשובות‬
‫ה‬ ‫‪77‬‬ ‫ה‬ ‫‪39‬‬ ‫ב‬ ‫‪1‬‬
‫ה‬ ‫‪78‬‬ ‫‪Fibroadenoma‬‬ ‫‪40‬‬ ‫ג‬ ‫‪2‬‬
‫‪79‬‬ ‫ה‬ ‫‪41‬‬ ‫ג‬ ‫‪3‬‬
‫סלייד – ‪hashimoto‬‬
‫ג‬ ‫‪80‬‬ ‫א‬ ‫‪42‬‬ ‫‪) thyroiditis‬בוטל(‬ ‫‪4‬‬
‫א‬ ‫‪81‬‬ ‫בוטל‬ ‫‪43‬‬ ‫ג‬ ‫‪5‬‬
‫ג‬ ‫‪82‬‬ ‫ג‬ ‫‪44‬‬ ‫ד‬ ‫‪6‬‬
‫ב‬ ‫‪83‬‬ ‫א‬ ‫‪45‬‬ ‫א‬ ‫‪7‬‬
‫א‬ ‫‪84‬‬ ‫ב‬ ‫‪46‬‬ ‫ד‬ ‫‪8‬‬
‫ב‬ ‫‪85‬‬ ‫ד‬ ‫‪47‬‬ ‫ב‬ ‫‪9‬‬
‫א‬ ‫‪86‬‬ ‫ד‬ ‫‪48‬‬ ‫א‬ ‫‪10‬‬
‫‪87‬‬ ‫א‬ ‫‪49‬‬ ‫‪Neuroblastoma‬‬ ‫‪11‬‬
‫ד‬ ‫‪88‬‬ ‫‪50‬‬ ‫ה‬ ‫‪12‬‬
‫ה‬ ‫‪89‬‬ ‫‪51‬‬ ‫א‬ ‫‪13‬‬
‫ג‬ ‫‪90‬‬ ‫ב‬ ‫‪52‬‬ ‫א‬ ‫‪14‬‬
‫ד‬ ‫‪91‬‬ ‫ה‬ ‫‪53‬‬ ‫ד‬ ‫‪15‬‬
‫ד‬ ‫‪92‬‬ ‫ב‬ ‫‪54‬‬ ‫ד‬ ‫‪16‬‬
‫‪a-feto-protein‬‬ ‫‪93‬‬ ‫ב‬ ‫‪55‬‬ ‫א‬ ‫‪17‬‬
‫‪B-HCG‬‬ ‫‪94‬‬ ‫ה‬ ‫‪56‬‬ ‫ד‬ ‫‪18‬‬
‫ב‬ ‫‪95‬‬ ‫ד‬ ‫‪57‬‬ ‫ד‬ ‫‪19‬‬
‫א‬ ‫‪96‬‬ ‫ד‬ ‫‪58‬‬ ‫ב‬ ‫‪20‬‬
‫ג‬ ‫‪97‬‬ ‫א‬ ‫‪59‬‬ ‫ד‬ ‫‪21‬‬
‫‪98‬‬ ‫ב‬ ‫‪60‬‬ ‫ב‬ ‫‪22‬‬
‫א‬ ‫‪99‬‬ ‫א‬ ‫‪61‬‬ ‫ד‬ ‫‪23‬‬
‫ד‬ ‫‪100‬‬ ‫ד‬ ‫‪62‬‬ ‫א‬ ‫‪24‬‬
‫ב‬ ‫‪101‬‬ ‫ד‬ ‫‪63‬‬ ‫א‬ ‫‪25‬‬
‫ב‬ ‫‪102‬‬ ‫בוטל‬ ‫‪64‬‬ ‫ג‬ ‫‪26‬‬
‫ב‬ ‫‪103‬‬ ‫ב‬ ‫‪65‬‬ ‫ב‬ ‫‪27‬‬
‫ד‬ ‫‪104‬‬ ‫ב‬ ‫‪66‬‬ ‫ג‬ ‫‪28‬‬
‫ד‬ ‫‪105‬‬ ‫ב‪+‬ד‬ ‫‪67‬‬ ‫ה‬ ‫‪29‬‬
‫‪106‬‬ ‫ג‬ ‫‪68‬‬ ‫ג‬ ‫‪30‬‬
‫ד‬ ‫‪107‬‬ ‫ד‬ ‫‪69‬‬ ‫ג‬ ‫‪31‬‬
‫ג‬ ‫‪108‬‬ ‫ג‬ ‫‪70‬‬ ‫ג‬ ‫‪32‬‬
‫ד‬ ‫‪109‬‬ ‫א‬ ‫‪71‬‬ ‫ד‬ ‫‪33‬‬
‫ה‬ ‫‪110‬‬ ‫‪72‬‬ ‫ג‬ ‫‪34‬‬
‫א‬ ‫‪111‬‬ ‫ג‬ ‫‪73‬‬ ‫ג‬ ‫‪35‬‬
‫ג‬ ‫‪112‬‬ ‫‪HL – nodular sclerosis‬‬ ‫‪74‬‬ ‫‪Leimyoma‬‬ ‫‪36‬‬
‫ד‬ ‫‪113‬‬ ‫ג‬ ‫‪75‬‬ ‫ד‬ ‫‪37‬‬
‫ב‬ ‫‪114‬‬ ‫ג‬ ‫‪76‬‬ ‫בוטל‬ ‫‪38‬‬

You might also like